Venus Envy

By Steve Goddard

ESA’s Venus Express mission has been studying the planet and a basic atmospheric model is emerging.

Venus Express probe – Image: European Space Agency
https://i0.wp.com/astronomyonline.org/SolarSystem/Images/Venus/VenusClouds_th.jpg?w=1110
Atmospheric model – Image: Astronomyonline.org – click for more

Venus has long been the CO2 bogeyman of climate science.  In my last piece about Venus I laid out arguments against the claim that it is a runaway greenhouse which makes Venus hot. This generated a lot of discussion. I’m not going to review that discussion, but instead will pose a few ideas which should make the concepts clear to almost everybody.

If there were no Sun (or other external energy source) atmospheric temperature would approach absolute zero. As a result there would be almost no atmospheric pressure on any planet -> PV = nRT.

Because we have a sun providing energy to the periphery of the atmospheric system, the atmosphere circulates vertically and horizontally to maintain equilibrium. Falling air moves to regions of higher pressure, compresses and warms. The greater the pressure, the greater the warming. Rising air moves to regions of lower pressure, expands, and cools. The amount of warming (or cooling) per unit distance is described as the “lapse rate.” On Earth the dry lapse rate is 9.760 K/km. On Venus, the dry lapse rate is similar at 10.468 K/km. This means that with each km of elevation you gain on either Earth or Venus, the temperature drops by about 10C.

It is very important to note that despite radically different compositions, both atmospheres have approximately the same dry lapse rate. This tells us that the primary factor affecting the temperature is the thickness of the atmosphere, not the composition. Because Venus has a much thicker atmosphere than Earth, the temperature is much higher.

dT = -10 * dh     where T is temperature and h is height.

With a constant lapse rate, an atmosphere twice as thick would be twice as warm. Three times as thick would be three times as warm. etc. Now let’s do some experiments using this information.

Experiment # 1 – Atmospheric pressure on Venus’ surface is 92 times larger than earth, because the atmosphere is much thicker and thus weighs more.  Now suppose that we could instantly change the molecular composition of Venus atmosphere to match that of Earth. Because the lapse rate of Earth’s atmosphere is very similar to that of Venus, we would see little change in Venus temperature.

Experiment #2 – Now, lets keep the atmospheric composition of Venus constant, but instead remove almost 91/92 of it – to make the mass and thickness of Venus atmosphere similar to earth. Because lapse rates are similar between the two planets, temperatures would become similar to those on earth.

Experiment #3 – Let’s take Earth’s atmosphere and replace the composition with that of Venus. Because the lapse rates are similar, the temperature on Earth would not change very much.

Experiment #4 – Let’s keep the composition of Earth’s atmosphere fixed, but increase the amount of gas in the atmosphere by 92X. Because the lapse rates are similar, the temperature on Earth would become very hot, like Venus.

Now let’s look at measured data :

Temperatures within Venus's atmosphere

http://www.astro.wisc.edu/~townsend/resource/teaching/diploma/venus-t.gif

Pressures within Venus's atmosphere

http://www.astro.wisc.edu/~townsend/resource/teaching/diploma/venus-p.gif

Note that at one Earth atmospheric pressure on Venus (altitude 50km) temperatures are only about 50 degrees warmer than earth temperatures. This is another indication that atmospheric composition is less important than thickness.

Conclusions : It isn’t the large amount of CO2 which makes Venus hot, rather it is the thick atmosphere being continuously heated by external sources. It isn’t the lack of CO2 on Earth which keeps Earth relatively cool, rather it is the thin atmosphere. Mars is even colder than earth despite having a 95% CO2 atmosphere, because it’s atmosphere is very thin. If greenhouse gases were responsible for the high temperatures on Venus (rather than atmospheric thickness) we would mathematically have to see a much higher lapse rate than on Earth – but we don’t.

WUWT commentor Julian Braggins provided a very useful link which adds a lot of important information.

“The much ballyhooed greenhouse effect of Venus’s carbon dioxide atmosphere can account for only part of the heating and evidence for other heating mechanisms is now in a turmoil,” confirmed Richard Kerr in Science magazine in 1980.

The greenhouse theory does not explain the even surface temperatures from the equator to the poles: “atmospheric temperature and pressure in most of the atmosphere (99 percent of it) are almost identical everywhere on Venus – at the equator, at high latitudes, and in both the planet’s day and night hemispheres. This, in turn, means the Venus weather machine is very efficient in distributing heat evenly,” suggested NASA News in April 1979. Firsoff pointed out the fallacy of the last statement: “To say that the vigorous circulation (of the atmosphere) smooths out the temperature differences will not do, for, firstly, if these differences were smoothed out the flow would stop and, secondly, an effect cannot be its own cause. We are thus left with an unresolved contradiction.”

======================================================

An update for those interested in what Venus looks like at the surface.

https://i0.wp.com/www.donaldedavis.com/BIGPUB/V13CLR2.jpg?w=1110

On March 1, 1982, the Soviet Venera 13 lander survived for 127 minutes (the planned design life was 32 minutes) in an environment with a temperature of 457 °C (855 °F) and a pressure of 89 Earth atmospheres (9.0 MPa). The photo composite above shows the soil and rocks near the lander.

Here’s another Venera image that shows a hint of yellow atmosphere. – Anthony

4 1 vote
Article Rating
475 Comments
Oldest
Newest Most Voted
Inline Feedbacks
View all comments
kwik
May 8, 2010 2:13 pm

What disturbs me is the following;
Personally I havent given Venus much thought before reading Steven Goddards posts here at WUWT. Why should I have. I have enough other thinks to worry about on a daily basis.
But now, reading about Venus, thinking about it, the grey cells working…over at Stephen Wildes post…it stands clear to me, at least for now….that, hey, it makes sense!
And then the disturbing thought;
Surely J. Hansen must know this too? Surely? I mean, if its your job to use the grey cells on such matters?
Surely?

John caten
May 8, 2010 2:16 pm

Haha, love the title! Now I’ll go back and read the article.

Troels Halken
May 8, 2010 2:32 pm

When you compress or decompress a gas, it either warms or cools. Due to the temperature differential to the surroundings, after some time it will cool or warm by exchanging heat with the surroundings until the temperature differential is approaching zero and hence the temperature of the gas approaches that of the surroundings.
Why should Venus behave any different?

Leonard Weinstein
May 8, 2010 2:38 pm

Steve,
You are correct on all the cases you quoted, but the basic reason Venus is hot is in fact due to the presence of greenhouse gasses combined with the high pressure. It is not how much of the gas is greenhouse gasses, but the fact that there is at least some present (even <1% CO2 or water vapor and 99% N2 would do about the same). If there were no greenhouse gas at all, but the gas pressure were still high, a lapse rate would still be present (but modified due to real gas effects and greatly different temperatures), but the surface of Venus would then be the location where all of the radiation to space would balance incoming absorbed energy. This would make the surface close to Earth's temperature, and the lapse rate would result in a very cold upper atmosphere. The presence of even a small amount of greenhouse gasses moves the location of the radiation that goes to space out to the edge of the atmosphere, and the lapse rate heats the surface. The main difference is where the radiation to space occurs.

Ian l. McQueen
May 8, 2010 2:40 pm

Experiment # 1 – Atmospheric pressure on Venus’ surface is 92 times larger than earth, because the atmosphere is much thicker and thus weighs more.
I am always puzzled by the statement that the atmospheric pressure on the surface of Venus is a large multiple of that on earth.
The MW of earth’s atmosphere is about 29, and that of Venus around 44. But gravity at the surface of Venus is only 0.904 that of earth.
Atmospheric pressure is the result of the weight of a column of gas above the surface pressing down. I won’t embarrass myself by trying to calculate the effect of higher MW and lower force due gravity on Venus, and compressibility of the gas(es), but I would expect the atmosphere of Venus to extend out a large multiple of the distance that it does on earth to produce an atmospheric pressure 92X that on earth. But I have not heard that the Venusian atmosphere extends out so far…..
So, the question remains why the atmosphere is so “thick”. It is not constrained to a fixed volume, but can expand as the mass of gas is increased.
A further niggle comes from the statement: “If there were no Sun (or other external energy source) atmospheric temperature would approach absolute zero. As a result there would be almost no atmospheric pressure on any planet -> PV = nRT.”
At 0K all gases would have condensed to liquid or solid, but the mass of the gas would still be present as a layer on the surface. I would think that the liquid/solid would be as heavy (=attraction due to gravity) as the atmosphere from which it was produced, and possibly a tiny bit higher since the molecules would be concentrated at the surface instead of extending a distance above the planet and thus minutely farther from the center of attraction of the mass of the planet.

kcom
May 8, 2010 2:46 pm

It’s always been clear to me from the timeline of the discoveries on Venus and the genesis of the global warming panic that a strong argument can be made that the scare aspects of global warming (its original and most honest name) are an example of a massive case of medical students’ disease. James Hansen got his nose so deep into Venus he couldn’t see his way out. Just because it’s not real, doesn’t mean it doesn’t seem real. The mind is a powerful thing and can overwhelm logic when placed under enough stress. Ask any medical student if this has ever happened to them.

May 8, 2010 2:55 pm

Leonard Weinstein
You are correct.
Earth already has plenty of greenhouse gases, so my point is that even if earth went to 100% CO2, the maximum temperature increase would be less than 36C – nothing like Venus. (In reality, it would get cooler because of the loss of water vapour.)

May 8, 2010 2:57 pm

Ian l. McQueen
You can infer how thick the atmosphere of Venus is from this diagram:
http://www.astro.wisc.edu/~townsend/resource/teaching/diploma/venus-t.gif

May 8, 2010 2:59 pm

Ian l. McQueen
Liquids are not very compressible and don’t heat up much under pressure. Consider the bottom of a swimming pool.

May 8, 2010 3:01 pm

On Earth we have water that Venus does not have. We have a wet lapse rate that is less than a dry lapse rate because of the processes of evaporation and condensation. Condensed moisture also controls the atmospheric concentration of CO2. If we had no atmospheric moisture, CO2 concentrations could be higher and the atmosphere could be heavier. If Venus ever had water like Earth, it must of boiled off because it was too close to the Sun and not in the sweet zone that sustains life here.

May 8, 2010 3:06 pm

John caten
Title is Anthony’s idea. He also added the ESA and Venera content.

rbateman
May 8, 2010 3:29 pm

One of the great things that NASA did was the planetary explorer missions. Without comparison to other planets, there would be little hope of understanding Earth. It is unfortunate that some have hypochondriated that splendid knowledge into doomsday scenarios. Earth, like the other planets, has been as it is for at least the last billion years. The foundation for what separates Earth from the other planets is Carbon-based life that transformed the place. Else, who knows what it might have become, or not become. Better start thinking about taking care of Earth’s precious Carbon, as geologic forces are relentlessly conspiring to sequester it.
I’m thinking of a Carbon Appreciation Day.

RobJM
May 8, 2010 3:32 pm

In addition to pressure, Venus also has high concentrations of other greenhouse gasses such as carbon monoxide. If one of these gasses absorbs at the frequency corresponding to earths atmospheric window then it could have a significant effect on temp. I also wonder if atmospheric pressure could explain the faint sun warm earth paradox.

May 8, 2010 3:36 pm

Leonard Weinstein says:
May 8, 2010 at 2:38 pm
… The presence of even a small amount of greenhouse gasses moves the location of the radiation that goes to space out to the edge of the atmosphere, and the lapse rate heats the surface. The main difference is where the radiation to space occurs.

That’s interesting Leonard. It would seem to a layman like me that a small amount of green house gas would radiate a small amount of energy back into space at the effective altitude of the gas. Are you saying that just a small amount of GHG at some altitude implies a large amount of re radiated energy is leaving the planet at that same altitude? I’m having trouble understanding that non-linear relationship between gas density and radiated energy.

Dave McK
May 8, 2010 3:40 pm

Willis- the lapse rate of earth’s atmosphere is NOT the lapse rate of a dry atmosphere.
In fact, the profile of earth’s atmosphere is that of a refrigerant system because of the working fluid water.
Venus has no such phase change gases in its atmosphere so it has the dry lapse rate.
The lapse rate of earth’s atmosphere is definitely not that of a dry atmosphere.
The are completely different- not similar at all.

IanH
May 8, 2010 3:54 pm

The modeling of Venus and earth atmospheres was reported (1,2): The authors agree adiabatic lapse rate is the crucial physical process. They point out three things
1) the adiabatic exponent for a hypothetical CO2 atmosphere replacing a nitrogen oxygen one of the same mass would result in a cooling of 6.4C.
2) Adding Carbon to oxygen in the atmosphere makes it slightly heavier, in principle resulting in tiny warming
3)Ultimately the CO2 dissolves in seas forming carbonate rocks, oxygen is thus lost from the atmosphere and atmospheric mass drops causing (tiny)cooling.
They conclude:
“Accumulation of large amounts of carbon dioxide in the atmosphere leads to the cooling, and not to warming of climate…. This conclusion has a simple physical
explanation: when the infrared radiation is absorbed by the molecules of greenhouse
gases, its energy is transformed into thermal expansion of air, which causes convective
fluxes of air masses restoring the adiabatic distribution of temperature in the troposphere.”

FWIW I am not a qualified climate scientist, take my reading with a pinch of salt.
1)
Environ Geol (2008) 54:1567–1572
Response to W. Aeschbach-Hertig rebuttal of ‘‘On global forces
of nature driving the Earth’s climate. Are humans involved?’’
by L. F. Khilyuk and G. V. Chilingar
2)
Energy Sources, Part A, 30:1–9, 2008
Cooling of Atmosphere Due to CO2 Emission
G. V. Chilingar a; L. F. Khilyuk a;O. G. Sorokhtin b
a Rudolf W. Gunnerman Energy and Environment Laboratory, University of Southern California, Los
Angeles, California, USA b Institute of Oceanology of Russian Academy of Sciences, Moscow, Russia

May 8, 2010 4:04 pm

Leonard Weinstein says:
. . . but the basic reason Venus is hot is in fact due to the presence of greenhouse gasses combined with the high pressure. It is not how much of the gas is greenhouse gasses, but the fact that there is at least some present . . .

The basic reason Venus is so darn hot is that it is so darn close to the sun. The high albedo moderates the temperature. The difference in composition affects the adiabatic lapse rate, but swapping Venus’ atmosphere with the same weight of air would give you no clouds and a much hotter planet.
The difference in lapse rates boils down to the gas molecules. Air is mainly diatomic, while Venus is triatomic CO2. The specific heat ratio Cp/Cv for air is ~1.4, CO2 is ~1.3, and that affects the temperature change as you move a parcel up and down, how much energy goes to ‘work’ as opposed to what’s left for temperature. Triatomic water vapor, our main ghg, has a Cp/Cv of ~1.3 like CO2. Methane, the next ghg suspect, also has a Cp/Cv of ~1.3. Sounds like a conspiracy.
Bottom line is that things on Venus are quite in accord with what you’d expect, no “runaway greenhouse” effect. I must acknowledge, though, that with no weather, Venus has solved the problem of climate change.

May 8, 2010 4:09 pm

Mike McMillan
There isn’t a large difference in lapse rates, and the albedo of Venus is very high, so it’s distance from the Sun has little effect on temperature.

Dr A Burns
May 8, 2010 4:09 pm

Troels,
The atmosphere is not static and conductive heat transfer to surroundings is small. Here’s a description:
“An adiabatic temperature change occurs in a vertically displaced parcel of air due to the change in pressure and volume (refer to the gas equation in section 1.2) occurring during a short time period, with little or no heat exchange with the environment. Upward displacement and consequent expansion causes cooling; downward displacement and subsequent compression causes warming. In the troposphere, the change in temperature associated with the vertical displacement of a parcel of dry (i.e. not saturated) air is very close to 3 °C per 1000 feet, or 9.8 °C / km, of vertical motion; this is known as the dry adiabatic lapse rate [DALR]. As ascending moist air expands and cools in the adiabatic process, the excess water vapour condenses after reaching dewpoint and the latent heat of condensation is released into the parcel of air as sensible heat, thus slowing the pressure-induced cooling process.”

George Turner
May 8, 2010 4:10 pm

Ian l. McQueen
Yep. I think the pressure going to zero argument is an oopsie. The pressure is determined by mass and gravity, and so is fixed (except for some small dynamic effects due to weather). It’s the volume and temperature of the atmosphere that vary.

May 8, 2010 4:11 pm

Another great post. I have been trying to go through similar ideas in the Climate Sceptics group for a while myself.
May I suggest a couple of interesting links:
(1) UTexas teaching notes about why there is an adiabatic lapse rate at all (hint: there is no mention of any GH effect)
(2) From the New Mexico State University a table about lapse rates in various atmospheres in the Solar System (and showing that composition might matter, alongside gravity)
In particular, the lapse rate is roughly g/Cp, where g is the acceleration due to gravity and Cp is the atmosphere’s specific heat at constant pressure (cp) divided by the molecular weight. It would be interesting to find out why exactly Cp for Venus would only be 85% of Earth’s.

May 8, 2010 4:11 pm

Forgot to close the hyperlink in previous comment. Apologies.

Jacob
May 8, 2010 4:14 pm

Why is the atmosphere on Venus 92 times more dense than on Earth? That is the big question, that I don’t understand.
Now, if the atmosphere is denser – it means there are 92 times more molecules of gas in the same volume (or layer) of the atmosphere, therefore they would absorb 92 time more outgoing radiation (much more anyway). So, the heat on Venus’ surface is caused by the greenhouse effect after all, by the fact that the greenhouse effect is much stronger there, due to the density of the atmosphere, and not necessarily due to it’s composition (Co2).
If pressure alone could cause heating – why isn’t the bottom of the oceans very hot?

Savant
May 8, 2010 4:21 pm

Steve,
You have almost got it now — here is the relevant question:
For Earth and Venus (as you note) the adiabatic lapse rate is similar; For Earth and Venus, the planetary equilibrium temperature is similar (Venus is closer to the sun but has a higher albedo). So now the question: To what altitude does the atmosphere mix vertically from the surface close to the adiabatic lapse rate (note on Earth that the true lapse rate is ~6.5o/km, e.g. the atmosphere is slightly stable)? You assume in your posts that it must be to the same pressure — but where does this assumption come from? On Earth, the temperate at 50 km is ~250K, if your 10K/km held to this altitude, the surface of Earth would be 750K — clearly something amiss. The answer is that at the tropopause OLR balances the incoming solar: outgoing energy (emission) from the atmosphere by cooler greenhouse gases and from the warmer surface (in the IR windows) balances the solar headed down.
So the important question for Venus is why is the tropopause at 60Km? You make the assumption that it is because this is (approximately) the same pressure as the tropopause on Earth. But there is no physical reason why the tropopause must be at the same pressure on other bodies. As on Earth, the tropopause is where the atmosphere must mix vertically to achieve thermal balance (e.g. read very nice discussion of radiative-convective models by Manabe). If instead of being made of IR absorbing constituents, the atmosphere was made only of N2 (even if it was as massive), the surface temperature would be much colder (and there would be a large day-night difference).
There is not much to this (at zeroth order) beyond the first law of thermodynamics in a compressible (nearly-ideal) gas.

Andrew W
May 8, 2010 4:26 pm

“In my last piece about Venus I laid out arguments against the claim that it is a runaway greenhouse which makes Venus hot. This generated a lot of discussion.”
The accepted view is that present day Venus is a result of a runaway greenhouse, she got too warm, her oceans boiled, the resultant water vapour created a super GH effect, which melted the planets crust, the oxygen released from ionize H2O combined with crustal carbon to create CO2. The planets surface is hot because of its present atmosphere, its atmosphere is a product of a runaway greenhouse.
“we have a sun providing energy to the periphery of the atmospheric system”
Not sure what you mean by this, the sun’s EM radiation is absorbed by atmospheric constituents throughout the depth of the Venusian atmosphere, not at the “periphery”.
“If there were no Sun (or other external energy source) atmospheric temperature would approach absolute zero. As a result there would be almost no atmospheric pressure on any planet”
The surface pressure is almost entirely independent of the temperature, assuming that all the atmospheric constituents remain suspended.

Dave
May 8, 2010 4:28 pm

Again, the height of the tropopause will change with the composition of the atmosphere: therefore, even if the lapse rate stays constant and the temperature at the tropopause stays constant, if reducing the amount of CO2 in the atmosphere causes the tropopause to decrease in height the surface will decrease in temperature. (And note that the troposphere in Venus contains a much larger percentage of the atmosphere than the troposphere on Earth, which would be consistent with a higher CO2 level pushing the tropopause upwards).
(also, while I agree that a runaway greenhouse on Earth is almost certainly not going to happen, I think that your back-of-the-envelope calculation of the effect of 100% CO2 atmosphere is wrong because the logarithmic forcing relationship isn’t going to hold above a couple thousand ppm CO2, and you are underestimating the potential for CO2 to lead to temperature increases at those extreme concentrations)

Stephan
May 8, 2010 4:32 pm

OT: One gets the distinct impression from recent posts here, at CA, and RC etc, that interest in the subject (AGW), is finally waning. Would be interesting to follow the sites statistics recently, to confirm/deny this. Basically the skeptics/deniers were right.. weather, aka climate is not changing due to AG. AGW news stories have certainly dipped. We can all go back and get a life instead of looking at graphs of temps, ice, polar bears, anxiety due to AGW, etc.. Of course a few diehards will keep banging away at it until they too, just give up, a good ol hahaha and LOL!

1DandyTroll
May 8, 2010 4:34 pm

Sometimes I figure GISS should’ve been named after you, sometimes not so much.
What defines atmospheric temperatures and what drives those temperatures? Is it just the proximity of a star? How about gravitational pul from say a black hole? Or just from being close to a gas giant like saturn and its gravitational pul?
But sure, get rid of all the stars and everything would’ve a temperature of absolute zero…. that doesn’t even compute.

Bob_FJ
May 8, 2010 4:38 pm

For anyone still believing that CO2 GHG is the fundamental reason for high surface temperatures, please consider this, (adapted from my comment in the earlier thread):
Here is an authoritative extract from the ESA, (my bold), so let’s examine it, particularly as to why Venus has a uniform average temperature everywhere. (over 117 earth days, and see link below)
“…[1] On Venus there are no day and night variations of the surface temperature. The heat is globally ‘trapped’ under the carbon-dioxide atmosphere, with pressure 90 times higher than on Earth.
[2] Instead, the main temperature variation is due to topography. Just like on Earth, mountain tops are colder, whereas the lowlands are warmer. The ‘only’ difference is that on Venus ‘cold’ means 447º Celsius, while ‘warm’ means 477º Celsius. Such high temperatures are caused by the strongest greenhouse effect found in the Solar System…”

[1a] When facing the sun, she is said to receive at the surface about 10% of sunlight. Whether this is a midday or total facing area average, or the effects of scattering, I don’t know, but whatever, the surface receives SOME solar energy. This energy amount must be lost back to space because the planet is apparently in thermal equilibrium. The fundamental process for this should be convection and conduction. Additionally, since infrared photography etc of the surface has been accomplished there is at least one window for some infrared to directly escape to space.
[1b] At nightime she no longer receives any solar energy, but has capability to lose heat in the same way as on the daylight side, over a period 117 times longer than on Earth. What is more, because the upper atmosphere is no longer heated by solar infrared, (~40% of sunlight), the temperature gradient of the atmosphere should increase, inferring increased conductive/convective cooling. Yet, there is no change in surface temperature! Such a condition would require an impossible perfect insulation layer, and no geothermal energy, but clearly, this is not the case.
[2] This part of the extract supports Steven’s hypothesis, but see also my comment in the earlier thread concerning the strange and unexplained dynamics of the atmosphere, according to the ESA. (that may have an astonishing “mixing” effect).

May 8, 2010 4:40 pm

Dave
You are ignoring the fact that Venus has almost 100% CO2 and it’s lapse rate is the same as Earth’s up to 60 km. The empirical evidence does not agree with your theory.

Jeff Green
May 8, 2010 4:40 pm

(kcom says:
May 8, 2010 at 2:46 pm
The mind is a powerful thing and can overwhelm logic when placed under enough stress. Ask any medical student if this has ever happened to them.)
James Hansen came to his conclusions based on observations. If you can prove him wrong you win. Do ou have a better explanation?

May 8, 2010 4:44 pm

Dave McK
Dry lapse rate means unsaturated (less than 100% humidity.)
Most of the troposphere is unsaturated.
http://www.physics.umt.edu/borealis/RH%2520Lab%2520Report_06.pdf

May 8, 2010 4:48 pm

George Turner
Gas pressure P = nRT/V
There is no term for either mass or gravity.

el gordo
May 8, 2010 4:54 pm

Stephan
Over at Deltoid its all political science these days, rarely do they discuss ‘graphs of temps, ice, polar bears, anxiety due to AGW, etc.’

wayne
May 8, 2010 4:55 pm

Steve, I don’t see how they get the lapse rate for Venus you mention. I tried three ways and each is much lower. Assuming 740K for the surface temp per Wiki, the top graph has two possibilities:
(740K-260K)/58km = 8.3 K/km
(740K-310K)/52.5km = 8.2 K/km
The lower graph, assuming Earth’s mean surface temp at 290K, based on pressure, not temperature is:
(740K-290K)/49.5km = 9.1 K/km
All of these are between the surface and to the point where ~90% of the mass of the atmosphere is lower.
Do you think their 10.468 K/km rate is more accurate? As you were saying, if co2 has it’s hand in it then the lapse should be much higher than the lapse rate of earth and I come up lower, not higher by the equally valid graphs (they tend to mirror what you read in Wiki).

May 8, 2010 4:55 pm

Andrew W
If that is the “accepted view” then I have even less respect for the perpetrators of this nonsense. Basalt melts at 1300 C.

MaxL
May 8, 2010 4:55 pm

As Kwik mentions I really have not given much thought to the atmosphere of Venus either. However, this discussion has been interesting and gets one to thinking. A couple of points need more fleshing out. The statement “If there were no Sun (or other external energy source) atmospheric temperature would approach absolute zero. As a result there would be almost no atmospheric pressure on any planet -> PV = nRT.” The ideal gas law has 3 variables, T, P and V. If you decrease T, as suggested, you can do this by keeping P constant, in which case V (volume) will decrease, you can keep V constant and decrease P or a combination of both. So it is not so simple as saying there would be no pressure if there was zero temperature. The gas molecules would still be there and have the same weight. This introduces another problem, namely the hydrostatic equation which states that the pressure of a column of air is solely a result of the weight of the air above. This applies most of the time to our atmosphere on Earth and is one of the fundamental equations in numerical weather models. Offhand, however, I am not sure how well it applies to Venus.
Another problem is the discussion of lapse rates and the adiabatic process. This requires the combination of both the “First principle of Thermodynamics” which relates heat to temperature, pressure and volume, plus the idea gas equation (PV = nRT). For an adiabatic process (no heat added or subtracted), when you combine these you get Poisson’s equations. The one most familiar is the equation relating temperature and pressure: T1/T2 = (P1/P2)^k
where k is found to be 0.286 for our atmosphere.
This is the equation for the dry adiabatic lines found on Skew-T or tephigram
diagrams. You can use this to find out what the temperature here on Earth would be at be at 9000 hPa given the temperature at 1000 hPa is about 293 K.
It is difficult on a blog to go into much more detail but I hope this helps out a bit.

kwik
May 8, 2010 4:56 pm

Stephan says:
May 8, 2010 at 4:32 pm
Yes, Stephan, but:
They are now promoting new scary stories.Acidification of the osceans seems to be the new big thing. In Norway there has been several stories in the news lately.And lo and behold, after a few months camaign a new research institute is now planned to study this.

Robin Kool
May 8, 2010 4:59 pm

In the end there is simply the warmth given by the sun plus the warmth produced inside Venus, and the warmth radiated into space.
My opinion:
It is the heat the atmosphere gets from the planet surface that determines how far the atmosphere expands into space, not the other way around.
It is not the distance the atmosphere expands into space that determines the temperature at the surface.
The air receives heat on the planet surface, expands, rises and cools down, and falls back. As it falls back and contracts, it warms up again, but it will not gain more heat than it has lost by expanding. As it has also lost heat that is radiated out to space, it will arrive at the surface of the planet cooler than it left it.
My reasoning:
1. OK. For the moment, let’s forget the green house effect and let’s start with a very cold atmosphere, contracted very close to the surface of Venus.
2. You say: “Because we have a sun providing energy to the periphery of the atmospheric system, the atmosphere circulates vertically and horizontally to maintain equilibrium.”
The energy of the sun must go through the atmosphere and warm the surface of the planet, which then warms the lower layers of the atmosphere for circulation to start happening.
(The sun providing energy to the periphery (= outside) of the atmosphere, means the periphery warms up. The air there expands and wants to rise, not sink, thus no circulation. The word periphery is usually used for ‘outside’ and seems misplaced here.)
3. OK, when such a dense atmosphere is heated up, the resulting upward force is greater that of the thinner Earth atmosphere heated up to the same temperature. This makes the Venus atmosphere spread out further into space than Earth’s.
4. And yes, when the expanding gas has reached the outer layer, and has cooled down, it will fall and contract and heat up, but it will not gain more heat than it has lost by expanding. As it has also lost heat that is radiated out to space, it will arrive at the surface of the planet cooler than it left it. (Otherwise it would be a perpetuum mobile).
5. As Venus has no oceans and no biosphere, it has a hell of an urban heat island (UHI) effect.
The energy that the sun radiates on the surface heats up a thin layer of surface to very high temperatures. The whole planet is one big heat island.
None of it is spread over a huge body of water like our oceans or converted into plant growth.
I would expect that to be the most important reason the surface temperature of Venus is so much higher than that of the Earth.
Plus, of course, that Venus is closer to the Sun and receives more heat per square meter or foot than Earth.
6. The much denser atmosphere is much better at absorbing the heat of the planet’s surface, than Earth’s thinner atmosphere. Giving it a huge upward, expansive force, that expands it far out into space.
But the extend to which it spreads out is determined by the heat it receives on the surface, so that after the hot air has reached the cold space and – cooled down – falls back, it does warm up, but not to higher temperatures than it had when it started at the surface of the planet.
In other words, it is the heat content of the atmosphere – heat it gets from the planet surface – that determines how far the atmosphere expands into space, not the other way around.
It is not the extend to which the atmosphere expands into space, that determines the temperature at the surface.
7. The heat content of the atmosphere if Venus is so high, because of the massive UHI effect and because the dense atmosphere is so efficient at absorbing heat from the surface.
(8. I imagine that without oceans there is no thunderstorm belt that transports massive amounts of heat to the highest layers of the atmosphere, bypassing the greenhouse effect – the thermostat Earth has in the tropics.)

u.k.(us)
May 8, 2010 5:03 pm

I’m in way over my head here, but doesn’t the “solar wind” change the depth of our atmosphere. Which would cause pressure and temperature changes?

George Turner
May 8, 2010 5:05 pm

I’d like to suggest another experiment for Steve.
To simulate the effects of an IR blocking atmosphere on a very dim planet, build a greenhouse. The glass for the greenhouse should be selected for absolute IR opacity across the band, and it can even be triple-paned and argon-filled to limit convection losses. To best mimic the dim light on the surface of Venus, build the greenhouse deep in the forest and wait for a very dark, overcast day. Sit inside with a good thermometer and see if the temperature approaches 900 degrees F.

May 8, 2010 5:09 pm
Jeff Green
May 8, 2010 5:10 pm

(http://en.wikipedia.org/wiki/Atmosphere_of_Venus#Structure_and_composition
The large amount of CO2 in the atmosphere together with water vapor and sulfur dioxide create a strong greenhouse effect, trapping solar energy and raising the surface temperature to around 740 K (467°C), hotter than any other planet in the solar system, even that of Mercury despite being located further out from the Sun and receiving only 25% of the solar energy Mercury does.[11])
Based on AGW theory if venus’s atmosphere were nitrogen, then infrared would continue back out into the space without being reactive to the nitrogen. The gas in the atmosphere does make a difference as to how much heat is held in.

May 8, 2010 5:12 pm

MaxL
Gas pressure is created by the motion of molecules. As you approach absolute zero, molecules cease to move. So there is no pressure.
Volume has to remain above zero, because the molecules have a finite size. So yes, it is that simple.

suricat
May 8, 2010 5:14 pm

Ian l. McQueen says: May 8, 2010 at 2:40 pm
“I am always puzzled by the statement that the atmospheric pressure on the surface of Venus is a large multiple of that on earth.”
If you blow away many of the lighter elements of a planet’s atmosphere with the ‘solar wind’ and continue to warm it, the heavier elements will stay there in the atmosphere to provide a greater surface pressure.
Having said that, I’m sure you realise that the heavier gasses should form a shorter altitude distance for any change in pressure. It’s curious that the lapse rate of both Venus and Earth are similar, but is this more to do with the similar orbital distance from Sol than the atmospheric makeup of these planets?
Well I think this is possible, but perhaps it’s more to do with the natural progression of (and here’s a new term) ‘nature’!
We have the ‘nature’ of Earth, and we have the ‘nature’ of Venus.
The history of Venus has led it to its current situation, and ditto for Earth. Although Venus has a lower gravity field than Earth it has lost many of its lighter elements. Thus, Venus’s troposphere contains compounds that are much heavier than those incorporated within Earth’s troposphere and this reflects in its higher surface pressure.
Best regards, Ray Dart.

wayne
May 8, 2010 5:14 pm

I ought to correct the percentage I noticed in my comment above, at ~50-55km altitude about 99% of the mass is below this point, not 90%. Technical, but to be as accurate as possible…

William Sears
May 8, 2010 5:21 pm

The adiabatic lapse rate argument works best with heating from below. This might be caused by geothermal activity since the atmosphere of Venus, unlike earth, is opaque. I believe that a number of scientists have studied the strange geology of Venus’ “not quite plate tectonics” and no doubt have already made this same suggestion. Popular knowledge dies hard though, even among scientists, and the runaway greenhouse analogy lives on.

GeoFlynx
May 8, 2010 5:28 pm

[snip – try rewording that, there’s no reason to hurl insults]

May 8, 2010 5:34 pm

William Sears
Whether the source of heat is geothermal or absorbed SW, the lapse rate is the similar on both planets – indicating that greenhouse effect is less important on Venus than advertised.

Jeff Green
May 8, 2010 5:37 pm

(http://en.wikipedia.org/wiki/Atmosphere_of_Venus#Structure_and_composition
The atmospheric pressure at the surface of Venus is about 92 times that of the Earth, similar to the pressure found 910 metres below the surface of the ocean. The atmosphere has a mass of 4.8 × 1020 kg, about 93 times the mass of the Earth’s total atmosphere.[1] The pressure found on Venus’s surface is high enough that the carbon dioxide is technically no longer a gas, but a supercritical fluid. The density of the air at the surface is 67 kg/m3, which is 6.5% that of liquid water on Earth.[1])
The carbon dioxide is so thick that it has more the properties of fluid rather than a gas. There are all kinds of studies and observations showing co2 blocks infrared energy.

Ian L. McQueen
May 8, 2010 5:38 pm

Stephan says:
May 8, 2010 at 4:32 pm
OT: One gets the distinct impression from recent posts here, at CA, and RC etc, that interest in the subject (AGW), is finally waning. Would be interesting to follow the sites statistics recently, to confirm/deny this. Basically the skeptics/deniers were right.. weather, aka climate is not changing due to AG. AGW news stories have certainly dipped. We can all go back and get a life instead of looking at graphs of temps, ice, polar bears, anxiety due to AGW, etc.. Of course a few diehards will keep banging away at it until they too, just give up, a good ol hahaha and LOL!
Staphan, in an ideal world you would be right. But, unfortunately, many governments have accepted the IPCC reports as gospel truth and have enacted legislation designed to reduce CO2 emissions. They will do nothing for the climate but WILL do great harm to the economy. It is as if we were on a hill overlooking a railway line on which a train is headed at high speed for a bridge that is no longer there. We know that a trainwreck is inevitable, but there is nothing that we can do to stop it.
IanM

May 8, 2010 5:45 pm

Jeff Green
There is no question that CO2 absorbs infrared.

Ian L. McQueen
May 8, 2010 5:47 pm

stevengoddard says:
May 8, 2010 at 4:48 pm
George Turner
Gas pressure P = nRT/V
There is no term for either mass or gravity.
Steven-
The term “R”, the gas constant, has units of 8.314472(15) J·K-1·mol-1

Ian L. McQueen
May 8, 2010 5:48 pm

stevengoddard says:
May 8, 2010 at 4:48 pm
George Turner
Gas pressure P = nRT/V
There is no term for either mass or gravity.
Steven-
The term “R”, the gas constant, has units of 8.314472(15) J·K-1·mol-1
To me, “mol” implies “mass”, and in a gravitational field it will have weight. Doesn’t this contradict your statement?
IanM

George Turner
May 8, 2010 5:52 pm

Steven,
Gravity doesn’t appear in PV=nRT, but it does appear in F=ma. The mass is the mass of the atmosphere and the acceleration is due to gravity. That gives the downward force an atmosphere exerts on the surface (minus a small error because gravitational acceleration decreases with altitude).
Given a planet’s surface area S, the average surface pressure must be F/S = mg/S.
Venus atmophere’s mass is approximately 4.8×10^20 kg and its surface gravity is 8.87 m/sec^2, so the force it exerts on the surface is 4.26×10^21 Newtons. This is spread across a surface area of 4.6×10^8 square km, or 4.6×10^14 m^2, so the pressure is 4.26×10^21 N/4.6×10^14 m^2 = 9.26×10^6 N/m^2 (Pascals), which is 1343 psi or 91.4 atmospheres.
Even if the atmosphere condenses to a liquid or freezes to a solid, the pressure on top of the rocks is still going to be 91.4 atmospheres.
🙂

May 8, 2010 5:53 pm

Ian L. McQueen
Joules is a unit of energy
Degrees Kelvin is a unit of temperature
moles is a non-dimensional number – 6.022 x 10^23 molecules
None of these are units of mass or gravity.

Jbar
May 8, 2010 6:00 pm

Steve,
I agree with your claim about the lapse rate explaining in part why the surface of Venus is so hot, but the lapse rate does not disprove the existence of a greenhouse effect, and I believe it is only part of the story about the high surface temperature.
First, a technicality – your dry lapse rate only applies for an adiabatic process where there is no condensation or heat transfer. It is an upper limit. The actual lapse rates on both Earth and Venus are lower than their dry lapse rates.
In your previous post “Hyperventilating on Venus”, you presented this plot (http://www.globalwarmingart.com/wiki/File:Atmospheric_Transmission_png) showing the transmissivity of various gases in Earth’s atmosphere. It shows that the 380-some ppm of CO2 in Earth’s atmosphere is enough to block all infrared radiation from 13 microns to 17 microns wavelength. Combined with the infrared opacity of Earth’s water vapor, that still leaves a broad window of atmospheric transparency to infrared between 8 microns and 13 microns, enough to keep the Earth at its current comfortable temperature range.
However, on Venus, the CO2 concentration is the equivalent of 89 million ppm, relative to Earth’s atmosphere, or 234,000 times the optical thickness of CO2 on Earth. As a result, the atmospheric transmissivity to infrared on Venus is quite different from the plot linked above.
I got into this Venus argument with a friend last year and to satisfy myself of the effect of CO2, I subscribed to Spectralcalc.com for a month to run some numbers. I was able to replicate the CO2 curve in the Atmospheric Transmission plot for Earth, and I found that for Venus, the absorptive range of the gas broadens so much to other wavelengths that the Venutian atmosphere is opaque to infrared from below 2 microns up to 28 microns except for three relatively narrow windows of transparency at 2.3, 3.4, and 5.7 microns, even for only 5 kilometers of atmospheric depth (near the surface). Also, my sources indicate there is enough water vapor in the Venutian atmosphere (somewhere above 50 km altitude) to effectively block infrared emissions above 28 microns.
This shows the Venutian atmosphere is way more opaque to infrared than Earth’s atmosphere and does indeed reveal a strong greenhouse effect on Venus. Even the lower atmosphere, while efficiently passing infrared from 28 microns and higher (because there is virtually no water vapor below the clouds) is still almost opaque from 2 to 28 microns and presents a substantial impediment to radiative cooling of the surface.
As I said in the beginning, I agree with your claim that the lapse rate helps to explain partly why the Venutian surface is so very hot, but I think that lapse rate is not the whole story. With the surface receiving only 1/6th the sunlight that the Earth’s surface receives, something has to act as a barrier to slow the rate of radiation from the surface to the much cooler haze layer at 30-50km altitude, and I think that something is the CO2 infrared opacity. I’m no atmospheric scientist or physicist and I lack the tools to model this, but I suspect that if CO2 were transparent to infrared the Venutian surface would be a lot cooler than it is, but still substantially hotter than the upper atmosphere thanks to the lapse rate.
Heat from below ground cannot possibly explain the high surface temperature as some have suggested. On Earth, the heat radiating from the ground is only 1/10th of a watt per square meter, and even if it is ten or 100 times that on Venus, this is still dramatically less than the visible light reaching the surface.
[Interesting factoid: On Earth, refraction or bending of sunlight makes the horizon light during dusk and dawn even though the sun is not up. Well according to one source, on Venus the refractive effect from the thick atmosphere is so much stronger than on Earth that even the darkest region of the dark side still gets some glow from the sunlit side.]

Mike M
May 8, 2010 6:01 pm

I disagree on the PV=nRT analysis Steve. Assuming the gas remains a gas, the pressure remains constant at the surface because we must obey conservation of mass. Given a control column of a given mass of atmosphere, open at the top to vacuum space, the pressure at the surface will not change** with temperature, (** ignoring, as another poster above rightly pointed out, differences in gravity WRT altitude), because it’s weight remains the same. So it’s ‘V’ that changes not ‘P’. As temperature decreases, the height of the column will decrease thus decreasing the volume.

Mike M
May 8, 2010 6:08 pm

Speaking of radiation… How much of Venus’s heat is coming from it’s own core? Given its formation closer to the Sun, I would surmise that Venus might have a greater proportion of heavier/more unstable elements at its core than Earth. ?

George Turner
May 8, 2010 6:11 pm

I need to amend my previous comment.
If the CO2 on Venus froze it would form dry ice about 2000 feet deep, on average, and would of course form glaciers, thus increasing the pressure in the lowlands and leaving the mountain tops in a vacuum.
As a side note, the black body spectrum at 735 Kelvin starts emitting at around 700 nm, in the visible spectrum, peaks at about 3.9 microns, then tapers off towards 15 microns.
blackbody calculator
Since very little energy makes it down to the surface, successfully maintaining Venus’ temperature purely due to IR blocking would require trapping 99% of the energy emitted between 1 to 15 microns. I don’t think CO2 alone is going to accomplish it.

DesertYote
May 8, 2010 6:11 pm

The whole idea of a runaway greenhouse effect causing the dense atmosphere of Venus is so silly that it could only have been as created in the minds of Marxist brainwashed pseudo-scientists with a political agenda. The Universities have been pushing the nonsense for so long that even some real scientists have been hood winked by it. It is not that Venus has an unusually high atmospheric density for its size and location, its that the Earth has an unusually low density. That density can be quite adequately be attributed by that absurdly big ball we call the Moon.
Why are so many so confused by the relationship of temperature and density. Gas is compressible so its density is highly related to pressure. Energy will distribute evenly through the mass. The more mass per volume, the more kinetic energy per volume, the higher the temperature. It is not pressure causing heating. Sheesh! Liquid is not a compressible fluid so its density is not very related to pressure. The behaviour of the Earth’s oceans tells us nothing about the behaviour of an atmosphere. So bringing up the thermal characteristics of the Ocean is meaningless.

kuhnkat
May 8, 2010 6:18 pm

Jeff Green, please educate yoursel on how little of the infrared band is active with CO2 even at the higher pressure and temp of the lower Venusian atmosphere.
IF all of the IR was concentrated in the tiny active bands of CO2 (which it isn’t, see black body and grey body emisssions), then CO2 could have a higher influence on temperature in the system. As it is the MASS of CO2 has a much larger effect than its GHG properties.

CRS, Dr.P.H.
May 8, 2010 6:24 pm

This is, without a doubt, the most authoritative description of the planet Venus and its environment: http://www.feedbooks.com/book/2482.pdf

SeanH
May 8, 2010 6:32 pm

With regards to the atmospheric pressure comment, it appears as though he is talking about the pressure by the gaseous envelope surrounding the planet, the atmosphere. If the temperature were dropped to 0K, the gases would solidify, leaving the planet without an atmosphere. Hence, no atmospheric pressure.

May 8, 2010 6:32 pm

There’s a basic fallacy here which Leonard Weinstein has covered fairly well. Yes, the adiabatic lapse rate is as described. With a gas in motion and under gravity, there will be a temperature difference across any layer approx as determined by the ~10 K/km figure.
But that’s a difference. It doesn’t tell you how hot it will be at top or bottom. The analogy is a battery. There’s a 1.5V difference end to end, but the actual voltage depends on how it’s connected. Where it is earthed.
A 9.2 MPa atmosphere of nitrogen will have a big temperature difference, top to bottom, but it is not “earthed” at the top. No heat is exchanged there. The temperature at the bottom is entirely determined by the radiative balance there. That’s why the bottom, in Venus conditions, couldn’t be at 700K. The 12000 W/m2 IR emitted at 700K would escape unhindered. The surface would have to cool until balance with avaible sunlight was restored. The whole column of N2 would cool with it, maintaining the lapse rate. The top would cool at the same rate as the bottom.
This is what the greenhouse effect does. It “earths” the system at TOA. CO2 emits to space at the top, not at the bottom. The radiative balance sets the temperature there. About 164 W/m2 (less than Earth) has to be emitted there to balance net sunlight absorbed (after albedo) and than determined the temperature there. The lapse rate then determines the temperature at the surface, and makes it hot. Just as on Earth, except that with our smaller GHG component, Earth also has significant emission from the surface at atmospheric window frequencies.

Stephan
May 8, 2010 6:40 pm

Well to those who replied above all I can say is Australia is out.. looks like the USA is gonna be out too.. I don’t think the French or Germans really care anymore. The British government will be much less interested that the Laborites.. and so on.. In fact it looks like Bolivia and the UN is now taking over the AGW cause. In fact I think the next Winter in both SH and NH (because that is the only thing the ordinary fellow/fellaw in the street really understands), is going to be so so cold…Good luck to ol Evo Morales Hahaha. BTW I think it is the actual weather that determines whether the ordinary person on the street will decide whether AGW is happening or not and most of the shift is due to that (ie very cold winters is Europe and USA this year). They are having doubts….

kuhnkat
May 8, 2010 6:43 pm

Nick Stokes,
“CO2 emits to space at the top, not at the bottom.”
Which side of the molecule is the TOP??
HAHAHAHAHAHAHAHAHAHAHAHAHAHAHAHA
In case any one listens to you, CO2 emits to space from tropopause out to the edge of the atmosphere here on earth and a similar depth on Venus. You are being simpleton again and more confusing than informative.

Steve Goddard
May 8, 2010 6:48 pm

I’m not sure where anyone got the idea that I am suggesting that there isn’t a greenhouse effect on Venus. It certainly wasn’t from anything I have written. This is a discussion of relative magnitude.

Richard Sharpe
May 8, 2010 6:55 pm

DesertYote at May 8, 2010 at 6:11 pm said:

Why are so many so confused by the relationship of temperature and density. Gas is compressible so its density is highly related to pressure. Energy will distribute evenly through the mass. The more mass per volume, the more kinetic energy per volume, the higher the temperature. It is not pressure causing heating. Sheesh!

Wikipedia defines temperature as the average kinetic energy of particles in matter. You seem to be talking about energy density. While Wikipedia often contains things I disagree with, there seems to be a disconnect here.

Steve Goddard
May 8, 2010 7:01 pm

George Turner
Your explanation is correct in that gravity is the cause of the pressure, which drives the temperature.

George Turner
May 8, 2010 7:03 pm

Well Steve, I just thought it was assumed. Since Venus is totally devoid of vegetation, the idea of a greenhouse on it is rather silly. ^_^
Anyway, given the little F=ma thing, imagine if we reduced Venus’ surface gravity by a factor of 90, to about 0.01G or so (Vulcan terraformers probably do this all the time). Then we have a planet with the same amount of CO2, the same clouds, the same basic atmospheric structure (just stretched way up), but a surface pressure that’s Earth normal. If GHG completely drives Venus’ temperature then the dramatic change in surface pressure wouldn’t matter in the long term, and it would still be 700K+. If adiabatic heating is largely responsible then the change should produce a much cooler surface.

DesertYote
May 8, 2010 7:15 pm

Nick Stokes
May 8, 2010 at 6:32 pm
“The top would cool at the same rate as the bottom.”
No it would not unless the density was constant which it would not be.

May 8, 2010 7:26 pm

Steve Goddard
“I’m not sure where anyone got the idea that I am suggesting that there isn’t a greenhouse effect on Venus. It certainly wasn’t from anything I have written. This is a discussion of relative magnitude.”

Well, Steve, what then is your point? Relative magnitude of what? The fact that the lapse rate creates a temperature difference between where GHG’s emit (high) and the surface has always been an essential part of the greenhouse effect.

Steve Goddard
May 8, 2010 7:27 pm

Mole is a non-dimensional number. A mole of H2 weighs much less than a mole of UF6

Keith Minto
May 8, 2010 7:30 pm

The heat MUST be created by planetary forces, see http://www.esa.int/esaSC/SEMUKVZNK7G_index_0.html .
It has long been recognised that there are simply not enough craters on Venus. Something is wiping the planet’s surface clean. That something is thought to be volcanic activity……..
and “Now we have strong evidence right at the surface for recent eruptions,” says Sue Smrekar, a scientist at NASA’s Jet Propulsion Laboratory in California. They do say They estimate that the flows are possibly as geologically recent as 2 500 000 years – and likely much less, possibly even currently active. In other words they don’t know, but something must be bringing heat from the interior of the planet to lava flow the surface. This must be the major phase change going on in ‘Her’. Recent laval surface remodelling seems to me to be the cause of the night/day hot surface temperatures, the sun’s role on the surface heat seems minor. The (relatively) waterless atmosphere without significant convective
forces seems to be keeping the heat in.
This enormous heat must be coming from the planet itself, the atmosphere is merely a blanket, the adiabatic lapse rate is too limp to explain such persistent high night /day heat.

DocMartyn
May 8, 2010 7:45 pm

Just why are there clouds? Sunlight hits the top of cloud, some scattered, some absorbed, clouds evaporate.
Why does Venus have clouds during the day time? For clouds to form a liquid must have been heated, evaporated, rose up through the atmosphere and then cooled as the pressure drops.
So, why are there clouds?

Leonard Weinstein
May 8, 2010 7:50 pm

I want to point out that the oceans of the Earth hold about 40 times as much Carbon compounds as the atmosphere, and that came from dissolved CO2. In addition, the carbonate compounds in the solid land surface and sea floor of the Earth contain about 40,000 times as much Carbon as the atmosphere, and that Carbon came from life on early Earth forming Carbonate shells and organic debris (and oil and coal) from early CO2, and that this debris was sequestered on the bottom of oceans and on land. The waste product of plants taking in CO2 is Oxygen. This implies that there was once mostly CO2 in early Earth atmosphere, and much was removed over time by living organisms. Additional atmospheric material was probably removed in the collision between Earth and another planet that resulted in the formation of the Moon. There is no big mystery why Earth has little CO2 and both Venus and Mars are mostly CO2. The differences in quantities depend mainly on the size of the planets and formation details.

George Turner
May 8, 2010 7:54 pm

An experiment:
We drop a spherical probe onto Venus, weighted on the bottom so it rolls right-side up. The probe is a hollow ball well insulated with asbestos (gotta use that stuff somewhere!) and with a large IR transparent window on top. The window is insulated against convection and conduction by having multiple panes, argon filled from a high-pressure canister and a pressure regulator.
The temperature on Venus’ surface is in balance, so the heat gained from downward radiation plus the heat gained from the atmosphere by convection equals the heat lost to upward radiation and the heat lost to convection.
The internal temperature of the well-insulated probe will stabilize to a value determined only by the radiant energy balance. If the probe is cooler than the outside atmosphere then the surface is losing more radiant energy than it’s gaining, which means Venus surface must gain heat from atmospheric convection. If the probe runs hotter than the outside atmosphere then the surface must be losing heat to the atmosphere by convection. The probe’s results should allow some numbers to be placed on the surface radiation balance.
Unfortunately, finding funds for a probe that might remove Venus’ status as a dire warning to Earthlings, and thus cause us to defund all future Venus missions, might be a bit difficult.

George Turner
May 8, 2010 7:59 pm


I’ll consult with the local florist, but I’m pretty sure that huge pressure differences don’t come included in the standard greenhouse package.
Hehehehe.

George Turner
May 8, 2010 8:00 pm

Darn it. That was in reply to Nick Stokes

DesertYote
May 8, 2010 8:00 pm

Richard Sharpe
May 8, 2010 at 6:55 pm
I was under the impression that it is average kinetic energy within a volume.

May 8, 2010 8:40 pm

DesertYote
Yes, you’re right – the dry lapse rate wouldn’t change, but would operate over a shorter distance as the gas cooled and contracted. But it doesn’t change my point that a N2 atmosphere would cool as the surface cooled in a way that just tracks that cooling. It doesn’t constrain the surface temperature.

J.Hansford
May 8, 2010 8:41 pm

Ian l. McQueen says:
May 8, 2010 at 2:40 pm
“……A further niggle comes from the statement: “If there were no Sun (or other external energy source) atmospheric temperature would approach absolute zero. As a result there would be almost no atmospheric pressure on any planet -> PV = nRT.”
At 0K all gases would have condensed to liquid or solid, but the mass of the gas would still be present as a layer on the surface. I would think that the liquid/solid would be as heavy (=attraction due to gravity) as the atmosphere from which it was produced, and possibly a tiny bit higher since the molecules would be concentrated at the surface instead of extending a distance above the planet and thus minutely farther from the center of attraction of the mass of the planet.”
———————————————————————————
At absolute zero…. No molecule is in a gasseous state…. It is solid and inert…. even with no pressure…. This is a theoretical limit of course…. for Absolute zero has never been observed….. So a planet with a 100 percent nitrogen atmosphere would have a surface of solid nitrogen at 0K. There would be no sublimation… Because no molecule has any energy left.
That’s my understanding anyway.

Dave McK
May 8, 2010 8:42 pm

stevengoddard says:
Steve- check out a basic HVAC manual- there is a reason nobody designs a heat pump with a gas that doesn’t change phase.
Next, calculate the enthalpy of the constituents of air = be generous and assume 1% h20 and 500ppm co2 – do the math and see who carries the joules. The water does more than 50,000 times the work of the co2. Nevertheless, co2 does a tiny bit of work- and that only increases the efficiency of the heat flow from boiler to radiator.
Examine the profiles of earth – overlay the charts with precipitation and IR radiation.
If you don’t know the difference between the lapse rate of dry air and the actual measured lapse rate on earth which is much different, then look it up. Dithering on distractions doesn’t change these 2 fundamental facts:
Earth has a heat pump with water as the working fluid.
Any increase in efficiency of the working fluid in a heat pump (like adding some heat capacity the way CO2 does) only slows the flow for the same work. Measure and calculate the work.
At least understand that thermodynamics is about heat and that is not measured in degrees. Heat is NOT measured in degrees and there is no way to convert degrees to watts. NOT possible.
You want to study heat, you have to actually speak about heat. This has been hugely ignored for a reason.
Meanwhile- every time you suggest that the CRU and the rest are unscientific- let me clear this up once and for all:
They are brilliant psychologists and have the best and brightest marketing psychologists money can buy. You only imagine they aren’t scientific because the sad sorry truth is that you have been, are being and will keep on being duped because they are smarter than you where it matters in this game. You just haven’t really got a grip on what the game is. You’ve been outsmarted and never really stood a chance. You still argue about weather.
Strut and fret and in the end – you get shafted because you weren’t watching the pea, you were watching TV.
Troposphere, shmoposphere – clouds, rain, snow.
Once it was ‘don’t talk about religion or politics’ and it was safe to talk about the weather because nobody got offended and everybody knew something to say.
Admit that it was a brilliant maneuver to suck common man into war over the most innocuous topic. Look at the ROI. Look how well it worked and continues to work.
No, sir, they are extremely scientific and the science is valid – the results prove it.
How long ago was venus a molten ball, man? And how many trillions will it cost to play make-believe?
You don’t need a weatherman to know which way the wind blows – Dylan.
Sorry for the long winded rant. It’s been building up. Nothing personal – I’m looking at where the train is about to go dig a dirt road and the passengers chat and gossip and that bothers me because I did my Dostoevsky period long ago and didn’t want to revisit it in america, in real time.
Hugo Chavez is a piker compared to the creeps stealing america. But hey- some evolution is about to happen and guess who will get the darwin awards- it won’t be Hansen, Schmidt and Jones. Who’s the unscientific one? Well, who has the whose money?
I think I can rest my case.

May 8, 2010 8:50 pm

Steve Goddard says:
May 8, 2010 at 7:27 pm
Mole is a non-dimensional number. A mole of H2 weighs much less than a mole of UF6

The mole is the SI unit of the amount of substance, it is not non-dimensional, a mole of a substance has a mass equal to the substance’s molecular weight.

GaryW
May 8, 2010 8:56 pm

Steve Goddard
I usually agree with your postings. This one I am having trouble with. Adiabatic Lapse Rate is about temperature changes as air masses move up and down in altitude. Environmental Lapse Rate is what I think you are trying to describe. That is the temperature lapse rate with altitude. That is what you get when the air is not moving. Yes, I realize the atmosphere of Venus is probably moving up and down. However, that movement balances out producing a net neutral effect with respect to pressure change.
Consider a tall column of perfectly insulated gas. Given sufficient time for temperatures to stabilize in the column, there will be no difference in temperature top to bottom. Though at a higher pressure, the bottom of the column will not be warmer than the top. Physics show us that the gas at the bottom is at a higher energy level because of the greater potential energy associated with the higher pressure. That does not mean, however, that the gas molecules at higher pressure exhibit greater kinetic energy than the those at lower pressure.
For planetary atmospheres, the Environmental Lapse Rate is a function of the energy lost versus gained from radiation and conduction at a given altitude. Pressure figures in only in the density of the molecules available to absorb and release energy and whatever phase or chemical changes may occur.
We should actually be calling what is happening on Venus a “blanket” effect, not a “greenhouse” effect.

May 8, 2010 9:02 pm

Dave McK
You can stop hyperventilating now. The average measured lapse rate on Earth is a little lower than the dry lapse rate. About 6.5. On Venus the average measured lapse rate is also a little lower than the dry lapse rate at 8.5. Not much difference.
Your argument doesn’t provide much useful information about why Earth is 20C and Venus is is 485C.

J.Hansford
May 8, 2010 9:06 pm

Jacob says:
May 8, 2010 at 4:14 pm
Why is the atmosphere on Venus 92 times more dense than on Earth? That is the big question, that I don’t understand.
Now, if the atmosphere is denser – it means there are 92 times more molecules of gas in the same volume (or layer) of the atmosphere, therefore they would absorb 92 time more outgoing radiation (much more anyway). So, the heat on Venus’ surface is caused by the greenhouse effect after all, by the fact that the greenhouse effect is much stronger there, due to the density of the atmosphere, and not necessarily due to it’s composition (Co2).
If pressure alone could cause heating – why isn’t the bottom of the oceans very hot?
————————————————————————————-
You are confusing gasses and liquids Jacob… Seems a few others are doing the same. Gasses are compressible, liquids are not….. Compress a gas enough, and it becomes a liquid…..
Compressing a solid or liquid results in a crystalline structure I believe, but only under immense pressure…..(?)

George Turner
May 8, 2010 9:23 pm

Gary W,
If the atmosphere on Venus stopped circulating, then I agree that the compression effects would dissipate. But the atmosphere there is constantly rotating in two giant Hadley cells. The air rises near the equator, moves to the poles, descends, and moves to the equator again, so it is always undergoing cycles of compression and expansion. This means it is also undergoing cycles of adiabatic heating and cooling.

Dave
May 8, 2010 9:29 pm

“You are ignoring the fact that Venus has almost 100% CO2 and it’s lapse rate is the same as Earth’s up to 60 km. The empirical evidence does not agree with your theory.”
I am not saying that the temperature contribution of GHGs comes from changing the lapse _rate_. I am pointing out that to use the lapse rate to determine the surface temperature, you have to know both the temperature at the tropopause and the altitude of the tropopause. GHGs (and the resulting optical thickness/depth/opacity) change the altitude at which the atmosphere emits to space. Once you know that altitude, and the temperature at that altitude, _then_ you can use the lapse rate to calculate the surface temperature.
A totally transparent atmosphere will result in a planet that emits to space from the surface, and therefore the temperature of the surface will be in radiative balance with the incoming radiation from the sun. As you add GHGs, the height at which the atmosphere emits to space will increase, and the temperature of the surface increases. There is not some magic concentration of GHGs at which Venus suddenly jumps from -46 degrees C (the temperature of the surface given the existing albedo and no greenhouse effect at all: yes, this requires assuming clouds that exist at that temperature and reflect incoming light but don’t trap outgoing light, but this is a thought experiment) to 450 degrees C.
Replacing 90% of your CO2 with Folgers Crystals (I mean, N2) would change the optical thickness, and therefore the height at which you emit to space, and therefore the surface temperature EVEN IF THE LAPSE RATE REMAINS CONSTANT.
“Mars is even colder than earth despite having a 95% CO2 atmosphere, because it’s atmosphere is very thin.”
No, Mars is even colder than earth despite having more CO2 than earth because it is a lot farther from the Sun, and the additional radiative forcing from the CO2 is not enough to make up for the lower solar radiation.
Look: there are now two models of the Earth + Venus that we’re discussing: the “Standard Model” which depends on GHGs, and the “Goddard Model” which apparently requires some non-zero amount of GHGs but mostly depends depends on pressure. Why do I prefer the Standard Model, if both models apparently get the surface temperature right? I prefer the Standard Model because using the same set of physical laws it can also explain the mesospheric, stratospheric, and thermospheric temperature profiles. It can explain why the tropopause changes altitude and temperature depending on latitude. Effectively, it is a sophisticated model that has been used by atmospheric and planetary scientists for decades and survived all their tests. Is it perfect? Well, it doesn’t, for example, tell us what Climate Sensitivity should be, but for problems like explaining Venusian, Terran, and Martian temperature profiles given existing GHG concentrations, clouds, solar radiation, etc. it does really well – whereas the Goddard Model can only explain temperatures from the surface to the tropopause, and only if you tell it where the tropopause is and what temperature the tropopause is.

Keith Minto
May 8, 2010 9:32 pm

George Turner says:
May 8, 2010 at 7:54 pm
George, I think you are on the right track, all you need is a downward sensor facing the crust in the shadow of the craft and an upward sensor sensing radiative energy from above and compare the two.
Willis, in the earlier thread, suggested that the surface pressure was such that a super critical fluid would occur, the images above would suggest otherwise.

Bamboozled
May 8, 2010 9:33 pm

What about the role of upper atmosphere sulfuric acid clouds? They are dense and heavy, right? Can they act like a glass of plastic film in a real greenhouse? http://astronomyonline.org/SolarSystem/VenusIntroduction.asp says “Extremely high atmospheric winds – 360 km/h – yet mild breeze near the surface”. This implies there is little heat exchange via convection near the surface. So, it’s like a car with rolled up windows in a parking lot under direct sunlight–it gets unbearably hot inside because hot air cannot escape. High atmospheric pressure and denser atmosphere is just the result of this.

May 8, 2010 9:35 pm

It is very important to note that despite radically different compositions, both atmospheres have approximately the same dry lapse rate. This tells us that the primary factor affecting the temperature is the thickness of the atmosphere, not the composition. Because Venus has a much thicker atmosphere than Earth, the temperature is much higher.
No it doesn’t, all it tells you is that the temperature gradient through the atmosphere is approximately the same. The actual temperature reached depends on the boundary condition, it doesn’t follow that the thicker atmosphere will be warmer. The composition does have a controlling role, a nitrogen atmosphere of similar pressure and thickness on Venus would result in a cold planet (surface ~230K).
With a constant lapse rate, an atmosphere twice as thick would be twice as warm. Three times as thick would be three times as warm. etc.
No, this does not follow!
Now, lets keep the atmospheric composition of Venus constant, but instead remove almost 91/92 of it – to make the mass and thickness of Venus atmosphere similar to earth. Because lapse rates are similar between the two planets, temperatures would become similar to those on earth.
No, the temperature difference from surface to tropopause would be the same, not the actual temperatures.
This and the associated thread show quite clearly that an IR-opacity of the atmosphere is the cause of atmospheric heating, i.e. ‘the greenhouse effect’.
Titan has a thicker atmosphere (mostly N2) than Earth yet is much colder, surface T 95K.

Spector
May 8, 2010 10:00 pm

To get the atmosphere on Earth as dense as Venus, I think we would ‘need’ something like a nearby supernova to flash-boil the ocean. If most of the energy received went into boiling the water, then the total mass of the planet should remain about as it was. I do not know if the atmosphere would cool and the oceans would reform after this event or if the new condition would remain stable. It would be a moot point.

Alvin
May 8, 2010 10:06 pm

Ugh, stop quoting Wiki as a source guys.

DesertYote
May 8, 2010 10:12 pm

Nick Stokes
May 8, 2010 at 8:40 pm
Got it. Sorry for not seeing the point you were trying to make to begin with.

May 8, 2010 10:26 pm

Phil.
You must have forgotten your high school chemistry.
A mole is most definitely a non-dimensional number. It is 6.023 * 10^23 molecules.
One of the interesting properties of gases (Avogadro’s law) is that they all occupy about the same volume at the same pressure and temperature, regardless of molecular weight. That is why the ideal gas law does not have a term for mass.
http://en.wikipedia.org/wiki/Avogadro's_Law
Avogadro’s law (sometimes referred to as Avogadro’s hypothesis or Avogadro’s principle) is a gas law named after Amedeo Avogadro who, in 1811,[1] hypothesized that “Equal volumes of ideal or perfect gases, at the same temperature and pressure, contain the same number of molecules.” Thus, the number of molecules in a specific volume of gas is independent of the size or mass of the gas molecules.

CRS, Dr.P.H.
May 8, 2010 10:35 pm

Hansen presents a dust insulation model as the cause of Venus’ high temperature in his 1967 article:
http://pubs.giss.nasa.gov/docs/1967/1967_Hansen_Matsushima.pdf

May 8, 2010 11:00 pm

stevengoddard says:
George Turner –
Gas pressure P = nRT/V . There is no term for either mass or gravity.

stevengoddard says:
MaxL –
Gas pressure is created by the motion of molecules. As you approach absolute zero, molecules cease to move. So there is no pressure. Volume has to remain above zero, because the molecules have a finite size. So yes, it is that simple.

Like special relativity, the gas pressure equation works well in the absence of gravity. Given the temperature and pressure at the surface of the earth, we should be able to calculate the volume of the atmosphere. V = nRT/P
I must be doing it wrong, because I get different volumes for toasty sea level Houston and cool mountainous Salt Lake City. I do get the same surface pressure, though.

May 8, 2010 11:11 pm

Mike McMillan
The ideal gas law works just fine, and gravity has nothing, nada, zippo to do with the accuracy of the equation.
You are comparing different temperatures and pressures in Houston, so you simply plug in different numbers into the equation. The ideal gas law works just as well in Timbuktu as it does in Salt Lake City.

jaymam
May 8, 2010 11:13 pm

Steve Goddard “If there were no Sun (or other external energy source) atmospheric temperature would approach absolute zero.”
But planets the size (or more) of the Earth and Venus have a significant internal heat source from radioactivity, which surely would heat the atmosphere.
Lord Kelvin, without knowing about radioactivity, calculated the age of the Earth as 40 million years.
In 1904 Rutherford, allowing for radioactive elements in the Earth’s core, calculated the age of the Earth in billions of years.

May 8, 2010 11:44 pm

jaymam
The amount of energy coming out of the earth is 44.2 × 10^12 W
http://www.agu.org/pubs/crossref/1993/93RG01249.shtml
The amount of energy reaching earth from the sun is 1.366 kilowatts per square meter * 510,072,000 km2 * 1,000,000 m^2/km = 69 x 10^16 W
http://en.wikipedia.org/wiki/Sunlight
The sun contributes more than 10,000 times as much energy to the atmosphere than does the internal heat of the earth.

Eli Rabett
May 9, 2010 12:11 am

Expanding on the points that Leonard Weinstein and Nick Stokes make. What is being discussed is the dry adiabatic lapse rate, which starts from the assumption that everywhere in the atmosphere the radiative energy absorbed is balanced by the energy emitted. If not, you get convective energy transport to re-balance the situation. That leaves gravitational compression, but it is not saying that radiative energy flow is negligible, just that it is balanced. As Nick and Leonard say, the lapse rate does not set the surface temperature, which is determined by the solar radiation absorbed at the surface and the IR re-emitted by greenhouse gases in the atmosphere.
What does the lapse rate determine? Looking at extremes is useful. Nick points out that if the surface is at absolute zero the thickness of the atmosphere (at least for an ideal gas) would be zero. What if the surface were at infinite temperature. Then the height of the atmosphere would be infinite also. From this, we conclude that the height of the atmosphere is determined by the lapse rate and the surface temperature
There are, of course, a few other things to consider. First, the dry adiabatic lapse rate is not 10 K/km for every atmosphere. For example it is 4.5 K/km on Mars and 2.0 K/km on Jupiter. Better put it is the ratio of the local gravitational constant divided by the specific heat (in J/(kg-K) or g/Cp. For the case of Venus, where the surface and the atmosphere are very hot, we have to account for the contribution of molecular vibrations to the specific heat, which will change with temperature and thus with altitude.

jcrabb
May 9, 2010 12:22 am

As day and night temperatures are so similar on Venus something must be maintaining the night time temperatures, especially considering the Venusian night is around 120 Earth days long, surely without the direct input of heat from the Sun over this period a significant amount of heat would be lost, unless there was a significant Greenhouse effect at work.

pft
May 9, 2010 12:46 am

On the subject of ocean depth, liquids are certainly compressible, just not as compressible as gasses. At 6 km depth for example, water would be at 600 atmospheres, density increases 3.7% and temperature would be 10 deg C above what water at 1 atmosphere AND the same heat content would be. Yet the temperature is only 3 deg C in the ocean deep. The only thing keeping water from freezing at ocean depth would seem to be waters compressibility, since little or no heat from the surface is reached at the bottom except via some convection, in fact it’s main heat source is the heat from the ocean floor and hydrothermal vents .
Air at 30,000 ft is at quite a low temperature, due to it’s low density and pressure, yet it’s heat content on a per molecule or mass basis is close to that at the surface. If it were adiabatically compressed to 1 atmosphere (ie, w/o losing heat), it’s temperature would be close to air at the surface. This points to the difference between heat and temperature.
In the upper atmosphere well above the stratosphere, temperatures reach 1000 deg C, yet air density is so low, you would feel little heat, as the mass and thus heat density is low. If you were able to survive the lack of oxygen, lethal radiation and needed no oxygen, you would quickly lose all your body heat despite the high temperatures. A real world example would be that of heating a thin needle to several hundred degrees C. It will not burn your finger since it’s heat content is so low (little mass) it dissipates quickly before it can do any damage.
Temperature of air at 2-3 meters (or 14,000 ft via satellite) or on the surface of the oceans is a very poor way to determine if our planet is heating. Temperatures can increase or decrease locally, or even in aggregate over short time periods (decades) but tell you little about if the heat content of the planet is increasing or decreasing.
The oceans are tremendous heat sinks with a heat capacity 50 times greater than the atmosphere. Heat gained by the atmosphere is quickly be absorbed by the oceans and temperature increases would be minimal as a result of evaporation and downward transport of heat due to increased salinity. In fact, the ocean surface area actually warms the atmosphere in the higher latitudes since it is at a higher temperature than the air. If not for the oceans ability to transport heat to the higher latitudes, we would be in a permanent ice age. Most of the Northern Hemisphere releases more energy to space than is received from the sun. It is only at the tropics that the energy balance is positive, and weather and the oceans then transfer this energy to the higher latitudes.
Attempts are made to determine the heat capacity of the ocean, but these are uncertain estimates and changes exceed the uncertainty of the estimates.

volauvent
May 9, 2010 12:51 am

P is given by the gravity (no gravity, the atmosphere escapes in space)
T is given by the heat equilibriums which are depending from many process including density, thus pressure… radiation and initial heating by the sun of course.
Only V could be the consequences of the others because there is no solid shell around the earth.
So you can’t prove anything from a general PV=nRT; you must go the the differential equations in each dV portion of the atmosphere (what are doing the models, but I do not say that the models are doing this well!).
Imagine that T is close to zero, with an atmosphere, PV/n is close, to zero , V/n is close to zero , so the change would be a very high density of the atmosphere. (mostly going to liquid under some temperatures; that’s happening for water, after all , with our current temperatures)
I am not sure that it is usefull to compare earth and venus as the conditions are so different, especially the content of the atmospheres and the very different temperatures.

kadaka (KD Knoebel)
May 9, 2010 12:56 am

Phil. said on May 8, 2010 at 8:50 pm:

Steve Goddard says:
May 8, 2010 at 7:27 pm
Mole is a non-dimensional number. A mole of H2 weighs much less than a mole of UF6

The mole is the SI unit of the amount of substance, it is not non-dimensional, a mole of a substance has a mass equal to the substance’s molecular weight.

A mole is just a number, a commonly used divisor when talking about a count of something.
You have a quantity of identical objects, be they atoms or ping-pong balls. You can describe the count in terms of thousands, millions, quadrillions, even moles. No one would argue a term like a quadrillion has a dimensional quality, in use you just take the count, divided it by a quadrillion, report that number. Exact same thing with a mole, you take the count, divide by a mole (6.02214 x 10^23), report that number.
3.28 * 10^24 = 5.45 moles. Left side is a count, an ordinary number. It has no dimensional quality. How can you argue the right side is not non-dimensional?

Troels Halken
May 9, 2010 12:58 am

Dr A Burns,
“The atmosphere is not static and conductive heat transfer to surroundings is small.”
Movement requires energy, and hence the energy must come from somewhere. After the initial compression of the gas, which releases energy in the form of heat, there has to be a heat/energy source to keep the gas warmer than the surrounding space.
I have no idea of how well insulated a planet is. Thinking about earth on a starry night, it is hardly perfect.

Dave McK
May 9, 2010 1:29 am

stevengoddard says:
May 8, 2010 at 9:02 pm
Dave McK
You can stop hyperventilating now. The average measured lapse rate on Earth is a little lower than the dry lapse rate. About 6.5.
————————————————
The adiabatic lapse rate of dry air is -9.8K/km
You are calling 2/3 ‘little’ and avoiding to name the real number.
Sir- 66% of something is significantly less than 100%.
The effect is due to water and the hadley pump that moves the working fluid from the heated surface to the infinite heat sink of space.
Don’t try to belittle me- you act like a damn climatologist.

Bob_FJ
May 9, 2010 1:30 am

George Turner Reur May 8, 2010 at 9:23 pm

[1] If the atmosphere on Venus stopped circulating, then I agree that the compression effects would dissipate.
[2] But the atmosphere there is constantly rotating in two giant Hadley cells. The air rises near the equator, moves to the poles, descends, and moves to the equator again, so it is always undergoing cycles of compression and expansion. This means it is also undergoing cycles of adiabatic heating and cooling.

[1] Yep. Any “initial point-in-time compression” of the atmosphere will result in increased T as a consequence of work done. (= mechanical energy). However, that “initial consequent heat” will be lost to the colder atmosphere above, (per thermo law 2), and for sustenance of it, equal replacement work energy in renewed air compression is necessary.
[2] At first sight that might partly explain an equatorial to poles redistribution of heat, but does not explain the reported daylight-time to night-time redistribution in terms of the ESA mapping of surface T‘s . However, how does your assertion fit in with the well reported large polar vortices, and how does it redistribute heat to mid latitudes?
Please elaborate on your assertion that Hadley cells arise at the equator and descend at the poles, (without any loss of T!!!), and provide a reference to show that these have been observed, including at what altitudes.
If you can answer that, I’ll elaborate more on the above.

Dave McK
May 9, 2010 1:37 am

The surface of Venus is dominated by volcanism. It has a crappy heat pump to lose the heat.
About 80% of the planet consists of a mosaic of volcanic lava plains, dotted with more than a hundred large isolated shield volcanoes, and many hundreds of smaller volcanoes.
Volcanoes less than 20 kilometres (12 mi) in diameter are very abundant on Venus and they may number hundreds of thousands or even millions.
Forget about greenhouse, dude. You aren’t helping.

Amino Acids in Meteorites
May 9, 2010 1:52 am

I’d say there’s not too much reason to be concerned, unless global warming was too big or too small. Other than that forget about it.

Dave McK
May 9, 2010 2:36 am

http://www.engineeringtoolbox.com/spesific-heat-capacity-gases-d_159.html
Gas or Vapor kJ/kg
Air 0.287
Carbon dioxide 0.189
Water Vapor 0.462
Steam 1 psia.
120 – 600 oF
That’s what it takes to change the temperature 1 degree K.
When CO2 changes from 1 to -1 C, a change of 2 degrees C, it radiates 2(0.189 kJ/kg) = 0.378 .
http://en.wikipedia.org/wiki/Enthalpy_of_vaporization
When water vapor changes from 1 to -1 (and condenses) it radiates 2257 kj/kg + 2(0.462 kJ/kg) = 2257.853776 kJ/kg.
It does this every single time you see a cloud.
But CO2 has no phase change so it carries no heat – the numbers:
All gases at the same temperature have the same number of molecules per unit volume. (Avogadro)
Water, being light, masses 18g/mole and CO2 masses 44 g/mole
Using 1 mole of air, just to make math easy:
We lowball the water in the atmosphere at 1% of the molecules
So, in a mole of atmosphere, we have 0.01 moles of water = 0.18g
now we highball the CO2 at 500ppm which is 0.0005, or 1/2000 of a mole of CO2.
1/2000 * 44g/mole = 0.000484 moles of CO2 = 0.021296g
So in our mole of air with but 1% H2O and a generous 500ppm CO2-
the water condensing radiates 0.18g * 2257.853776 kJ/kg = 406.41367968 J
while the CO2 radiates 0.021296g * 0.378 kJ/kg = 0.008049888 J
the ratio of 0.008049888/406.41367968 = .00001980712855516645290496438242332
or as much to say that water vapor in the example carries 50486.873814890343815963650674393 times more heat than the CO2 does.
And that’s just rain. If it turns to snow- multiply by 5-6.
Meanwhile, Venus is a ball of active volcanoes with a dry heat pump to radiate it poorly.
That is why Earth’s climate doesn’t resemble that of Venus.
Forget about CO2.

Gareth
May 9, 2010 2:53 am

Two fascinating posts along a similar lines to this one elsewhere.
They raise an interesting idea in my head. Are our notion of ice ages wrong? An increase in glaciation (for whatever reason) reduces sea levels. As sea levels sink the temperate zone where mammals and plants can most easily thrive moves with it onto what was formerly sea floor. What happens to temperatures? If the place where you are getting your proxy data from doesn’t move it would record a dropping temperature due to the atmosphere there getting thinner but there would still be plenty of surface with a thick and warm atmosphere.
Conversely, increases in sea levels (for reasons of less glaciation or volcanic and tectonic activity displacing water) would increase the apparent temperature for any given location that stays above water. The atmosphere appears to get thicker because it is being pushed upwards.
Are sea level changes factored in to temperature reconstructions?
Troels Halken said: When you compress or decompress a gas, it either warms or cools. Due to the temperature differential to the surroundings, after some time it will cool or warm by exchanging heat with the surroundings until the temperature differential is approaching zero and hence the temperature of the gas approaches that of the surroundings.
Why should Venus behave any different?

Convection currents on Earth cause warm air to rise, expand and cool whilst pushing cool air down, compressing and warming it. If Venus has convection currents it is not behaving any differently.

kwik
May 9, 2010 3:41 am

Dave McK says:
May 8, 2010 at 8:42 pm
Dave, what was the point of that “rant”?
Did you appeal to authority? Or that “they” have more money?

Mike M
May 9, 2010 3:44 am

stevengoddard says: “The ideal gas law works just fine, and gravity has nothing, nada, zippo to do with the accuracy of the equation.”
Yes, the equation itself works fine because it has nothing to do with gravity. But in this case pressure is a function of gravity. You are ultimately implying, (as I understand you), that the mass of a given control volume of air in a gravitational field decreases with temperature; that one half the temperature will give us one half the pressure at the surface. That sir is impossible. The amount of air remains the same so its weight remains the same and so the pressure at the surface remains the same.
I have two volley balls, (the reason air exists! ;), pumped them up to the same pressure at the same temperature – they weigh the same. If I cool one off to half (K) temperature of the other, it shrinks to half the volume of the other – but they still weigh the same. Instead of volley balls, we have two columns of air as control volumes in earth’s gravitational field with rigid walls and open at the top to vacuum space. They are each 1 inch square with the same amount air at the same temperature in each. The weight of the air in each of them exerts 14.7 pounds of force at the bottom and the very top-most molecule in each of them is at the same altitude. If we cool one column to say half the temperature of the other, the altitude of its top-most molecule will decrease to roughly one half the altitude of the other, (I’m not certain of the exact height but it will be a lot less than the other one).
I submit that the pressure in each column AT THE SURFACE will remain the same. As we go up in altitude however, that’s where we see a difference in pressure bewtween the two columns. The warmer column has a higher pressure than the cooler one at any altitude above the surface. The difference is zero at the bottom and increases the higher we go until it becomes infinite beyond the top of the cooler column.
But of course there are no walls up there, the air from the warmer column, having a greater pressure at altitude than the cooler one – flows into the cooler column increasing its weight and the measured barometric pressure at the surface.

May 9, 2010 4:10 am

Dave McK
There are more than 50 active volcanoes on Earth’s land surface, not to mention hundreds under the oceans. Their tiny contribution to the energy budget is much less than the measurement error.
http://www.geo.mtu.edu/volcanoes/world.html

May 9, 2010 4:22 am

The lapse rate on Venus is constant to 60 km. On Earth it is constant to about 14 km. Surface pressure on Venus is 92 times larger than Earth.
60/14 = 4.28
ln (92) = 4.52
Those numbers seem to be pretty close together, and hint that pressure differences explain the height of the tropopause, rather than greenhouse gas concentrations.
Earth’s atmosphere is mainly opaque to IR due to water vapour. Venus atmosphere is mainly opaque to IR due to spectral broadening of CO2 under high pressure. The addition of more greenhouse gas should be a minimal effect compared to the pressure differences.

May 9, 2010 4:22 am

Gareth,
Thanks for that interesting link:
http://www.countingcats.com/?p=4745

Mike M
May 9, 2010 4:25 am

” Dave McK says: Troposphere, shmoposphere – clouds, rain, snow.”
I concur, 99% of the difference between the atmospheric temperatures of Earth and Venus is because of that amazing stuff called water. Without water I’d guess that we’d be almost as hot as Venus no matter how much CO2 there was. To me, the difference in temperature illustrates the immense power of water’s negative feedback.
People understandably have a lot of trouble wrapping their minds around the idea that something can cool off or warm up without losing or gaining heat and I therefore surmise that to be the reason so many choose to ignore the latent heat of water in vapor form being carried up adiabatically to higher altitudes.

David Bailey
May 9, 2010 4:26 am

Anthony,
I think this could be a really big nail in the coffin of climate change because it demonstrates more clearly than anything else that the normal critical processes that operate in science haven’t worked in “climate science” for some time! Can you get this published in a journal somewhere?

May 9, 2010 4:35 am

Mike M
Without water the earth would be much hotter, but not for the reasons you mention.
If there were no oceans, no limestone would have formed, and the atmosphere would be much more dense.

May 9, 2010 4:37 am

Gareth
I was thinking the same thing. Small differences in atmospheric pressure could explain the early faint sun paradox and possibly ice ages as well. Certainly something worth looking into.

Mike M
May 9, 2010 4:41 am

My prior comment, the phrase “infinite difference”, I meant ratio-metrically. (We need an edit feature!)

Jbar
May 9, 2010 5:00 am

Nick Stokes,
The 700K surface of Venus doesn’t emit 12,000 W/m2. That would be (close to) the emittance of a black body radiating at near 700K, but Venus’s surface is NOT a black body.
Venus’s surface must be 733K to remain in radiative balance with sunlight because almost the entire infrared spectrum is blocked by the CO2 atmosphere and water vapor at high altitude. When you block parts of the infrared spectrum, the Stefan Boltzmann law no longer applies. Instead of having a simple formula based on T^4, now you have to integrate over the whole emission spectrum wavelength by wavelength, excluding the wavelengths that are blocked.
When you start blocking parts of the emissions spectrum, it takes a much larger increase in temperature than given by the T^4 law to emit the same amount of energy.

Jbar
May 9, 2010 5:12 am

Doc Martyn:
There are clouds because of the atmospheric lapse rate that Steve Goddard talks about.
The lower atmosphere is too hot for certain compounds to condense, but the upper atmosphere (50-70 km altitude) is much cooler, allowing them to condense into clouds. (Example: the pressure at 50km altitude is just below 1 atmosphere and the temperature is 57C.) At the altitudes where these clouds exist, there is very likely a phenomenon of solar energy vaporizing the clouds and heating the atmosphere, the heated gas rises to a higher altitude but cools thanks to the lapse rate, forming new clouds.

May 9, 2010 5:22 am

Jbar
Your arguments don’t work.
Night on Venus is thousands of hours long, and the temperature is the same as the day side – which doesn’t receive much SW either.
Almost all of the IR is absorbed by Earth’s atmosphere, but we seem to be able to keep temperatures below 733K.

DavidB
May 9, 2010 5:26 am

There seem to be two possibilities here:
either
(a) Steven Goddard has made a major, Nobel-worthy breakthrough in atmospheric physics
or
(b) he has fundamentally misunderstood and misapplied the ‘ideal gas law’.
As a scientific layman I am not competent to say which, though, like Quasimodo, I have a hunch.

May 9, 2010 5:29 am

Jbar says:
May 9, 2010 at 5:00 am

You’re right when the atmosphere blocks IR. I was referring to the supposed GHG-free atmosphere.
Alternatively, it is still true that the surface emits (about) 12,000 W/m2. The S-B law applies at that level. Of course, many wavelengths are than quickly absorbed in the real atmosphere. But they were emitted.

Jbar
May 9, 2010 5:30 am

Dave:
May 8 6:48 PM Steve Goddard more-or-less states that there are both lapse rate and greenhouse effect. (Steve:”I’m not sure where anyone got the idea that I am suggesting that there isn’t a greenhouse effect on Venus. It certainly wasn’t from anything I have written. This is a discussion of relative magnitude.”)
So from that I take it his opinion is that it’s just a question of what share of the high surface temperature is due to each effect. However, he doesn’t really say what % is lapse rate and what % is greenhouse. I think none of us in this forum have to tools to calculate that. (Although I’ll take a wild guess its 50/50, plus or minus 25% either way. Does that help???)

May 9, 2010 5:33 am

I’ve written a post here to try to explain the close relation between the adiabatic lapse rate and the greenhouse effect. It doesn’t make sense to say that warming is due to the lapse rate and not the GHE. They go together.

Tom in Florida
May 9, 2010 5:41 am

I have not seen any discussion as to WHY the atmosphere on Venus evolved differently than on Earth. To me that is an important question. If the evolution is different, then there there should be little fear that Earth will end up like Venus.
I understand that Venus has no magnetic field that has allowed the solar wind to strip away lighter elements including water vapor. Add in the slower, retrograde rotation and any comparison to how Earth might evolve starts to become meaningless.

Troels Halken
May 9, 2010 5:51 am

Gareth,
“Convection currents on Earth cause warm air to rise, expand and cool whilst pushing cool air down, compressing and warming it. If Venus has convection currents it is not behaving any differently.”
As I understand it, convection comes from air that is being heated (and hence expands and then rise as it is lighter that the surrounding air) either by the sunlight on the surface or by the sunlight itself or both. If we turn off the energy source the convection stops, which also happens on earth during the night. What this tells us, is that without an external energy source as the sun, the temp on the surface on the earth will drop rapidly due to energy being lost to space.

May 9, 2010 6:04 am

stevengoddard says:
May 8, 2010 at 10:26 pm
Phil.
You must have forgotten your high school chemistry.
A mole is moist definitely a non-dimensional number. It is 6.023 * 10^23 molecules.

Clearly then it is not dimensionless, as I said it is the unit that represents the amount of a substance.
One of the interesting properties of gases (Avogadro’s law) is that they all occupy about the same volume at the same pressure and temperature, regardless of molecular weight. That is why the ideal gas law does not have a term for mass.
Of course it does!
PV=mRT/M where m is the mass of the gas and M is the molar mass.

Jbar
May 9, 2010 6:07 am

Spector:
Let’s do a thought experiment. If a “supernova flash-boils the ocean” as you suggest, then you have an extremely high water vapor concentration in the atmosphere causing a surface pressure of 265 atmospheres (including N2 and O2). At that concentration, water vapor’s capacity to block infrared would turn Earth into a Venus, at least temporarily, and the oceans might never condense again. At that pressure, surface temperature would have to fall below 390C for water to condense.
On the other hand, the whole atmosphere would be enshrouded in water vapor clouds. Some clouds reflect light but others trap infrared, so whether or not this brave new world would stay stuck in “runaway greenhouse” mode or reflect more light back into space and cool back down (allowing the oceans to reform) depends on what type of clouds dominate. Even if the globe cooled enough to form oceans, there might still be enough water vapor left in the atmosphere to keep the surface at some higher pressure and temperature than today (greater than 100C), depending on the balance of cloud types and global albedo.
Either way, the intense blast of radiation from any supernova close enough to boil the oceans would directly kill every living organism on the planet instantly. Game over. Reboot.

Jbar
May 9, 2010 6:14 am

Jaymam:
Energy emissions from the Earth’s interior amount to 1/10th of a watt/m2, thousands of times less than the surface radiation from the sun. This energy source is only significant to the behavior of thousands of miles of rock with a low heat transport rate, not the atmosphere or oceans with their much higher heat transport rates.

Editor
May 9, 2010 6:24 am

stevengoddard says:
May 8, 2010 at 5:53 pm

Ian L. McQueen
Joules is a unit of energy
Degrees Kelvin is a unit of temperature
moles is a non-dimensional number – 6.022 x 10^23 molecules
None of these are units of mass or gravity.

I’m not sure what people are arguing about at this point, but let me try to set some basics right. Much of physics can be reduced to the basic units of mass, length, and time. Before the term “SI” replaced “metric”, i.e. back in my high school physics class, we talked about measuring things in CGS (centimeter, gram, second) vs. MKS (meter, kilogram, second) units. Maybe also FPS (foot, poundal, second) but let’s not go there. I’m not sure why we didn’t include temperature, probably because we were looking at static and dynamics (hanging signs, weights on massless ropes a pulleys and baseballs thrown on flat, airless planets).
So, the ideal gas law is
pV = nRT
From Wikipedia (which is a good source for some stuff like this), In SI units, p is measured in pascals; V in cubic metres; n in moles; and T in kelvins. R has the value 8.314472 J·K^-1·mol^-1.
A pascal is pressure, and is newton per meter squared. A newton is force, and its units come from F=ma, so mass times distance per time squared, or g·m·s^-2.
Therefore, a pascal is g·m^-1·s^-2 due to dividing be square meters.
Volume is m^3, n is dimensionless (a number of molecules, but granted what I call a “honorary unit” of mol), temperature just K.
R has to make the dimensions balance. Wikipedia uses Joules in its J·K^-1·mol^-1. A joule is a unit of energy, which is force times a distance (parallel to the direction of the force!). We have force above, so a joule’s dimensions are g·m^2·s^-2.
Does all this balance? pV is g·m^-1·s^-2 · m^3 = g·m^2·s^-2
nRT is g·m^2·s^-2·K^-1·mol^-1 · mol · K = g·m^2·s^-2
Yay – it balances. Mass is a basic unit in pressure and energy.
However, mass can be ignored for looking at small quantities of gas as it is compressed and expanded adiabatically. It cannot be ignored when looking at tall columns of a gas, e.g. a column of an atmosphere, because the mass above any point in the column determines the pressure at that point.
Assuming a well mixed column, as we seem to have at Venus thanks to surface heat causing convection to mix the troposphere, then given the temperature at any point in the column, we can determine the temperature when a handful of gas there is moved anywhere else in the column.

Jbar
May 9, 2010 6:27 am

jcrabb
Most of the action in Venus’s atmosphere takes place in and above the cloud layer (50-70km altitude). Here clouds absorb sunlight on the sun side and race around at high speed to emit infrared radiation on the night side, keeping the temperature of this high atmospheric layer relatively uniform in spite of the slow rotation of the surface. The uniform temperature of this high altitude zone helps to set the uniform temperature distribution of the lower atmosphere and surface. (Not sure of the details of that.)
Venus’s atmosphere is highly refractive, so that sunlight wraps around the planet from the sun side all the way to the dark side, much much more than the dawn/dusk effect on Earth (although I have no idea how much sunlight actually reaches the dark side). That provides some solar heating of the surface even on the dark side. However, I am still at a loss to explain why the surface temperature is uniform all the way around unless the high refractivity of the atmosphere makes the surface lighting uniform. (Doubtful)

Jbar
May 9, 2010 6:32 am

pft
The ocean does not freeze at the bottom because water is most dense a few degrees above freezing. It cools at the surface of the ocean and then sinks down before it freezes. It cannot freeze then because there is no mechanism to cool it to a lower temperature down there.
We’re very lucky that water works this way or ice would continue to pile up on the ocean floor until all the oceans and lakes would be entirely frozen over and our planet would be an ice ball.

Editor
May 9, 2010 6:33 am

stevengoddard says:
May 8, 2010 at 5:53 pm
Degrees Kelvin is a unit of temperature
<rant>
There are degrees Celcius, there are degrees Fahrenheit, there are no such things as degrees Kelvin. The term is just kelvins, lowercase! Otherwise, for consistency’s sake you’d want to talk about length meters (aren’t those rulers?), time seconds (which almost makes sense to distinguish from angle seconds), and frequency hertz (units named for people are always lower case).
If we’re going to get the basics right, lets start with the names.
</rant>

May 9, 2010 6:37 am

Jbar says:
May 9, 2010 at 5:00 am
Nick Stokes,
The 700K surface of Venus doesn’t emit 12,000 W/m2. That would be (close to) the emittance of a black body radiating at near 700K, but Venus’s surface is NOT a black body.
Venus’s surface must be 733K to remain in radiative balance with sunlight because almost the entire infrared spectrum is blocked by the CO2 atmosphere and water vapor at high altitude. When you block parts of the infrared spectrum, the Stefan Boltzmann law no longer applies. Instead of having a simple formula based on T^4, now you have to integrate over the whole emission spectrum wavelength by wavelength, excluding the wavelengths that are blocked.

Whether the atmosphere is blocked has nothing to do with the emissivity of the surface, which remains a near black body.

Jbar
May 9, 2010 6:43 am

A “mole” is shorthand for chemists to make it easier for them to do their maths.
It is a number of grams equivalent in value to the molecule’s molecular weight. Eg. one gram-mole of water = 18 grams of water (approx), same as the molecular weight of water.
A mole of any substance contains the same number of molecules – Avogadro’s number – 6.022 E23 molecules.
So if a chemist knows that one molecule of oxygen (O2) reacts with two molecules of hydrogen (H2), they know that they need one mole of oxygen (16 grams) to react with 2 moles of hydrogen (2 grams) to make 1 mole of water (18 grams).
It is much more convenient than counting molecules!
Engineers play with bigger numbers than chemists so US and British engineers like to use “pound-moles”, or the number of pounds with the same numeric value as the molecular weigh.

Editor
May 9, 2010 6:47 am

Oh – I forgot to mention a couple posts back about my “honorary” units. I’ve found them handy in some calculations and in “dimensional analysis” to give units to technically dimensionless quantities. The simplest example is pi, 3.14159….
Pi is dimensionless, so the equation circumference = 2 x pi x radius is just length on both sides. If we use the honorary units of radial meters and circumferential meters, then pi takes on units of circumference/radius and becomes easier to track. You can also thrown in units for diameter and radius lengths to keep the “2” involved in the dimensional analysis.
Dimensional analysis, by the way, is something I learned quickly in physics class, but isn’t taught very well. I remember reading an article in Science News decades ago discussing that and claiming that students taught it understood physics much better and applied various formulae correctly.
One thing I never did learn, and was in part my downfall in college physics, is the Greek alphabet. Both upper and lower case. Reading equations like “P = squiggle times squiggle-with-extra-zig divided by lambda” was not the way to learn that stuff. At least I know lambda, well, lowercase lambda anyway.

jaymam
May 9, 2010 6:56 am

The amount of energy coming out of the earth is 4.42E+13 watts
The amount of energy reaching the top of the atmosphere from the sun is 1366 watts/m2
The average albedo of the Earth is about 0.3 therefore 410 watts/m2 is not reflected
Area of Earth’s surface 510,072,000 km2 = 5.1E+14 m2
Area of Earth with the sun shining on it is 4 times less, i.e. 1.275E+14 m2
Energy from the sun is 1.275E+14 * 410 = 5.22E+16 watts
5.22E+16 divided by 4.42E+13 = 1182, or 0.085%
0.085% should not be ignored.

May 9, 2010 7:05 am

jcrabb says:
May 9, 2010 at 12:22 am
As day and night temperatures are so similar on Venus something must be maintaining the night time temperatures, especially considering the Venusian night is around 120 Earth days long, surely without the direct input of heat from the Sun over this period a significant amount of heat would be lost, unless there was a significant Greenhouse effect at work.

The relatively small difference between day and night temps intrigues me as well. Wouldn’t a reasonable assumption be that the thick atmosphere is circulating so fast that it is distributing energy around the planet so that convective heat distribution is dominant?

Jbar
May 9, 2010 7:06 am

Dave McKay
The 380 ppm of CO2 in our atmosphere absorbs ALL the infrared radiation between 13 and 18 microns wavelength trying to leave the surface of the Earth on its way to space. The CO2 then reradiates that energy in all directions, up, down, and sideways. (Anything going sideways has to travel through so much more atmosphere than what’s going straight up that it effectively winds up getting all reabsorbed, so the net effect is for the radiation to either pass up or be reflected effectively straight down.) So the CO2 acts like a few-percent reflective mirror of infrared surrounding the Earth.
The atmosphere of Venus contains 89,000,000 ppm (relative to Earth), which is enough to block all the infrared radiation between 2 and 28 microns wavelength except for 3 narrow windows of transparency and everything longer than 28 microns. So even a few kilometers of Venus’s CO2 atmosphere absorbs the majority of radiation trying to leave the surface and reradiates it in all directions (but, effectively, 50-some% up/ 40-some% down). Then the next few km higher does it again, and the next few km, and the next, until the radiation finally gets to the top of the atmosphere and can get out freely into space. So the CO2 in Venus’s atmosphere acts like a tall stack of 40-50%-reflective infrared mirrors.
There’s water vapor high up in Venus’s atmosphere, and that water is enough to block all the infrared radiation longer than 28 microns, the big chunk that CO2 misses, adding H20-greenhouse insult to CO2-greenhouse injury.

björn
May 9, 2010 7:12 am

If the seas of Venus boiled in a runaway greenhouse meltdown, where is the water?

Enneagram
May 9, 2010 7:16 am

A different approach to Venus:
https://ssl.scroogle.org/cgi-bin/nbbwssl.cgi

Onion
May 9, 2010 7:20 am

Without greenhouse gases in Venus’s atmosphere, Venus would be hundreds of degrees cooler. That means Venus is that hot because of the greenhouse effect.
Talk about pressures and lapse rates is just dancing around the above fact.

Jack Morrow
May 9, 2010 7:50 am

Steve Goddard
I guess all those moles or co2 is causing all your cold weather in CO.

Marc77
May 9, 2010 7:55 am

Based on this text and others, it seems Venus is hot because of its thick atmosphere. The cause of this runaway-building-of-a-thick-atmosphere might be its proximity with the sun. Now it is in equilibrium because of the high albedo of its atmosphere(negative feedback).
The way I understand the adiabatic lapse, I might be wrong, is like this.
If you have a differential of pressure through a column of gas, you have more molecules on one side. I the gas is well mixed, all molecules will have the same distribution of chaotic-kinetic-energy(heat). So the side of the column with more molecules will have more heat.
Now if there was no movement, after some time I guess the warmest molecules would get on top and the coldest at the bottom.
So basically, the system has to be somewhere between those two tendencies.
I also took a look at what a supercritical fluid is. Based on wikipedia, “In addition, close to the critical point, small changes in pressure or temperature result in large changes in density, allowing many properties of a supercritical fluid to be “fine-tuned” “. So it seems a supercritical fluid CO2 has the compression properties of a gas, not of a liquid.
Finally, there is also a place on Earth where temps are high, under the crust. It is also possible that Venus atmosphere could be a good insulator just like Earth’s crust, so the geothermal heat could build there. Just like I believe the geothermal heat could build under a thick layer of ice on Earth, but this is another subject.

Spector
May 9, 2010 8:11 am

I have often thought that Venus, being much closer to the sun than the Earth, must intercept a larger flux of gases from the solar wind and the extreme outer layers of the solar atmosphere. Perhaps the atmosphere of Venus is as dense as it is because that planet has had a better opportunity to scoop up gases from the sun. [My speculation.] For some gases, there may be equilibrium condition where the rate at which each gas is acquired is balanced by the rate it escapes back into outer space.
If the Earth had the same surface pressures as Venus, I am sure that the ground temperatures would be much higher here also. I do not think CO2, per se, is the real problem.

Pascvaks
May 9, 2010 8:15 am

On the motion to the motion to the motion..
Everyone in favor of NASA being given the mission of diverting a comet into Venus in the next two decades (and thereby gaining the wherewithall to save us from a similar impact) and beginning the transformation of Venus into a habitable planet say YES!
Those opposed?
The motion is carried and will be open for discussion for the next session, following the Memorial Day and 4th of July Break! One hour, equally divided will be allotted to the discussion by the Committee of the House.

GeoFlynx
May 9, 2010 8:17 am

So let me get this straight, you have written two “Venus” blogs that have displayed your misunderstanding and misapplication of the ideal gas laws, and yet you wish everyone to believe that your insight on that planet trumps all the scientists at NASA and JPL?
REPLY: Still snotty and condescending but better than the last one that was snipped

Ralph
May 9, 2010 8:25 am

>>>With a constant lapse rate, an atmosphere twice as thick would
>>be twice as warm. Three times as thick would be three times as warm.
>>>etc. Now let’s do some experiments using this information.
Without wishing to stir up too much controversy (remember last time !) , this is the same argument as Willis Eschenbach’s Steel Greenhouse posting (below). Willis doubled the number of steel spheres, to do exactly the same thing – double the temperature on the surface.
http://wattsupwiththat.com/2009/11/17/the-steel-greenhouse/
Pretty obvious, really.
.

May 9, 2010 8:31 am

GeoFlynx
If you have a specific objection, please post it. There is no constructive way to respond to ad hom attacks.

May 9, 2010 8:32 am

Jack Morrow
It is nice today, but supposed to be 45 degrees for a high on Wednesday. That is close to normal for January 1.

May 9, 2010 8:35 am

jcrabb
Some greenhouse you are describing. It doesn’t lose even one degree of temperature over 1400 hours of pitch black. You should patent it. You will make a killing.

OkieSkeptic
May 9, 2010 8:37 am

Dry lapse rate looks suspiciously like ‘thermal gradient’ and this gradient shouldn’t occur without a thermal difference between the upper atmosphere (at space) and the surface. From this viewpoint, if the top of a dry atmosphere were at the same temperature as the surface, there would be a pressure difference from gravity but no thermal gradient. That gradient in the Venus example should be mostly from solar heat trapping at the bottom of the atmosphere.
One major difference between Earth and Venus is Nitrogen content and Venus would have probably have been entirely different if it had as much as the Earth’s atmosphere. The million dollar question is how Venus and Mars managed to be without it as it was abundant in the early solar system.

Squarebob Spongepants
May 9, 2010 8:37 am

DaveMcK says:
When water vapor changes from 1 to -1 (and condenses) it radiates 2257 kj/kg + 2(0.462 kJ/kg) = 2257.853776 kJ/kg.
It would be a wonderful world if it did radiate heat like that – but almost always the heat is conducted by cold surfaces (having already radiated the heat away) or transformed into kinetic energy warming supercooled gases, which have also already radiated the heat away.

May 9, 2010 8:43 am

Ralph
Actually the argument is different from what Willis posted. I’m not discussing the radiative budget – rather just the basic physics of gas warming as it moves to areas of lower pressure lower in the atmosphere. The higher the pressure, the higher the temperature.

DesertYote
May 9, 2010 9:06 am

Dave McK
May 9, 2010 at 1:37 am
OH NO! Its worse then we thought. The green house affect is melting the planet!

DesertYote
May 9, 2010 9:34 am

Tom in Florida
May 9, 2010 at 5:41 am
I have pointed out here and in another post that one really big difference between the Earth and Venus is our monstrously oversized moon which played a major role in the creation of our atmosphere and plays a major role in weather today, but no one seems to remember what was once understood science.

J.Hansford
May 9, 2010 9:41 am

OkieSkeptic says:
May 9, 2010 at 8:37 am
“……One major difference between Earth and Venus is Nitrogen content and Venus would have probably have been entirely different if it had as much as the Earth’s atmosphere. The million dollar question is how Venus and Mars managed to be without it as it was abundant in the early solar system.”
————————————————————————————
It’s always puzzled me that peculiarity….. Just why is it that Earth has 70% nitrogen in it’s atmosphere, whilst Mars and Venus barely have any?
G’dam mystery if you ask me:-)

Ralph
May 9, 2010 10:11 am

>>stevengoddard says: May 9, 2010 at 8:43 am
>>Ralph
>>Actually the argument is different from what Willis posted. I’m not
>>discussing the radiative budget – rather just the basic physics of gas
>>warming as it moves to areas of lower pressure lower in the atmosphere.
>>The higher the pressure, the higher the temperature.
Two sides of the same coin, is it not….
.

gallopingcamel
May 9, 2010 10:24 am

stevengoddard says:
May 8, 2010 at 2:57 pm
QUOTE
You can infer how thick the atmosphere of Venus is from this diagram:
http://www.astro.wisc.edu/~townsend/resource/teaching/diploma/venus-t.gif
UNQUOTE
That graph appears to support your hypothesis. It will be interesting to see what rebuttal James Hansen can come up with.
It looks as if there might be a “Goldilocks Zone” on Venus at an altitude of over 50 km!

Doug Proctor
May 9, 2010 10:45 am

With all these comments, some expanding, some disagreeing, some clarifying, I no longer know how much of the original Note stands. Would it be possible, once the Comments are closed, to modify as appropriate, with footnotes if necessary? This is, I suppose, the benefit of peer-review before publication: what remains is cleaned up and ready for distribution.

MaxL
May 9, 2010 10:50 am

Just one final post on the subject and I will leave it at that. When we meteorologists speak of “atmospheric pressure” here is what we are referring to:
This from the AMS Glossary of Meteorology http://amsglossary.allenpress.com/glossary
“atmospheric pressure—(Also called barometric pressure.) The pressure exerted by the atmosphere as a consequence of gravitational attraction exerted upon the “column” of air lying directly above the point in question.”
If others have a different definition of atmospheric pressure, then that should be stated at the beginning of their thesis so that we are talking about the same things.

dr.bill
May 9, 2010 11:10 am

Steve (Goddard),
Your insistence on relating pressure to molecular motion, and heating due to pressure, is ruining what should have been a good discussion. I believe that it also impedes your understanding of your own proposal. As many others have put forth, and I will re-iterate, the atmosphere is warmed from the bottom, and only then will things start moving up. It is not the other way around. Where the surface gets its heating from is another matter. It might be simple percolation of Solar energy down to the surface, whether directly or in the form of a bit of back-radiation from the atmosphere, or it might be vulcanism, or it might be something that the Borg are doing with slipstream drives, but it is the bottom that does the heating of the atmosphere, not the other way around.
Regarding pressure, it can be caused by molecules in motion. It can also be caused by the simple weight of whatever is sitting on something else. It can furthermore be caused by the application of any force of any kind from any source to whatever is feeling the pressure. We figure out the pressure at depth in the oceans, for example, by calculating the weight of the water above that depth, and then we add the atmospheric pressure that exists at sea level, which in turn is due to the weight, not the motion, of the air molecules stacked up above that. This is basic Physics. It doesn’t matter if the atmosphere is in the form of a gas, as solid, or a slurpy. It’s just its weight that determines the pressure at ground level, or the bottom of the ocean, nothing more.
I say this as a physicist who has taught many subjects, including Thermodynamics, over many decades, but you do not need to simply take my word for it. There are 10’s of thousands of textbooks that will tell you the same thing.
Regarding your original proposal, the data you presented shows that there is no need to invoke a “runaway greenhouse” effect on Venus, any more than there is on Earth. The graph of Temperature vs Altitude that you include looks very much like that of the Earth in the bottom two regions. There is a linear “lapse rate region” up to 60km, followed by an isothermal region for about 10km, much like the bottom part of our own stratosphere.
If one can extrapolate back down to the surface using the lapse rate that can be found from the graph, it would have a temperature of about 500°C or so, purely on the basis of ordinary atmospheric dynamics, with no need for massive effects of any other kind. Note again, that by extrapolating back down to the surface, I am not saying that the atmosphere of Venus heats up because you are going down. I am purely using the apparent straight line to estimate the value at its bottom end, given what the graph shows for its top end.
The higher regions of Venus depart from Earth’s pattern, but the lowest 60km looks very much like a “regular troposphere”, albeit much thicker than our own, which only shows that it is being heated more strongly, from below, than is the case on Earth. That “thickness effect” may also be seen on our own planet. As you likely know, the tropopause in our Polar regions is only at a height of about 8km, while in the Tropics, it is generally at a height of 16km, or even a bit more above very hot places. That is not a coincidence. It is simple cause and effect, and they need to be put in the right order, which you are not doing. Umbrellas don’t cause rain.
I do not like “getting personal or confrontational”, and I generally like the things you post on WUWT, but I think it’s time you stopped being so dogged about “being right”, and accepted the valid and well-intended interventions of many other posters who have carefully explained their points of view, only to be summarily dismissed by a one-liner from you. This isn’t very respectful, there is no need for it, and it simply impedes understanding.
/dr.bill

Roger Clague
May 9, 2010 11:44 am

The Greenhouse Effect (GHE) and the thermodynamic lapse rate.
The phrase GHE muddles together the effect and the suggested cause. It suggests that the warming effect of the atmosphere is caused by the chemical structure of some of the gases in air, molecules that are made of more than one type of atom, especially CO2.
There is warming in addition to that caused by the sun, calculated using the concept of energy balance and the Stefan/ Boltzmann Law. What causes the extra warming?
Steve Goddard’s recent posts say the extra warming is explained by the pressure of the atmosphere. There is no need to consider the chemical composition. All gases create pressure, on Earth mainly N2 and O2, on Venus mainly CO2. All gases contribute to the extra warming. All of the extra warming is explained by physics. There is no need to consider the structure of these gases.

Bruce
May 9, 2010 12:29 pm

Nice post. I like the Venera images. Since it’s VE parade day in Moscow, let’s toast a Russian aptitude for Polar scientific expeditions, space exploration, and the encirclement of Axis armies.

May 9, 2010 12:32 pm

dr.bill
How is Venus “warmed from the bottom” during it’s thousands of hours long nights?

May 9, 2010 12:36 pm

Venus has no oceans, so it has no limestone. No limestone means a thick atmosphere of CO2. A thick atmosphere means high temperatures.
It took decades for science to accept the obvious fact of continental drift. Scientists can be just as bone headed as anybody else.

Nullius in Verba
May 9, 2010 12:38 pm

Here’s a paper by Carl Sagan that discusses the Venusian ‘greenhouse’.
http://adsabs.harvard.edu/full/1967ApJ…149..731S
You might be surprised by the mechanism he proposes for it.
Doesn’t it look familiar?

LarryOldtimer
May 9, 2010 1:36 pm

Since neither Mars nor Venus has any planetary magnetic field, I would expect that the solar wind carries away any lighter gases, such as H2O vapor, leaving behind the more dense gaseous molecules. With it being far to hot on Venus for carbon based life, there is no conversion of CO2 to oxygen and carbon.
Venus is vastly different from Earth in most respects. To attempt to equate the workings of the atmospheres on Venus and Earth is an exercise in futility. Too, we know very little about the charastics and composition of Venus itself.
Back when I was a lad, from what I personally read (and I had a greal of interest in things scientific), it was speculated that the temperature of Venus was about 125 degrees F, and that Venus was a “wet” world, since the dense clouds were thought to be of water. Very warm and balmy, was the scientific speculation circa 1950 or so.
“A little knowledge is a dangerous thing.” ~ Alexander Pope
To attempt to “extrapolate” only a little knowledge into a “grand theory” is one of the most dangerous of all things that can be done..

dr.bill
May 9, 2010 2:03 pm

stevengoddard:
May 9, 2010 at 12:32 pm
dr.bill
How is Venus “warmed from the bottom” during it’s thousands of hours long nights?

I don’t know Steve,
Perhaps we should try to find out.
/dr.bill

May 9, 2010 2:06 pm

I am finding this thread as baffling as the previous one about Venus. Why are the following points not obvious?
(1) The surface must support the weight of the atmosphere, regardless of its temperature. Gases have weight! Thus, the surface pressure is determined by the mass of the atmosphere and the gravity of the planet, independent of temperature.
(1′) There is a very small effect of temperature, since it does affect the depth of the atmosphere, and gravity diminishes slightly with height. This is a small effect.
(2) An atmosphere should approximately follow an adiabat — that is, pressure and temperature varying at the adiabatic lapse rate — provided the conditions are suitable for convection. This is true in the lower atmosphere of both Earth and Venus. But convection will only occur if the fluid is heated from beneath. In other words, the net heat flow in the fluid in the convective zone must be upward.
(3) If the atmosphere of Venus were heated from the top — which I gather is Steve’s theory — then there should be no upward heat flow in the lower atmosphere. Ergo, no convection. The temperature profile would be much flatter than the adiabatic lapse rate. Indeed, in the long run it should be quite flat, an isothermal atmosphere.
(4) This is not what is happening. Thus, the surface is heating the atmosphere at the bottom. Internal energy sources, volcanism and such, are orders of magnitude too small. The surface must be heated by the short-wave radiation that reaches it. That is, it must be heated by sunlight (however attenuated). This net heat input to the surface accounts for the net upward heat flow in the atmosphere.
(5) The surface is really hot, and so it radiates strongly. (Most natural surfaces are pretty good blackbodies at IR wavelengths.) In fact, the surface radiates dozens of times more IR radiation than actually emerges into space at the top of the atmosphere. So almost all of this must be absorbed by the atmosphere, and reradiated (both up and down, of course, in each layer).
So we have an atmosphere heated from below, by a surface that is getting its energy input from short-wave radiation. The thermal radiation from the surface is strongly absorbed by the atmosphere. If we turned off this absorption and made the atmosphere transparent to IR, then the planet would quickly cool off to a much lower temperature, until radiative equilibrium would be re-established. Doesn’t that pretty much define the greenhouse effect?
I keep expecting this thread to be about WHY Venus has developed such a strong greenhouse effect, and whether the term “runaway greenhouse” appropriately describes this process. Fascinating question, and present theories involve a good deal of speculation. I would probably count as a moderate AGW skeptic — a “luke-warmer” in common parlance — but the discussion here does not inspire me with confidence that this community has a firm grasp on the basic physics. Sorry to be so blunt, but the thing is pretty distressing to someone who reads WUWT regularly and with pleasure.
I’d like to see Steve respond to the points about hydrostatic equilibrium and the upward heat flow associated with the adiabatic lapse rate.

lgl
May 9, 2010 2:33 pm

If the clouds at 40-60 km height absorbs most of the LW from below, and I believe they do, then the lapse rate should make the surface at 730 K with any composition of the atmosphere below the clouds.

Nullius in Verba
May 9, 2010 2:45 pm

Here’s another clear explanation of the role of the adiabatic lapse rate in ‘greenhouse’ warming.

May 9, 2010 3:04 pm

Nullius in Verba – that’s a fantastic link you’ve posted! Thanks!
Just one last contribution to the topic. I urge everyone to take a holistic look at Venus: it’s a very peculiar planet, and the composition of the atmosphere and surface temperature are just a couple of its peculiarities. Somebody has already mentioned the extensive (apparent?) volcanism, and how young the surface is. That’s not all: Venus rotates very slowly and in the wrong direction, as if something had hit and toppled it. Such a cataclysm alone would have resurfaced it several times over. Also where is plate tectonics? What is the origin of all strange surface features that don’t appear anywhere else in the Solar System? Etc etc
I know there’s people trying hard to build a rover that would resist the inferno and I am looking forward for such a mission to take place.

Rob R
May 9, 2010 3:28 pm

Steven
Over at “The Reference Frame” Lubos Motl has a detailed dicussion of your WUWT posts on Venus temperature.
You will be interested to find that in his conclusion he is largely in agreement with you. i.e. that the greenhouse effect on Venus amounts to a few dozens of degrees C and that the very high temp at the surface relates primarily to the environmental lapse rate.

May 9, 2010 3:38 pm

Ben Schumacher
The pressure is determined by the weight of the gas above it as you said. And according to the ideal gas law, the ratio of temperature/volume is controlled by the pressure. I’m not sure what you find baffling about that.
I’m not making any attempt to explain what the heating mechanism is. Just pointing out that the temperature profile in Venus’ atmosphere indicates it is an adiabat.
The purpose of this exercise is to argue that even if earth’s atmosphere was 100% CO2, temperatures would be nothing like they are on Venus.

kadaka (KD Knoebel)
May 9, 2010 3:41 pm

It’s been confusing on both articles how people are talking about pressure.
Pressure is force exerted on a surface, can be expressed as pounds per square inch or even newtons per square meter. Atmospheric pressure is being discussed, which basically is the average of the forces involved from gas molecules hitting a surface.
So if the temperature drops to where all the gas molecules precipitate out, what atmospheric pressure is acting on a vertical surface? None. What if I set up a pressure gauge with a horizontal surface pointing upwards, a meter above the ground, what would it read? Nothing, as all the formerly gaseous molecules are already in a layer on the ground below the gauge. Same thing if the horizontal surface is pointing downwards, rotated to vertical, or in any position.
Yes, you can talk of pressure from a substance laying on a surface that’s not completely vertical. But that would be pressure based on gravitational force, while here atmospheric pressure (all-surrounding) is being discussed. So why all the nit-picking?

Bob_FJ
May 9, 2010 3:41 pm

Jbar Reur May 9, 2010 at 6:27 am

[1] Most of the action in Venus’s atmosphere takes place in and above the cloud layer (50-70km altitude). Here clouds absorb sunlight on the sun side and race around at high speed to emit infrared radiation on the night side…
[2] Venus’s atmosphere is highly refractive, so that sunlight wraps around the planet from the sun side all the way to the dark side, much much more than the dawn/dusk effect on Earth (although I have no idea how much sunlight actually reaches the dark side). That provides some solar heating of the surface even on the dark side. However, I am still at a loss to explain why the surface temperature is uniform all the way around unless the high refractivity of the atmosphere makes the surface lighting uniform. (Doubtful)

[1] But the downward infrared cannot heat warmer air below, per thermo law 2. Also, the temperature gradient clearly shows heating from the surface, the candidates being absorbed sunlight and geothermal.
[2] With sulphuric acid etc clouds, haze, and a dense atmosphere, scattering must be greater than on Earth, but some of the Venera photos show pronounced shadows around the rocks. Thus the effect that you propose is most unlikely to be large, assuming the photos are gen.

GeoFlynx
May 9, 2010 3:42 pm

Didn’t mean to be condecending, but using the closed volume ideal gas laws to describe atmospheric pressure, equating temperature with atmospheric pressure, and implying that gravity disappears at the lowest temperatures are your ascertions. When lucid comments are made to correct these misconceptions there is no acknowlegement nor attempt at retraction. It would therefore appear that your intent is somewhat other than achieving an understanding of the question at hand.

May 9, 2010 3:58 pm

GeoFlynx
The fact that you don’t understand something is an indication only of your own lack of understanding. You are making ridiculous straw man arguments about gravity disappearing.
CO2 freezes long before reaching absolute zero. If you turned the atmosphere of Venus into a huge chuck of dry ice at 1K, it would continue to exert the same amount of pressure downwards on the surface of Venus. At the same time the vapour pressure of CO2 would be very close to zero.

May 9, 2010 4:26 pm

If you changed the composition of Venus atmosphere to 100% nitrogen, the planet would be much colder due to the lack of absorption of IR. The atmosphere would also necessarily be much thinner (lower volume.) P = nRT/V
However, if you changed the composition of Venus atmosphere to 95% nitrogen and 5% CO2, the temperature and height of the atmosphere would not be hugely different than it is at present. Why? Because the greenhouse effect is logarithmic. After the first few percent, additional CO2 makes much less difference to the temperature.
Consider earth, where a doubling of CO2 only increases temperatures by <1 (Lindzen) to 3 (IPCC) degrees C.

May 9, 2010 4:32 pm

GeoFlynx
The pyramids of Egypt exert a lot of pressure on the ground below. If you were buried 3,000 years ago underneath the pyramid you would be crushed. However, if you were buried in a tomb inside the pyramid, your remains might survive long enough to be made fun of by Steve Martin.
Gravity has not disappeared for King Tut. There are different manifestations of pressure and stress, and your simple view of the world is inadequate for complete analysis.
Don’t mean to be condescending (note spelling.)

May 9, 2010 4:45 pm

Steven,
“If you changed the composition of Venus atmosphere to 100% nitrogen, the planet would be much colder due to the lack of absorption of IR.”
I actually agree with that last post, and I think it’s pretty much mainstream science. But how is that statement different from the Wiki:
“Without the greenhouse effect caused by the carbon dioxide in the atmosphere, the temperature at the surface of Venus would be quite similar to that on Earth.”
which in your last article you said was all wrong?

Keith Minto
May 9, 2010 5:08 pm

Ignoring composition and circulation, does it not mean, comparing height of atmosphere alone, that Earth could never have a ‘runaway’ warming?
pft May 9 12:46 said,

Air at 30,000 ft is at quite a low temperature, due to it’s low density and pressure, yet it’s heat content on a per molecule or mass basis is close to that at the surface. If it were adiabatically compressed to 1 atmosphere (ie, w/o losing heat), it’s temperature would be close to air at the surface. This points to the difference between heat and temperature.

This is similar to my thoughts in the last thread i.e. higher pressure temperature readings are higher because they are simply more efficient. This is the result of a higher molecular collision rate on the sensor, so I would suggest that some (not all) of the apparent higher temperature at the bottom of a column of air is due to a density effect…. that is, you reading the heat content of the air more efficiently.
This confuses the current rationale behind the adiabatic lapse rate slightly.

OkieSkeptic
May 9, 2010 5:17 pm

Steve:
“However, if you changed the composition of Venus atmosphere to 95% nitrogen and 5% CO2, the temperature and height of the atmosphere would not be hugely different than it is at present. Why? Because the greenhouse effect is logarithmic. After the first few percent, additional CO2 makes much less difference to the temperature.”
I have difficulty seeing this. I can understand the reason for the thick CO2 blanket since there are no water/carbonates to remove the volcanically generated CO2. However, this wouldn’t apply to nitrogen and there should still be less of it relative to Venusian gravity using earth as an example. Couldn’t the major source of heat trapping on Venus simply be because of the thick atmosphere and lack of water vapor, making vertical convective transfer to space difficult (unlike Earth)? It has already been mentioned that the ground level density of the CO2 may be in the supercritical or near liquid state, which would make vertical thermal transfer very difficult relative to that of a gas.
As covered in other comments regarding CO2 depth pressure and temperature, if one could enclose a vertical section of Venusian atmosphere in a sealed insulated tube (closed system), after a period of time the temperature would completely equalize throughout the volume through convective transfer even though the gravity pressure at the tube bottom would still be 93 atmospheres. If this insulated tube of CO2 where originally in space and dropped into Venus’ gravity field, there would be immediate heating at the ‘bottom’ from the pressure increase but this would be cause by mass and potential/kinetic energy transfer to that section. Again when this closed system is left alone in the constant gravity field, the temperature would eventually equalize through convective transfer throughout the gas (at a higher temperature because of the gravity potential energy added) even though the pressure would be higher at the ‘bottom’ of the tube.
If this were not the case, all one would have to do for free energy is to install a heat engine between the top and bottom of the tube since there would be a constant temperature difference even in the closed system.

May 9, 2010 5:22 pm

However, if you changed the composition of Venus atmosphere to 95% nitrogen and 5% CO2, the temperature and height of the atmosphere would not be hugely different than it is at present. Why? Because the greenhouse effect is logarithmic. After the first few percent, additional CO2 makes much less difference to the temperature.
http://en.wikipedia.org/wiki/Spectral_line#Spectral_line_broadening_and_shift

Jbar
May 9, 2010 5:26 pm

RealClimate [yeah, yeah, I know] provides some insight into why Venus’s surface temperature is so uniform even though there is very little surface wind to redistribute energy, and minimal light on the dark side.
The Venutian atmosphere is incredibly massive, roughly 40% of the weight of Earth’s oceans. Consequently it would take an incredibly long time for the entire atmosphere to heat up or cool down. (In contrast, much of Earth’s thin atmosphere easily heats and cools in a day.) So expecting the surface temperature of Venus to vary would be like expecting the temperature at the bottom of Earth’s ocean to vary. This massiveness means that the lower atmosphere of Venus is able to equilibrate in temperature all the way around the planet with very little convection (i.e. very low wind). A simple solution when you think about it.
Venus Express finds no significant “daily” variation in Venus’s atmospheric temperature below an altitude of 45km. That altitude is equivalent to a depth (or rather, pressure) of two Earth atmospheres. All the action takes place in the upper atmosphere. The high speed winds in the upper atmosphere transfer energy from the day side to the night side and by the time you get down to 45km altitude, the temperature there remains constant. (This is a few km below the lowest cloud layer.)

May 9, 2010 5:34 pm

Something for those inclined to do a bit of web search…months ago, I noticed how the NASA planetary atmospheric science pages sadly lacked any reference to the “greenhouse effect”. I wonder if that’s still the case?

Dave McK
May 9, 2010 5:48 pm

stevengoddard says:
May 9, 2010 at 12:36 pm
Venus has no oceans, so it has no limestone. No limestone means a thick atmosphere of CO2. A thick atmosphere means high temperatures.
Bless the clams for saving us from venereal discomfort, for while Venus was being repaved with lava, they were constructing the white cliffs of Dover and starting to crawl from the sea.
Did I get the physics of this right, finally, Steve? It’s all about the fizz in the beer?
I think Dr. Pangloss was right and it’s time to get back to the farm where they don’t believe the rooster actually makes the sun come up.

May 9, 2010 5:53 pm

Jbar
The Real Climate explanation makes perfect sense. It is yet another indication that the climate of Venus is controlled by the pressure of the atmosphere.
All arguments lead to the same conclusion – i.e. if the Earth’s atmosphere was 100% CO2, the Earth would not be hot like Venus.

May 9, 2010 5:54 pm

Jbar, another aspect of the “low wind” on Venus is that, because of density, it still transfers a huge amount of heat. 10 km/h at the surface has been mentioned, but the gas heat capacity is (very roughly) 90X greater than Earth, so its heat transfer capability is like a 900 km/hr blast here.

May 9, 2010 5:58 pm

Nick Stokes,
If there were no greenhouse gases or clouds (as Wikipedia implied) temperatures would be very cold.
But if Venus had the same relative composition atmosphere as Earth, it would still be very hot because the atmosphere on Venus is much more dense.

Xyrus
May 9, 2010 6:11 pm

First let’s clear up some very basics physics.
The ideal gas law is appropriate when talking about gases in confined volumes. When talking about atmospheric pressure, you use the barometric formula.
http://en.wikipedia.org/wiki/Barometric_formula
There are several constants used that are specific to the atmosphere you are trying to use it on.
Atmospheric pressure is dependent on mass and gravity, among other things, and is derived from the ideal gas law.
To wit, as you approach zero kelvin your atmospheric pressure approaches infinity, not zero. (NOTE: Of course, under that scenario you can’t use basic formulas anyway.)
But back to the main discussion. Mr. Goddard is proposing that pressure is somehow driving the temperature on Venus. However, this makes no sense. Titan is a solid counter-example. It has a dense N2 based atmosphere and yet it is still bone chilling cold. There is simply nothing there that can hold onto the meager amounts of heat it receives from the sun (or Saturn).
Venus is another matter entirely. You don’t need a complex experiment to verify that the atmosphere is preventing almost all heat from escaping. We have a good idea of the surface temperature. We also can figure out how much combined IR radiation is escaping the planet (both reflected and emitted through day and night side IR measurements). If what is radiating from the planet is much less than what it should be given the other constraints, then SOMETHING between the surface and the TOA is preventing heat from escaping.
If it were merely density of the gas and NOT composition, then we can verify this quite simply. Take a few different gases (GHG and non-GHG) and place them in IR transparent containers at various pressure levels. Fire an IR laser through each sample at each level and measure. If pressure is indeed the key player, then we should see similar results across all the gas samples. However, if composition is the key player, then we should see disparate results across the gas samples. GHG gasses at higher pressures would absorb more IR while non-GHG gasses would show little to no differences regardless of pressure level.
Fortunately, these have already been done many times over in establishing thermal properties for various gases. It should come as no surprise that atmospheric composition is what matters. Density matters as well. After all, if you don’t have much of an atmosphere you’re not going to be absorbing much of anything no matter the composition. But you can have an N2 atmosphere at 1000 atmospheres and it still won’t create a greenhouse effect. Saying otherwise is the equivalent of saying you can can make salt taste like sugar by adding more salt.
Can what happened to Venus happen to Earth? Not very likely. We’d have to try really hard to do so, and it would take a long time of trying hard (centuries). I’d like to think that we would be smart enough to notice palm trees growing in Antarctica and know that something might be amiss.
At any rate, composition AND density determine the IR profile of an atmosphere. Basic high school/college level physics that has been validated many thousands of times in labs across the planet.
~X~

May 9, 2010 6:14 pm

Title: Radiative Transfer Within the Mesospheres of Venus and Mars
Authors: Ramanthan, V. & Cess, R. D.
Journal: Astrophysical Journal, Vol. 188, pp. 407-416 (1974)
Bibliographic Code: 1974ApJ…188..407R
http://adsabs.harvard.edu/full/1974ApJ…188..407R
Title: The Thermal Balance of the Venus Atmosphere
Authors: Crisp, D. & Titov, Dmitri
Issue Date: 1995
http://trs-new.jpl.nasa.gov/dspace/bitstream/2014/18365/1/99-1842.pdf
Interference of spectral lines in thermal radiation from the lower atmosphere of Venus
T. S. Afanasenk and A. V. Rodin, 2006
http://www.springerlink.com/content/n6272554n127155u/

Abstract The absorption spectrum and thermal radiation fluxes in the lower atmosphere of Venus are calculated using the theory of molecular state interference in the strong collision approximation. Comparison is made with the absorption and radiative transfer calculations in terms of the statistical theory of collisional line broadening and based on an empirical form factor. The calculations show that the line broadening mechanism does not affect the thermal regime of the atmosphere at heights above 60 km, but affects significantly the behavior of the greenhouse effect below the cloud layer.

Venus as a more Earth-like planet
Håkan Svedhem, Dmitry V. Titov, Fredric W. Taylor & Olivier Witasse, 2007
http://www.nature.com/nature/journal/v450/n7170/full/nature06432.html
The structure of Venus’ middle atmosphere and ionosphere
M. Pätzold, B. Häusler, M. K. Bird, S. Tellmann, R. Mattei, S. W. Asmar, V. Dehant, W. Eidel, T. Imamura, R. A. Simpson & G. L. Tyler, 2007
http://www.nature.com/nature/journal/v450/n7170/full/nature06239.html

Jbar
May 9, 2010 6:16 pm

To Nick Stokes:
OK, let’s say the surface emits 12,000 W/m2. Yet Venus only radiates 300-400 W/m2 from the top of the atmosphere, and the net radiation from the surface must be on the order of 70 W/m2. (That is 16% of the radiation received by Earth’s surface, which is what my source says reaches the surface of Venus.)
Doesn’t this prove how dramatically large the greenhouse effect is on Venus??!! The surface radiates 12,000 W/m2, but 11,930 W/m2 gets reflected back down to it by the atmosphere!!! That is a SUPER-GREENHOUSE.

Leonard Weinstein
May 9, 2010 6:18 pm

Jbar,
The radiation between two surfaces (or a surface and a gas layer) at different temperatures will have both an outgoing radiation from the surface and an incoming absorbed radiation from the gas layer. The NET flux is the difference, but the fact of the outgoing radiation is valid. The surface of Venus does radiate about 12,000 W/msq (assuming near unity emissivity), but absorbed back radiation is also very close to the same level due to the absorbing greenhouse gasses back radiating. The process is actually much more complicated, both due to the specific active wavelengths in the gas and due to it being an extended thickness effect. The emissivity and absorptivity of the ground may not be real close to 1 but probably is not too far off at the ground temperature and long wave radiation wave lengths. I think your disagreement is more one of semantics, not truth vs false.

Jbar
May 9, 2010 6:25 pm

Marc77:
Another thing about supercritical fluids – the supercritical point is the point at which the transition between liquid and gas disappears. There is no longer liquid and gas – there is only this supercritical fluid thing.
Something weird must go on at the altitude in Venus’s atmosphere where the pressure makes the CO2 supercritical.

May 9, 2010 6:31 pm

I appear to have mis-linked a citation in the above post:
Deconflicting the conflation …
Tectonic Effects of Climate Change on Venus
F. S. Anderson and S. E. Smrekar, 1999
http://trs-new.jpl.nasa.gov/dspace/bitstream/2014/18365/1/99-1842.pdf
Title:The Thermal Balance of the Venus Atmosphere
Authors: Crisp, D. & Titov, Dmitri
Issue Date: 1995
http://trs-new.jpl.nasa.gov/dspace/bitstream/2014/32268/1/95-1621.pdf
Unfortunately, this scan is unreadable.

Jbar
May 9, 2010 6:36 pm

Bjorn & others: You haven’t heard the story of the “runaway greenhouse”?
Supposedly a long time ago Venus may have been like Earth, but being so close to the sun the oceans got hotter until the ocean started to boil. Water is a good greenhouse gas, so once the water started to boil, it made the greenhouse effect stronger, boiling more water, which made the GH effect stronger still, etc. and the greenhouse “ran away” as the oceans completely boiled away.
After that, there might have been 100 atm of water vapor in the atmosphere for some time. With water vapor reaching all the way to the top of the atmosphere, solar radiation could knock the hydrogens loose from water. Venus’s gravity is strong enough to hold onto oxygen but not hydrogen, so the hydrogen gets kicked out into space. Eventually almost all the hydrogen from the water got stripped away from Venus.
Meanwhile all the free oxygen (created by the stripping away of hydrogen from water) reacted with anything it could near the hot surface, especially carbon. Eventually this turned all the carbon on Venus into CO2. 90 atmospheres of CO2 makes a strong enough greenhouse to heat the surface to 470deg C.
Some evidence – what little water vapor there is left in the upper atmosphere is rich in deuterium. Deuterium is twice as heavy as hydrogen and so it’s harder for solar radiation to knock into space, so it gets more concentrated as the hydrogen goes away faster.

Jbar
May 9, 2010 6:43 pm

OkieSkeptic:
Venus has about 3% nitrogen, equivalent to roughly 3 Earth atmospheres of partial pressure. Earth in contrast has 0.8 Earth atmospheres partial pressure of nitrogen.

Keith Minto
May 9, 2010 6:43 pm

Okie Skeptic May 9 2010 5:17.
Again when this closed system is left alone in the constant gravity field, the temperature would eventually equalize through convective transfer throughout the gas (at a higher temperature because of the gravity potential energy added) even though the pressure would be higher at the ‘bottom’ of the tube.
If this were not the case, all one would have to do for free energy is to install a heat engine between the top and bottom of the tube since there would be a constant temperature difference even in the closed system.

More like radiative transfer as you are assuming(I think) no convection in your column.
You are right on the last point, a point I feel is the Achilles Heel of Steves conjecture. If Steve is correct, then he has invented the Venusian equivalent of perpetual motion.

dr.bill
May 9, 2010 6:44 pm

stevengoddard: May 9, 2010 at 12:32 pm
dr.bill
How is Venus “warmed from the bottom” during it’s thousands of hours long nights?

Hi Again Steve,
As I said a little earlier, I don’t know the answer myself, but I would give serious thought to the suggestion made by:
Jbar: May 9, 2010 at 5:26 pm
It makes more sense than anything I have previously seen.
/dr.bill

May 9, 2010 7:14 pm

Xyrus
You wrote ” as you approach zero kelvin your atmospheric pressure approaches infinity, not zero. “
PV = nRT
Pressure is proportional to temperature, not inversely proportional as you claim. Now start over.

May 9, 2010 7:17 pm

Xyrus
If you think pressure is inversely proportional to temperature, try sticking an aerosol can in the oven. Actually, don’t try that. You will probably get hurt.

May 9, 2010 7:19 pm

Nick Stokes
Your heat transfer analogy doesn’t work, because the amount of required work also increases by 90X.
The key issue is wind velocity, because that limits how far the differentially heated air can move in a given amount of time.

Phil C
May 9, 2010 7:31 pm

MaxL-
The gas law applies to gases. As the temperature drops you get a change in phase and the law no longer applies. If Venus were moved out beyond Pluto, where there would be minimal heat from the sun or any other source, the temperature would start to drop. The volume of the atmosphere would decrease. When the temperature reached ~200 degK the CO2 would change to a liquid or solid, depending on the pressure, and eventually all turn into solid CO2 as the temperature fell. The other trace gases would condense out at their characteristic temperatures too, eventually turning almost all the atmosphere into solid.
The ESA work shows that Venus has a very strange atmosphere and climate compared to earth, primarily due to the lack of water. Venus lies just sunward of the habitable zone and apparently lost all of its water, probably even as it formed.

May 9, 2010 7:42 pm

Bob_FJ and Steve, if pressure could create heat it would mean gravity is not a conservative force. Energy could be extracted from gravity and converted to heat via the “Goddard PV = nRT” relation.
If Steve was right you would have to take an oxygen tank with you to the Arctic because low temp would mean low pressure. Low pressure systems would have lower temperature than high ones. Etc.
If it’s not greenhouse I think the answer lies more along the lines of: 1) There’s a lot more internal planet heat than given credit for. 2) There’s heat generated somehow from the solar wind and Venus gets twice as much of that as Earth.

Editor
May 9, 2010 7:45 pm

Xyrus says:
May 9, 2010 at 6:11 pm

Atmospheric pressure is dependent on mass and gravity, among other things, and is derived from the ideal gas law.
To wit, as you approach zero kelvin your atmospheric pressure approaches infinity, not zero.

Only if the gov’t is trying to bail out the atmosphere (it’s too big to fail!) by adding more air. Otherwise, the pressure at ground level doesn’t change, as the weight of the air above doesn’t change. The height of the atmosphere shrinks until it turns to liquid or ice, then it still shrinks, but a lot less quickly.

May 9, 2010 7:49 pm

Jbar says: May 9, 2010 at 6:16 pm To Nick Stokes:
Did you mean to address that to me? What you say has been my argument from way back.

May 9, 2010 7:51 pm

Paul Clark
Try again. Low temperature means low volume in Antarctica. The atmosphere is thinner at the poles.
PV = nRT
I really suggest that everyone pass a high school chemistry class before trying to tackle this problem.

May 9, 2010 7:55 pm

Jbar
It is extremely unlikely (impossible) that Venus was ever like Earth, because there are no limestones on Venus. The atmosphere must have always been very thick and hot.

May 9, 2010 8:04 pm

Steve Goddard
“If there were no greenhouse gases or clouds (as Wikipedia implied) temperatures would be very cold.”

Well, what they said was that they would be Earth-like. Maybe that’s high-ball. But the estimate you put up against the Wiki statement was 400C. And you said, with emphasis:
“The high temperatures there can be almost completely explained by atmospheric pressure – not composition.”
And here
“Your heat transfer analogy doesn’t work”
I wasn’t making an analogy. I was just pointing out that at 90 atm, even a 10 km/h wind can transfer a lot of heat. Heat transfer goes as mass flux, not velocity.

May 9, 2010 8:07 pm

The ideal gas law and Avagadro’s Law have been around a lot longer than I have.
The ideal gas law has four degrees of freedom (pressure, volume, number of molecules, temperature.) You can’t infer what one variable will do in response to a change in another, without also knowing what the other two are doing. For the sake of these discussions we can assume that n is constant. So the three degrees of freedom are P, V and T which are related by P = T*c/V where c is a constant.
If we assume in a given atmosphere that vapour pressure P is fixed through a reasonable range of temperature – then as T increases V also has to increase, and as T decreases V also has to decrease.

Bob_FJ
May 9, 2010 8:27 pm

ALL: Concerning the assertions above of high emissions from the surface, per Stefan-Boltzmann, I suggest that the calculation is incorrect, as discussed below:
a} Imagine a block of dry ice. It will emit a low level of EMR as a consequence of its T. Now let surface melting occur; what happens? Does it emit from the newly constituted solid surface, straight through the liquid, or does the thin liquid become a new effective surface, or what?
(Incidentally, if the wet surface behaves like a body, and the solid beneath continues to emit in the same way, then a wet body would emit more than a dry one…. Without considering evaporative cooling, if any).
b} If a dry body is immersed in say an Earthly thin gas, having a lower temperature, there will not only be EMR emission resulting in net HEAT loss, but also HEAT loss via conduction and convection. Where does this “extra energy” come from without upsetting the balance of the EMR? Or, to put it another way, the surface molecules are busy losing energy as EMR, (photons), and as HEAT, (KE). Can they do that without affecting each other?
c} Perhaps b} can be ignored in a thin gas, but what if the gas is at high pressure, and approaching a liquid state? (either absorbent or transparent)
d} What if we substitute a liquid in c}? We are back to a}, except the liquid is thicker/deeper.
As an engineer, I’ve puzzled over this stuff for many years but have not found it described anywhere. (with modest looking, admittedly).
Here is one typical assertion from above:
“…it is still true that the surface emits (about) 12,000 W/m2. The S-B law applies at that level. Of course, many wavelengths are then quickly absorbed in the real atmosphere. But they were emitted.”
I suggest that this is incorrect. Also, I believe that conductive/ convective/ advective surface heat loss must be very high on Venus for Steve‘s hypothesis to work. Can anyone show that these suggestions are wrong? (without simply appealing to authority)

May 9, 2010 8:53 pm

Nick Stokes
If you have 90X as much thermal flux and 90X as much mass, they cancel each other out. Wind can move heat no faster than it’s velocity.
As far as atmospheric composition goes, it makes little difference on Venus if it has 5% CO2 or 95% CO2. It is only the first few tenths of a percent that are really important to the greenhouse effect (relative to the importance of pressure.).

May 9, 2010 8:55 pm

Steve Goddard
“The atmosphere is thinner at the poles.”
This is getting surreal. The air pressure at the N pole surface is basically the same as elsewhere, and the density is higher (colder air). The S Pole has lower pressure, but nothing to do with the ideal gas law – just altitude.
You’re getting tied in knots by talking about volumes in a continuum situation. You’d do better to use the ideal gas law in the form:
P=(R/M) ρT
where ρ = density and M=molecular weight. (R/M) is constant.

Keith Minto
May 9, 2010 8:57 pm

stevengoddard says:
May 9, 2010 at 7:51 pm
Paul Clark
Try again. Low temperature means low volume in Antarctica. The atmosphere is thinner at the poles.

Wouldn’t be because of the rotation of the earth would it?

May 9, 2010 8:58 pm

I’m surprised that people are still arguing about this. You can see very clearly that temperature increases steadily downwards through the Venusian atmosphere.
http://www.astro.wisc.edu/~townsend/resource/teaching/diploma/venus-t.gif
If the atmosphere were thinner (like earth) the temperature would be much lower. Does anyone actually doubt this? This isn’t theory. It is observational data.

May 9, 2010 9:02 pm

Nick Stokes,
I said “can be almost completely explained by atmospheric pressure – not composition.”
Given the presence of a minimal amount of greenhouse gas, the pressure becomes dominant. The whole point of these articles is to demonstrate that Earth could not become like Venus, unless all the limestones dissociated. That would require a catastrophic event which would kill all life on earth anyway.

May 9, 2010 9:15 pm

Nick Stokes
Again PV = nRT
The North Pole has equal pressure (as you said.) n is fixed and the temperature is lower, so the volume also has to be lower. Lower volume means with fixed n means higher density (as you said.)
What is confusing to you about this? The ideal gas law works just fine, like it always has.

May 9, 2010 9:19 pm

Nick Stokes
Perhaps you are being confused by my use of the word “thinner?” I am using “thinner” to mean exactly that – the height of the atmosphere is smaller at the poles, because the atmosphere is more dense at the lower temperatures.

May 9, 2010 9:24 pm

The whole point of these articles is to demonstrate that Earth could not become like Venus, unless all the limestones dissociated.
Ummmm? Whats Up With That?
I think you have taken a fairly common statement “the atmosphere of both Earth and Venus demonstrate Greenhouse heating” and constructed a straw man “AGW will cause a runaway Venusian Greenhouse.” There may be some idjits bouncing around claiming that – but it ain’t mainstream AGW. Step away from the straw man, Steve.

May 9, 2010 9:50 pm

Steve Goddard:

“You can see very clearly that temperature increases steadily downwards through the Venusian atmosphere.”

Yes, and there’s a fundamental problem this observation presents to conventional physics. There’s no reason that the lower layers shouldn’t reach thermal equilibrium with the upper layers and warm them to the same temperature regardless of this pressure difference.
Take geothermal heat. About 200km down all rock is molten at at least 1000C. That heat can not ultimately go inward; it must all come outward. There should be an inverse square relation to the heat dissipation. The surface only being a fraction of an Earth radius higher than 200km down should be:
Earth radius = 6371km.
1/(Earth radius)^2 divided by 1/(Earth radius – 200km)^2 times 1000C = 938C.
The Earth’s surface should be 938C. It appears heat is disappearing. Meanwhile, the ocean below the thermocline is refrigerated at 2 to 4C when it should be boiling from the geothermal heat. The mid/low ocean can’t be cooling from evaporation on the surface because the upper ocean layer is warmer than the mid to lower levels. Heat can’t be extracted from a colder place through a hotter place. It appears to be a case where energy is either created or destroyed.
My own belief is that cooling might be caused by a thermoelectric effect related to the vertical voltage gradient.
Steve, there is apparently some sort of relation between temp and pressure in the air but it’s not one of cause and effect. We just don’t know what it is yet.

May 9, 2010 10:01 pm

Steve Goddard
Well, Paul Clark said:
“If Steve was right you would have to take an oxygen tank with you to the Arctic because low temp would mean low pressure. Low pressure systems would have lower temperature than high ones.”
to which you responded:
“Try again. Low temperature means low volume in Antarctica. The atmosphere is thinner at the poles.
PV = nRT
I really suggest that everyone pass a high school chemistry class before trying to tackle this problem.”

Nothing Paul said related to the height of the atmosphere.

May 9, 2010 10:08 pm

Ron Broberg
The idea that Earth could become like Venus was a central point of Sagan’s TV show – Cosmos. A whole generation was corrupted by this idea.

Carrick
May 9, 2010 10:10 pm

Nick Stokes:

This is getting surreal. The air pressure at the N pole surface is basically the same as elsewhere, and the density is higher (colder air). The S Pole has lower pressure, but nothing to do with the ideal gas law – just altitude.

“Thinner” I believe refers here to height not density. See this.
The reasons for this are a lot more complex that just e.g. the ideal gas law, though. (Ref.

May 9, 2010 10:12 pm

Straw Man? LMAO – This is just one of thousands of similar articles:
http://www.is.wayne.edu/mnissani/a&s/GREENHOU.htm

If the process continues unchecked, it could reach a point of no return, as it apparently had on Venus. As the Earth heats up, water would convert into steam, which is a greenhouse gas. The Earth would get hotter and hotter. As the atmosphere heats up, more CO2 might escape from its present location in ocean rocks and shells. Beyond a certain point, the process may be self-sustaining. Venus tells us how far such a process can go—hellish temperatures, a cloud cover that lets less sunlight reach the earth but helps trap the heat, enormous pressures at ground levels weighing heavily upon everything and distorting the landscape. Venus provides us a timely warning. If we pay no attention, in some 800 years life on Earth might perish.

May 9, 2010 10:39 pm

Just to clarify my last comment: there is a PV = nRT relation for temps and pressure for changing situations in such as low pressure systems, Hadley cells, etc. But, it does not apply to the correlation between temp and pressure vertically as found on Earth and Venus.

Keith Minto
May 9, 2010 11:46 pm

stevengoddard says:
May 9, 2010 at 9:19 pm
Nick Stokes
Perhaps you are being confused by my use of the word “thinner?” I am using “thinner” to mean exactly that – the height of the atmosphere is smaller at the poles, because the atmosphere is more dense at the lower temperatures.

And because of the rotation of the earth/atmosphere system, troposphere goes to 7km at the poles and to 17km at the equator.

Spector
May 9, 2010 11:57 pm

In general, I believe that an adiabatic lapse rate in a planetary atmosphere is evidence of an active convection process. Without convection, such as in the stratosphere, higher parcels of air may be warmer than those below. Perhaps there must always be a greenhouse effect to force the tropopause above the ground level.
If Venus had an atmosphere as thick as it is now, but with a composition similar to the Earth, I suspect an active convection system would also exist and produce a lapse rate similar to that of the current Venusian atmosphere. Perhaps this is why Hungarian scientist Ferenc Miskolzi has said the magnitude of the greenhouse effect is limited by the thickness of the atmosphere.

Ralph
May 9, 2010 11:59 pm

>>>Ben Schumacher says: May 9, 2010 at 2:06 pm
>>>An atmosphere should approximately follow an adiabat — that is,
>>>pressure and temperature varying at the adiabatic lapse rate —
>>>provided the conditions are suitable for convection.
You don’t need convection for an adiabatic lapse rate – it is purely gas density/pressure that makes the gas hot. I have adiabatic warming in my bicycle pump, but no convection.
http://en.wikipedia.org/wiki/Lapse_rate
But adiabatic temperatures can reduce, as the heat is conducted or radiated away, as in my bicycle pump getting cooler – and the result on Venus would be a cooler but denser atmosphere (but roughly the same pressure, I presume, because of the same weight of gas above the surface).
So the current high temperatures on Venus must be augmented by solar input (or geothermic processes) to keep this high temperature. I would suggest that the atmosphere is thick enough and opaque enough to absorb a large amount of solar radiation.
.
.

Ralph
May 10, 2010 12:08 am

>>>Paul Clark says: May 9, 2010 at 9:50 pm
>>> There’s no reason that the lower layers shouldn’t reach thermal
>>>equilibrium with the upper layers and warm them to the same
>>>temperature regardless of this pressure difference.
They may well do, in terms of molecular velocities (temperature), but the lower atmosphere has many more molecules/cm2 – so that same ‘temperature’ per molecule equals much more heat. That’s the whole point about compressing gasses and an adiabatic lapse rate.
But again that higher pressure does not preclude the lower atmosphere radiating and convecting away its heat, and becoming denser and cooler.
A HIGH PRESSURE GAS DOES NOT HAVE TO BE WARM. Touch the sides of a diver’s gas cylinder, and tell me if it is warm. Thus Venus must have a thermal input, to maintain its high temperature – otherwise it would end up like the diver’s very dense, very high pressure, but very cold gas cylinder.
.

Ralph
May 10, 2010 12:16 am

Just one question about the Venera pics – why did they never ‘normalise’ the colours? They have a colour chart there for comparison, but I have never seen an image of the surface as it would look under Earth-like light conditions.
.

Ian H
May 10, 2010 12:17 am

@Paul Clarke and others: The Pressure-Temperature link exists only in gasses that are vertically mixed. Water (being essentially incompressible) doesn’t do this even with vertical mixing. The earth’s crust doesn’t do this either. Not only is it almost incompressible, but it is also essentially static. So comments about a relationship between pressure and the temperatures of the sea and earth’s crust are way off topic.
A column of gas without vertical mixing would also be all the same temperature. Wherever there is some amount of vertical mixing however, the high pressure regions low down will be heated by this, and the upper low pressure regions will be cooled. A temperature gradient will then form.
Overall temperatures will be set by the part of the atmosphere which is in radiative equilibrium with space. On earth most radiation reaches the ground, and the surface itself is directly heated. Therefore the temperature at the surface mostly sets the overall temperature of the atmosphere. Since the rest of the atmosphere is at much lower pressure it will be much cooler than the radiative equilibrium temperature. On venus it is not the surface but the the upper cloud layers that are directly heated by the sun. The upper clouds therefore reach the temperature of radiative equilibrium with space, and the surface, which is at much higher pressure will end up being much hotter.
All this is approximate and ignores other effects, including the greenhouse effect which deals with radiative transfer within the atmosphere. But to first order this serves as an adequate explanation of why Venus is so much hotter than the Earth.

Ralph
May 10, 2010 12:29 am

Steve.
If you are implying from this post that Venus should have a much higher laps rate, due to its 99% CO2 atmosphere being a so-called ‘greenhouse gas’, that is not immediately obvious in your post.
You need a graph of lapse rate with 0.1% CO2 (Earth) and a lapse rate with 99% CO2 (Venus) being much steeper (because the latter should, according to green logic, capture more solar energy, which is distributed through the Venusian atmosphere by convection).
But this is something that Venus does not display, with both Earth and Venus having the same lapse rate.
Ergo – CO2 is not the big driver of temperature on Venus, and nor on Earth.
.

dr.bill
May 10, 2010 1:46 am

OK, I’ll try this once more.
When using the Ideal Gas Law, or any other equation of state for a gas, it is important to consider ALL of the quantities involved. It is also important to consider the circumstances. A sealed rigid volume such as a SCUBA tank isn’t the same as an inflated rubber balloon, and neither of those is the same as an open column of air “standing up” in a gravitational field. Details matter, and people have been conflating many issues incorrectly, or forgetting certain constraints and complications.
The Gas Law can be written as P = (N/V)kT, and the quantity N/V is called the particle density. There is no fixed volume for our atmosphere. Its volume can increase or decrease as it expands and contracts. In this context, it is the product of particle density and temperature that must remain fixed if the pressure is not to change.
This happens all the time, all around the planet. The surface temperature changes RADICALLY with time and location. Nevertheless, the pressure at sea level NEVER changes in any SIGNIFICANT way from 1000mb, no matter where you are on Earth, or what time of the day or time of the year it happens to be. Yes, it does change, and that is why we use barometers, but the AMOUNT of change is MINISCULE in comparison with the corresponding changes in temperature. Sea level pressure in the Arctic is essentially the same as sea level pressure in the Caribbean, despite the large differences in temperatures.
When you start climbing above sea level, however, you can’t just use the Gas Law by itself. You have to consider the effects of gravity. One of the consequences is the fact that pressure and particle density do not change in the same way as we go to higher altitudes. Both temperature and particle density decrease as we move upward in the troposphere, and because pressure depends on the product of these things, the pressure decreases more quickly than either temperature or particle density. At the tropopause, the particle density is typically down to about 30% of its sea level value, but the air pressure has gone down faster, and will only be about 20% of its sea level value.
The devil is in the details, and many people on this thread, and particularly Steve, are ignoring them. Actually, I think the tide has reversed on this. Many other posters appear to “get it”, but Steve doesn’t seem to be doing so. Another case of “right answer, wrong logic”, I guess.
/dr.bill

Gail Combs
May 10, 2010 3:28 am

Jacob says:
May 8, 2010 at 4:14 pm
Why is the atmosphere on Venus 92 times more dense than on Earth? That is the big question, that I don’t understand.
Now, if the atmosphere is denser – it means there are 92 times more molecules of gas in the same volume (or layer) of the atmosphere, therefore they would absorb 92 time more outgoing radiation (much more anyway). So, the heat on Venus’ surface is caused by the greenhouse effect after all, by the fact that the greenhouse effect is much stronger there, due to the density of the atmosphere, and not necessarily due to it’s composition (Co2).
If pressure alone could cause heating – why isn’t the bottom of the oceans very hot?
_________________________________________________________________________
J.Hansford says:
You are confusing gasses and liquids Jacob… Seems a few others are doing the same. Gasses are compressible, liquids are not…..
_________________________________________________________________________
PV=nRT is the ideal GAS law. How it applies to Venus and Earth is the topic under discussion.
“A law relating the pressure, temperature, and volume of an ideal gas. Many common gases exhibit behavior very close to that of an ideal gas at ambient temperature and pressure. The ideal gas law was originally derived from the experimentally measured Charles’ law and Boyle’s law. Let P be the pressure of a gas, V the volume it occupies, and T its temperature (which must be in absolute temperature units, i.e., in Kelvin). Then the ideal gas law states
PV=nRT
where n is the number of moles of gas present and R is the universal gas constant,….”

Source: http://scienceworld.wolfram.com/physics/IdealGasLaw.html

May 10, 2010 5:35 am

dr.bill
Remarkable that I have to explain this again. Please read my posts carefully before posting.
At temperatures where the atmosphere is a gas, the pressure is fixed by the weight of the column of air. The temperature controls the volume (i.e height of the atmosphere.) That is why the cold poles have a thinner atmosphere than the tropics.
P = T *c/V
This is the most fundamental physics. When the temperature decreases, the volume has to decrease. The volume is area * height. Area is fixed, so the only thing which can change at the cold poles is the height of the atmosphere.

DavidB
May 10, 2010 6:10 am

Consider a cylinder containing a quantity of gas. The cylinder is closed at one end and has a piston at the other. Suppose now we force in the piston, compressing the gas. What happens to the temperature of the gas?
Applying the ideal gas law, using the equation pV = nRT, the volume V is reduced by the compression, and nR is unchanged, so, other things being equal, T must be reduced. Thus we reach the remarkable conclusion that compressing a gas reduces its temperature!
Of course this is quite fallacious. In reality, compressing the gas increases its temperature because the kinetic energy of the moving piston is transferred to the molecules of the gas. To balance the equation, the pressure p is also increased. If the cylinder is perfectly insulated the increased T will be maintained indefinitely, so long as the piston is held in position. In practice, perfect insulation is impossible, and T will fall by conduction and radiation until it returns to equilibrium with the environment. As T falls, assuming the piston is held in position, pressure will also fall, though not back to its original level before the gas was compressed.
In this example the fallacy in the original argument was obvious, because we knew the conclusion was false, and it was easy to see the loophole in ‘other things being equal’. But in other cases the fallacies may not be obvious. The moral is that the equation of the ideal gas law cannot be applied blindly without considering the physical processes involved.

the fritz
May 10, 2010 6:34 am

experiment #5 Imagine the venusian atmosphere would be still heavier and no sunlight would reach the surface; the temperature would still be greater and would you still speak about greenhouse effect to explain the great temperature at the venusian surface?

Dave
May 10, 2010 7:02 am

“Why? Because the greenhouse effect is logarithmic. After the first few percent, additional CO2 makes much less difference to the temperature.
Consider earth, where a doubling of CO2 only increases temperatures by <1 (Lindzen) to 3 (IPCC) degrees C."
Again, NOT TRUE. The forcing from CO2 is logarithmic only within certain a certain range about the current concentration. At very high concentrations it approaches linearity again as the 15 um band saturates and the weak bands become predominant. This has been pointed out to you multiple times, and yet you keep repeating it.
If you want a reference, see Figure 1 in:
http://journals.ametsoc.org/doi/pdf/10.1175/1520-0469%281977%29034%3C0448%3AARCMSO%3E2.0.CO%3B2
(I might also recommend http://www.agu.org/journals/ja/v085/iA13/JA085iA13p08223/, one of the earlier calculations of the full temperature profile of Venus' atmosphere)

May 10, 2010 7:05 am

DavidB
If you push a piston down into a cylinder, you are exerting a force which directly increases the pressure. That is why the temperature increases. Your claim that “everything else is held equal” is incorrect.

Mike M
May 10, 2010 7:24 am

Your insistence on relating pressure to molecular motion, and heating due to pressure, is ruining what should have been a good discussion. ….Regarding pressure, it can be caused by molecules in motion. It can also be caused by the simple weight of whatever is sitting on something else. …. This is basic Physics.

Exactly what I have been trying to get Steve to respond to. He refuses to acknowledge that because PV=nRT does not include any term for gravity, it does NOT mean that a gas is magically immune to the law of gravity. It is basic physics.
Steve, true or false, if there was no gravity there would be no atmospheric pressure?
You might escape earth’s gravity if you run fast enough but you won’t escape this question.
Another thing Steve, you dismissed my notion that water made all the difference between Venus and Earth per my reasoning. I did some further thinking that I hope you will at least consider and comment without going back in time to limestone.
I looked up the annual rainfall on earth and came up with the number ~5 x10^14 cubic meters. Every last drop of that rain represents an amount of latent heat energy to evaporate it and the huge majority of that heat went into liquid water
on the ground, (a small amount happened in the air, i.e. virga).
So I went further and astounded myself even doubting the numbers I came up with. Maybe I made a mistake or relied on wrong data or something but please follow it and you be the judge of that.
If you take that number and divide by seconds per year it comes to 16 million m^3/sec. At 1000 kg per m^3 water, that’s 16 billion kg of water evaporating on mostly the surface per second.
<a href="http://www.thermexcel.com/english/tables/vap_eau.htm"Evaporating one kg of water requires 2,257,920 joules of heat. That works out to 3.4×10^16 watts of evaporation – something Venus, without water, does NOT have going for it.
So I divided that number by earth’s surface and get about 67 watts/m^2.
If that is correct then it most definitely is NOT something to snort at!
NASA puts incoming solar irradiance at 340 watts/m^2 so that comes to 20% of earth’s ERB being manipulated by water enthalpy as I think another poster above correctly termed it – ‘a heat engine’.
Recognize that although the heat is collected at the surface it is NOT released at the surface … it is released at cloud formation altitude. The heat was carried up totally unaffected by very significant amount of ‘green house’ gases below cloud formation altitude. Pick the average altitude AGL – I don’t know. 5000 feet? 10000 feet?
As I see it, whatever the best average altitude value, it represents a chunk of atmosphere that ~20% of solar heating did NOT have to pass through on its journey re-radiating out to space because it was carried up there mechanically. (And remember, just because the water vapor cooled off as it rose to that altitude it did NOT lose any heat! “adiabatic”.)
So I stand on my assertion that having 2/3 of our planet covered in water is what drives our climate and the reason we are so much cooler than Venus. PV=nRT has nothing to do with it.

May 10, 2010 7:32 am

Dave
The weak bands are linear, but with a small slope. That is why they call them “weak bands.”

May 10, 2010 7:37 am

Mike M
As I have stated about 14 times now, the atmospheric pressure (in temperature ranges where it is a gas) is set by the weight of the atmosphere above it. P is fixed by the weight of the column of air above.
Venus has a large value of P because it has a large value of n. The compressibility of gases is not linear, and as a result, T is high on Venus.
PV = nRT

May 10, 2010 7:41 am

The main reason why earth is cooler than Venus is because we have oceans which remove CO2 (term n in the ideal gas law) from the atmosphere in the form of limestone. This causes the atmospheric pressure to be much lower and thus the temperature, as specified by the ideal gas law.
PV = nRT

GaryW
May 10, 2010 7:43 am

OK, let me see if I can state what Steve says in a way that makes sense to us engineer types: We have three main processes producing a higher surface temperature on Venus than Earth. The first, of course is solar energy influx greater than that of Earth. The second is a blanketing layer of thick clouds that has a very high albedo. That cloud layer not only reflects most solar energy away from the planet but also reflects most energy from below them downward. That is the same mechanism we experience here on Earth that causes clear, starry nights to be colder than cloudy nights. The third is vertical motion of the atmosphere produced by some as yet poorly defined energy source that causes rising gases to cool and falling gasses to warm. The environmental lapse rate is primarily the result of this bi-directional vertical motion within Venus’s thick atmosphere.
I don’t know if I agree with the above description. I’ll have to dust of my calculator and see if the numbers work out.

May 10, 2010 7:47 am

DavidB
Is the purpose of a bicycle pump to increase the pressure or the temperature? You drive the pump in order to increase the pressure, and a side effect is that the temperature increases too because of the non-linear compressibility of gases.

May 10, 2010 7:53 am

My 7:41 post was not explained properly wrt how n relates to limestone removing CO2.
The existence of limestone reduces the mass of the atmosphere, which reduces P, which in turn reduces T.

Colin
May 10, 2010 7:53 am

A reply to Ralph’s question dated May 10, 2010 at 12:16 am:
> Just one question about the Venera pics – why did they never ‘normalise’ the
> colours? They have a colour chart there for comparison, but I have never seen an
> image of the surface as it would look under Earth-like light conditions.
Have a look at http://www.solarviews.com/raw/venus/v13corr.jpg – the surface looks black like lava.
Further discussion about the color of light on Venus is available at Don Mitchell’s excellent site: http://www.mentallandscape.com/C_CatalogVenus.htm

Spector
May 10, 2010 8:21 am

RE: Ralph: (May 9, 2010 at 11:59 pm) “You don’t need convection for an adiabatic lapse rate – it is purely gas density/pressure that makes the gas hot. I have adiabatic warming in my bicycle pump, but no convection.”
Apples and Oranges — an adiabatic lapse rate and adiabatic warming are two different things. At ground level we often have stable inversion conditions where a layer of cold air at the surface is under relatively warm air above. Also the stratosphere is characterized as a zone where temperature increases with altitude. Adiabatic cooling can only happen if a relatively warm parcel of air rises to a region of lower pressure, where it expands and cools as a result. That is convective cooling in a nutshell. If there are no rising or falling parcels of air then there can be no adiabatic cooling or warming.
I believe convection stops at the tropopause because that is the approximate altitude where the atmosphere finally becomes so thin that the greenhouse effect is unsupported and air that has risen to that level is finally able to release or relay its cargo of heat by direct radiation to space. This would suggest that the convection process might act as a safety valve to prevent ground temperatures rising above the lapse-rate span to the tropopause.

Jbar
May 10, 2010 8:29 am

Nick Stokes, in re May 9, 7:49 PM
Yes I did mean to address you Nick. So thanks for setting me straight on that surface black body radiation thing (no, seriously). It’s the MechEs who get trained in that radiative heat transfer stuff (steam boilers and all that). Not my specialty.
Fascinating stuff, science. I could do this all day if someone PAID me for it.

May 10, 2010 8:39 am

GaryW
A much simpler empirical exercise would be to answer qualitatively this simple question.
What would happen to temperatures on Venus if you could strip away 99% of it’s atmosphere (making it like earth?)
Hint:
http://www.astro.wisc.edu/~townsend/resource/teaching/diploma/venus-t.gif

David L.
May 10, 2010 8:47 am

Thinking about the insulating property of the atmosphere as a function of thickness got me to thinking about insulating a house. Certainly the R-value goes up as the thickness of the fiberglass goes up. It also reminded me of the poor insulating properties of windows. Typically they put Argon in the higher grade double panes, but according to AGW theory wouldn’t you want to fill them with CO2? That way the windows could reradiate the heat back into the house. So if this doesn’t really work for double pane windows, should we expect a few ppm to work in the atmosphere?

May 10, 2010 8:50 am

Mike M
The ideal gas law is independent of gravity. That does not mean that any of the individual terms in the ideal gas law are independent of gravity. For instance, atmospheric pressure (P) is determined by the weight of the column of air above it.
P = cT/V where c is a constant
If P increases due to increased mass above you (i.e. you are descending through the atmosphere) then the ideal gas law tells us that the ratio of T/V must also increase. And because gas compressibility is not linear with pressure, we can then infer that T has to increase.

dr.bill
May 10, 2010 8:54 am

stevengoddard: May 10, 2010 at 5:35 am
dr.bill
Remarkable that I have to explain this again.
Please read my posts carefully before posting…..

I have read ALL of your posts, Steve, which is what prompted me to try and explain the basic Physics of the situation, which I am eminently qualified to do. You keep bouncing around like a pinball, reversing cause and effect, using equations in ways that aren’t valid, and making one loose assertion after another, and then changing them to other things that are even looser, as has been pointed out by many posters before me. This discussion is no longer even remotely scientific. It’s more like the stupidity my kids used to get up to in the back seat of the car. “Do too!. Do not! …..” Bah……
It is time for me, at least, to go spend my time doing something more productive.
/dr.bill

Jbar
May 10, 2010 8:56 am

Steve Goddard, in re May 9, 7:55Pm, “extremely unlikely that Venus was ever like Earth” “The atmosphere must have always been thick and hot”.
Nobody even knows what the primeval atmosphere of even Earth was like. There is no proxy measure for surface pressure. The latest theory I heard is that Earth originally had very little atmosphere after it accreted and that bombardment with comets is what provided the water, O2, N2, CH4, etc. In that case, Venus being closer to the sun would be even more dependent on comets as a source of gases. There is no way to know its atmospheric composition or pressure at the end of the Late Heavy Bombardment.
I’m just repeating the narrative of the “runaway GH theory” of Venus for those who haven’t heard it (apparently). “Once the hypothetical ocean started to boil, that increased the water vapor GH effect, leading to more boiling, more GH, more boiling, and the [so-called] runaway greenhouse”. Since nobody really knows what Venus’s primordial atmosphere was like, this is merely “an” hypothesis.
“Venus didn’t have limestone.” Initially Earth didn’t have limestone either, supposedly, but I have not heard any account from a single geologist or planetary scientist suggesting that Earth’s atmospheric pressure was every more than a few atmospheres, so what was the form of all the carbon that must have been here after the Late Heavy Bombardment if it wasn’t in the atmosphere?? Let’s just agree that nobody knows. (And by the way carbonate rock can also form from chemical weathering and precipitation, not just seashells.)

Jbar
May 10, 2010 9:21 am

Steve Goddard, in re may 9, 10:08PM
“The idea that Earth could become like Venus was a central point of Sagan’s TV show – Cosmos. A whole generation was corrupted by this idea.”
Sorry, Steve, Earth COULD become like Venus. It’s not a corruption. (Sagan had a TV show???) As the sun converts hydrogen into helium, it’s core becomes denser and hotter thanks to the changing kinetics and thermodynamics of the fusion reaction, and as a result the sun is gradually becoming hotter over the gigayears. It is estimated to have been 30% cooler at the time of Earth’s formation (which presents all kinds of questions as to why the Earth was not therefore an ice-ball then, but let’s not get into that).
Over time, the sun will continue to get hotter, at a slowly accelerating pace, and it is estimated that in about a billion years it will be hot enough to raise the ocean temperature enough to create the runaway greenhouse effect of Venutian notoriety thanks to the increasing water vapor concentration leading to increasingly higher GH effect, more boiling, more GH, until the oceans are boiled dry. This would create a surface pressure of almost 250 atmospheres, and stop plate tectonics dead in its tracks.
After that, it may take a very very long time, much longer than it “MAY HAVE” on Venus thanks to Earth’s protective magnetic field, for solar radiation to dissociate all the water vapor and strip away all the hydrogen from the atmosphere, but eventually it will happen. And all the free oxygen that is liberated by this will have to oxidize something. First all the (dead) biomass will turn to CO2 (adding still more to the weight and pressure of the atmosphere) long before all the water vapor has been stripped of hydrogen, and eventually maybe it will get hot enough to bake all the CO2 out of the carbonate rock, aka limestone. Plate tectonics having been stopped, anything that comes out of any remaining volcanos will stay in the atmosphere. We’ll probably even wind up with sulfuric acid clouds.
Fortunately, AGW is nowhere NEAR strong enough to make this happen.

kwik
May 10, 2010 9:35 am

Steven, again thank you for taking your time doing this post. It has really started the gray cells working.
I have read the post many times now, and its starting to sink in. Most people here in Norway remembers PV=NRT .
The problem I think is this lapse rate stuff in combination with the ideal gas law.
Its the notion that we all know that if you compress a cylinder with a piston, we know that temerature must go up.
Because PV=NRT.
But after this compression ….thats the point I got problems with your post.
At first.
Then its the lapse rate. I understand now what you are saying.
I think.
You say that hot molecules migrate upwards, and radiates heat to space.
And sinks.
So the sun (with gravity as helping hand) is acting almost like a continous piston….

May 10, 2010 9:38 am

Jbar
Regardless of the early history of either planet, it is unlikely that Venus was ever like current day earth because there is no limestone on Venus.

May 10, 2010 9:39 am

dr.bill
Your appeals to your own authority and non-specific complaints don’t impress me.

May 10, 2010 9:43 am

kwik
You are correct that there has to be a a continuous external heat source to keep the gradient from equalizing. I’m not attempting to explain cause in this article, just pointing out that there is a vertical temperature gradient through Venus atmosphere which corresponds with pressure.

May 10, 2010 9:46 am

Jbar
Yes, when the sun becomes a red giant the polar ice caps are going to melt, but that has nothing to do with what we are discussing.
The greenhouse effect can not turn earth into Venus because of all the CO2 trapped in limestone.

May 10, 2010 9:52 am

Count me as also unimpressed with dr. bill. He sounds like a perfumed academic coasting on tenure when he says he’s “eminently qualified.”
dr. bill doesn’t understand that science is done by batting around ideas until only the conclusions that remain standing are accepted as valid. That’s what we’re doing here. But dr. bill can’t stand the heat.
Anthony is pretty good about publishing articles. Since dr. bill is so eminently qualified, maybe he’d like to write up a rebuttal, and see what it’s like to be on the receiving end of the pot shots.

May 10, 2010 9:59 am

dr.bill
If you disagree with me, then please use your eminent qualifications to demonstrate that Venus high temperature is unrelated to it’s atmospheric pressure.

Jbar
May 10, 2010 10:12 am

David L.
The thickness of CO2 you can put between two window panes would not absorb enough IR to make any difference. There’s not enough mass. Besides, if I’m not mistaken glass reflects IR. [Which is supposedly why greenhouses get hot, but actually it’s because the hot air is physically sealed in by the glass and not allowed to convect with the rest of the atmosphere!!! So “greenhouse effect” is actually a gross misnomer!!! What happens in a greenhouse is COMPLETELY different from what happens in the atmosphere due to greenhouse gases!]
They put argon in windows “because it reduces convection currents”, but I suspect the effect is pretty small. Air Products and Air Liquide must be sitting on piles of argon because it’s left over from making liquid nitrogen and liquid oxygen (it’s 1% of our atmosphere).

May 10, 2010 10:16 am

Smokey
Good points. I have yet to see anyone argue against the central thesis that the key difference between the temperatures of Earth and Venus is the atmospheric pressure. Some people are getting bogged down in specific details that have way too many degrees of freedom to define in a simple model.

Jbar
May 10, 2010 10:21 am

Steve Goddard:
“all the Co2 is trapped in limestone”
So how do you know that when the water vapor/ biosphere oxidation greenhouse raises Earth’s surface temperature to 523.15C that it won’t cause massive amounts of buried limestone to rise to the decomposition temperature of 825C, thereby releasing CO2 into the rock and atmosphere??
Where are your computer models?

May 10, 2010 10:31 am

Jbar ,
The IPCC estimates that each doubling of CO2 raises earth’s temperature by 3C. That means a maximum possible increase of about 36C.

Spector
May 10, 2010 10:39 am

RE: stevengoddard: (May 10, 2010 at 8:50 am) “P = cT/V where c is a constant”
I believe the fallacy here with the use of the ideal gas law may be that it only applies in those cases where no net transfer of energy occurs. Once energy transfer is allowed then all bets are off.
Isothermal and Adiabatic Processes. A process that takes place at a constant temperature is called isothermal Process. For a gas expanding isothermally the general law becomes
PV=mRT=constant
which states that the pressure decreases as the volume increases (Boyle’s law). When the expansion or compression of a gas takes place without transfer of heat to or from the gas, the process is called adiabatic. An ideally adiabatic process would have to take place in a container whose walls were perfect thermal insulators. The practical cases in which the expansion or compression of the gas takes place so rapidly that there is negligible heat transfer may be treated as adiabatic processes.”
College Physics Weber, White, and Manning, pp289, 1952

May 10, 2010 10:50 am

Steven,
Great post, and thanks for answering comments. That really fleshed out the idea for me.
It appear fair to say changing the Earth’s temp to something like Venus’ would be overwhelmingly dependent on pressure and only slightly on composition. I also like very much the idea Venus never had a less dense atmosphere.

dr.bill
May 10, 2010 10:51 am

Smokey: May 10, 2010 at 9:52 am
Your sense of smell is quite impressive! Not so good re the “coasting”. I am also under no “heat” here. I am not the one misinterpreting the principles of Physics. Steve is. In posting his proposal, he was, in effect, asking for “heat” to be directed at him. If none had been warranted, I would not have provided any, but “heat” was not my intention – simply clarification. I would suggest that Steve is the one having a hard time with the “heat”.
stevengoddard: May 10, 2010 at 9:59 am
My “complaints” were very specific for the first posts that I made. I am also not appealing to “my own authority”. Everything I said is part of standard Thermodynamics, and can be readily verified by you or anyone else. You’re missing the your own point here in suggesting that I provide a “counter-proposal”. I was not the one who proposed the explanation for Venus. You did. I do not know enough about Venus to try and explain what might be going on there. I doubt that anyone else does either in any quantitative way at the present time. I do, however, fully understand Thermodynamics, which was the subject matter of my comments.
/dr.bill

May 10, 2010 11:11 am

Spector
They are different forms of the same equation. I was just assuming that nR is a constant c, because the number of molecules isn’t changing in the atmosphere.
There are a lot of very complex dynamics going on in the real atmosphere, which is why we have climate models.

May 10, 2010 11:13 am

Spector
There is no “fallacy.” Rather it is simplifications and approximations to demonstrate the underlying principles.

May 10, 2010 11:19 am

dr.bill
There are dozens of degrees of freedom in the system governing the atmosphere. No simple equation is going to accurately explain everything that is going on from absolute zero up to the melting point of basalt, and you can nitpick from now to eternity.
Unless you have a fundamental disagreement that the high temperature of Venus is primarily caused by the high atmospheric pressure, what is your point?

May 10, 2010 11:51 am

TallDave
Exactly, thanks.
I wouldn’t say that Venus never had a thin atmosphere – we don’t have any way of knowing that. It probably built up over some period of time.
But we can say that Earth will never become like Venus, short of a cataclysmic event which raises Earth’s temperature well over 850C.

dr.bill
May 10, 2010 12:03 pm

stevengoddard: May 10, 2010 at 11:19 am
“…you can nitpick from now to eternity.”
Nitpicking is at the heart of science. The details matter.
Regarding “what is my point?”, I would just say the following:
The high atmospheric pressure of Venus is caused by the fact that it has a lot of atmosphere. I don’t know how it got all that atmosphere, and neither does anyone else. Once that atmosphere is present, however, it is not a foregone conclusion that it will get hot. In order for any atmosphere to have a lapse rate, and vertically declining temperature profile, two things are needed: a source of energy at the bottom, and a gravitational field to provide the buoyancy effect that allows the gases to rise. The principal energy source on Earth is the Solar energy absorbed by the surface, which then warms the air, which then rises, cools, sinks, gets re-heated, and all the rest, over and over again. I don’t know what is providing the heating on Venus, and neither does anyone else, but it is not caused by simple pressure. If simple pressure were the cause of temperature, you wouldn’t be able to touch a SCUBA tank.
Off-topic: I was rather disappointed to have you and Smokey, both of whom I respect, “throw darts” at me because I happen to have an education. I worked for that education, and I financed it by working in construction and mining, and by winning scholarships. Ever since collecting all my “pieces of paper”, I have worked hard to improve my understanding of Science and the world in general. I continue to do so, and I have learned a great deal from things I have read on WUWT. To be denigrated simply because I have a PhD in Physics makes about as much sense as denigrating someone because they don’t.
/dr.bill

MarkR
May 10, 2010 12:17 pm

I’m confused, Steve.
Why do the gases in Venus not radiate like Earth’s atmosphere appears to? After all, if heat flux is proportional to T^4, then we should get some very significant radiative loss from the lower atmosphere. Apparently mean Venusian emissivity is over 0.8.
So the outgoing heat flow from Venus’ surface is about 8 kW m^-2, and the heat in from the Sun is significantly less than that (Top of atmosphere is what, ~2.6 kW m^-2, most of which is reflected?)
As I see it, the only way to solve this is one of the following:
1) Venus’ isn’t radiating on that scale & it disproves our radiation laws
2) Something is preventing the heat from escaping at that rate
3) Venus is cooling rapidly
4) Conservation of energy is wrong
5) There is a massive source of heat that Venus gets, we don’t, and it’s not the Sun.
Where has my logic gone wrong?

Jeremy
May 10, 2010 12:34 pm

@omnologos says:
May 8, 2010 at 4:11 pm
In particular, the lapse rate is roughly g/Cp, where g is the acceleration due to gravity and Cp is the atmosphere’s specific heat at constant pressure (cp) divided by the molecular weight. It would be interesting to find out why exactly Cp for Venus would only be 85% of Earth’s.

Well, you’re using little-g, which is not independent of the planet’s mass. Big G would be the universal gravitational constant here. So assuming you mean to use little g, then both Cp and ‘g’ would change between Earth and Venus.

May 10, 2010 12:36 pm

dr. bill,
My apologies for the perfumed academic comment. You write well. Why don’t you write an article for WUWT?
I tend to sympathize more with the people who write the articles, because they’re opening themselves up to some serious criticism here. It’s not like the climate peer review referees who hand-wave their friends’ papers through to friendly journals, while everyone puts roadblocks in front of skeptics’ submissions.
This is how science should be. When you post an article here, you don’t have pals to run interference like climate related journals do.

May 10, 2010 12:36 pm

dr.bill
Earth would also have a lot of atmosphere, except that we have huge amounts of CO2 sequestered in limestones which formed in the oceans.
I r have an iducation two from some real guud universitees.

May 10, 2010 12:39 pm

MarkR
I think the Real Climate explanation is a good start. The atmosphere of Venus has a very large thermal mass and doesn’t change temperature quickly.

May 10, 2010 1:17 pm

dr.bill
I looked some more at your post and it gave the distinct impression that you did not read my article or comments carefully. Your claim that I am blaming static pressure for the heat is absurd.

Because we have a sun providing energy to the periphery of the atmospheric system, the atmosphere circulates vertically and horizontally to maintain equilibrium. Falling air moves to regions of higher pressure, compresses and warms. The greater the pressure, the greater the warming. Rising air moves to regions of lower pressure, expands, and cools. The amount of warming (or cooling) per unit distance is described as the “lapse rate.” On Earth the dry lapse rate is 9.760 K/km. On Venus, the dry lapse rate is similar at 10.468 K/km. This means that with each km of elevation you gain on either Earth or Venus, the temperature drops by about 10C.

Jbar
May 10, 2010 1:24 pm

I’ve estimated the surface temperature of the Earth if the oceans were to vaporize. Let’s call it the
“STEAM-BATH EARTH”
scenario.
(“Steam-bath Earth”, you heard it here first, May 10, 2010.)
(Let me also claim here a first for “Pressure-cooker Earth scenario” while I’m at it. and “Sauna Earth scenario”, “Steam Room Earth”, “Jacuzzi Earth”, “Super-sauna Earth”, “Sweat lodge Earth”, “Fat Farm Earth”, also replace “Earth” in any of the preceding with “world” or “planet”. “Pressure-cooker Planet”. That has a sort of ring, don’t it.)
At a surface pressure of 247 atmospheres with an atmospheric depth of 139 km from the surface to the altitude of 1 atm pressure, the surface temperature could potentially exceed 850 deg C, depending on the environmental lapse rate. (I used 4 C/km, a little below the adiabatic lapse rate for water vapor, which is roughly half that of air or CO2.) That temperature would exceed the decomposition temperature of limestone, releasing CO2 from rock to the atmosphere, eventually turning the Earth into Venus. At this pressure the water vapor would be supercritical, whatever that’s worth.
Of course, this is strictly a small Excel “back of envelope”, and who really knows what kind of chemical reaction would occur under those extreme conditions! Also, it would depend on how the 100% cloud cover affects Earth’s albedo. As I mentioned many posts ago, some clouds have very high albedo while others have a very low albedo, so it all depends on what type of clouds dominate. Another caveat: the lapse rate could be lower. We all know how dark it gets in a thunderstorm. Water vapor clouds could reduce the sunlight reaching the surface to lower levels than on Venus, thereby reducing the surface temperature and the environmental lapse rate.
HOWEVER, I have the transmittance spectrum plots of water vapor right in front of me for present-day Earth conditions in the tropics and these plots tell me that with a few thousand times as much water vapor as that, the “Fat farm Earth” scenario would be exceedingly good at obstructing nearly 100% of infrared longer than 1 micron at that kind of optical depth. Therefore it could take VERY LITTLE sunlight reaching the surface to raise temperatures to 850C in a “Super-sauna world”.
The point being,
Steve Goddard, “So certain are you? Always with you it cannot be done. Hear you nothing that I say?”

George Turner
May 10, 2010 1:44 pm

Steven,
As a quick thought experiment I tried this, but probably got it wrong because it just occured to me.
Venus irradiance at the top of atmosphere is 2613 W/m^2, but about 75% is directly reflected, so the atmosphere absorbs 653 W/m^2. Averaged over the surface area of a ball that’s 163 W/m^2. Only about 65 W/m^2 reaches the surface (again averaged over the surface of the entire planet, day & night).
Since Venus is in thermal equilibrium, it must re-emit 163 W/m^2, which comes from the top of the clouds and up. Stefan Boltzmann’s law puts the temperature at 231 Kelvin, which would be the temperature at the cloud tops. The surface temperature is then found by taking the adiabatic lapse rate, the height of the cloud tops, and the temperature of the cloud tops, just as Carl Sagan did.
So now, to eliminate the greenhouse effect, I eliminate the radiation that hits the surface, which is about 65 W/m^2 (this needs to be double checked – it’s based on 10% of the top of atmosphere). Since that radiation never bounces back up to the cloud tops (via whatever route), the atmosphere now only radiates 98 W/m^2 (163 – 65). That gives a Stefan-Boltzmann temperature of 203 K at the cloud tops, a decrease of only 28 degrees.
If the cloud tops stay about where they were, the adiabatic lapse rate would give a decrease in Venus surface temperature of about 28 degrees, and there can be no greenhouse effect operating because it no longer has any surface illumination at all, which is by definition a requirement of the greenhouse effect.

Nullius in Verba
May 10, 2010 1:47 pm

“Initially Earth didn’t have limestone either, supposedly, but I have not heard any account from a single geologist or planetary scientist suggesting that Earth’s atmospheric pressure was every more than a few atmospheres, so…”
During the Hadean period, an Earth atmosphere of 200atm has been suggested. The surface pressure was so high that a liquid ocean is likely, even though the surface temperature was far above 100C.
“In order for any atmosphere to have a lapse rate, and vertically declining temperature profile, two things are needed: a source of energy at the bottom, and a gravitational field to provide the buoyancy effect that allows the gases to rise.”
Strictly speaking, it requires a source of energy to maintain permanent temperature differences at the surface, to drive convection. Only a difference in temperature can drive a heat engine to do work.
“I don’t know what is providing the heating on Venus, and neither does anyone else, but it is not caused by simple pressure.”
Not simple pressure, no.
There are two essential factors to the greenhouse effect.
There needs to be sufficient differential heating at the surface to drive convection. This differential heating does not have to be very much. It can be as small as you like, so long as it drives convection faster than heat can diffuse. So long as you get vertical mixing being forced, an adiabatic lapse rate will dominate.
And the emission of radiation to space needs to be at least partly from high above the surface. (Radiation internal to the atmosphere has no effect.) While the adiabatic lapse rate sets the gradient of the straight line, the emission to space sets the intercept.
The visible surface at the wavelengths at which the planet radiates, which in the case of a planet with an opaque atmosphere is high above the solid surface, settles at the radiative equilibrium temperature. Then gas descending from this altitude is compressed and increases in temperature. It’s important to say it isn’t “heated”, because no heat is exchanged in order to do so, and in particular, this means there is absolutely no need for a major source of heat at the surface to achieve and maintain this high temperature. The direct source of the energy is the gravitational potential energy, but the conversion is driven by the entropy driving the convection.
The physics is essentially very similar to that of a refrigerator. Gas is driven round a cycle during which it is compressed and expanded, and as a result maintains a temperature gradient between the two ends. The temperature of one end of the heat pump is anchored to a large heat reservoir (the outside of the refrigerator, or outer space) and the temperature of the other is held by the heat pump in a fixed relationship to it (the coolbox, or the surface of the planet). Convection acts as the refrigerator’s pump/motor.
The primary difference between Earth and Venus for explaining the differing climate is the mass of the atmosphere, but there are other essential aspects to the physics too.
steven goddard,
I want to express again my appreciation for this post. I’ve been trying to tell people about this way of looking at the physics for some considerable time. Many I talked to understood it immediately, but I got a lot of determined and not always constructive opposition, too. So I’m not surprised at the reaction here.
Incidentally, all of this is perfectly standard climate science – as promulgated by the likes of Sagan, Manabe, Ramanathan, and so on in the technical literature. But when talking to the general public, they switch to this nonsensical primary-school version of longwave radiation being “trapped”. I believe this is actually based on a purely-radiative model of a non-convective atmosphere that they use as a toy example for teaching, to which they then try to add in convection as some sort of minor modification to it. However, with convection, the “greenhouse effect” is by an entirely different mechanism to which the non-convective/radiative language is inappropriate.
But it’s an absolute joy to see it flying from the masthead at WUWT. So my thanks.

May 10, 2010 1:49 pm

Jbar
At earth surface temperatures, water vapour saturates and condenses between 0 and 100mb. You can’t get a vapour pressure on earth greater than 100mb (1/10th of an atmosphere) outside of an enclosed vessel.

May 10, 2010 1:58 pm

Need to qualify that last remark. A volcano or fire could produce vapour pressures on the surface of 1000mb.

GeoFlynx
May 10, 2010 2:17 pm

Ben’s comments below express my views better than I can. Apparently I can not compose my concern with the basic physics in the last two Venus posts without incurring “moderation”. I acccept this this is my fault alone for being too passionate. However, if basic physics concepts need to be altered to discount the possibility of anthropogenic global warming perhaps that is its greatest validation.
Ben Schumacher says:
May 9, 2010 at 2:06 pm
I keep expecting this thread to be about WHY Venus has developed such a strong greenhouse effect, and whether the term “runaway greenhouse” appropriately describes this process. Fascinating question, and present theories involve a good deal of speculation. I would probably count as a moderate AGW skeptic — a “luke-warmer” in common parlance — but the discussion here does not inspire me with confidence that this community has a firm grasp on the basic physics. Sorry to be so blunt, but the thing is pretty distressing to someone who reads WUWT regularly and with pleasure.

dr.bill
May 10, 2010 2:26 pm

re Smokey: May 10, 2010 at 12:36 pm
(and peripherally, Steve)
Don’t sweat the “perfumed” or “taking heat” comments. I work in an environment where I’m out-numbered at least 10 to 1 by devoted Koolaid drinkers, and I’ve taken a lot of abuse over the past 10 years or so by (quite publicly) pointing out the flaws in their religion. I don’t give a rat’s ass about the abuse, but I do care about the corruption of Science, so I just keep hammering away at them, as they keep jumping from one ill-conceived imminent calamity to the next.
In the case of Steve’s article, I interpreted him to say that pressure caused temperature, and (by my perhaps flawed reading) he still appears to be saying that. Perhaps it’s just a matter of semantics, and perhaps we can just give that a rest. I also am appreciative of the fact that people who present full-blown articles here are opening themselves up to attacks of all kinds, and often from those who have the worst of motives. You may rest assured that I am not among those people. Any comment I make is intended to improve understanding, or to point out what I see as an error. Whether my comments accomplish that, of course, is another matter, but I am also happy to have errors in my own thinking pointed out to me. The final goal is to know what is real, no matter who proposes it.
I look forward to reading more from both of you in the future.
/dr.bill

Jbar
May 10, 2010 3:13 pm

MarkR
It’s number 2: “something is preventing the heat from escaping at that rate.”
That “something” is the infrared absorbance of CO2.
It HAS to be, because there isn’t 8000 W/m2 shining back down on the planet from space, and there sure as hell isn’t 8000 W/m2 coming from the planet’s interior. Not even close.
On Earth, with only 3 pounds of CO2 in the atmosphere per square foot of surface, CO2’s absorbance spectrum is fairly narrow, much less than that of water vapor.
On Venus, with almost 95 TONS of CO2 in the atmosphere per square foot of surface, or almost 40 pickup trucks/ sq foot (yes, a new unit of measure), CO2’s absorbance spectrum becomes so broad that it blocks out almost all infrared from 2 microns to 28 microns. What little water vapor there is in the upper atmosphere (similar in mass to Earth, actually) blocks everything from 28 microns and higher, just like it does on Earth. (RealClimate stated that it is impossible to image Venus’s surface in infrared except for a narrow IR “window” close to 1 micron wavelength.)
So out of the 8000 watts (or 12,000, depending on who’s calculating, but let’s use your 8000 number) leaving the surface, the thick CO2 atmosphere reflects more than 7900 watts back to the surface. The difference, the amount that makes it out to space, is enough to balance out what little sunlight heats the surface, averaged around the planet, so that Venus’s surface temperature remains constant even with this enormous flux of energy flowing up and down.
The massiveness of Venus’s atmosphere was invoked to explain why the surface temperature is uniform around the globe even on the dark side where there is little or no illumination. It is massive enough that it can transport the small amount of solar heating energy from the day side all the way around to the night side with low wind speed. This is especially true since it is not just surface winds carrying that energy but wind over kilometers of altitude that is constantly radiating 8000 W/m2 up and down between wind and ground while moving a paltry 50 W/m2 equivalent horizontally to the dark side of the planet.
Let us consider what would happen if Venus instead had a thick nitrogen/oxygen atmosphere and somehow had clouds with the same albedo as Venus has today. These gases are transparent to infrared (not counting whatever makes up the clouds). That 8000 W/m2 radiating from the surface would go straight up to the cloud layer unimpeded by the O2 and N2 and vaporize the clouds, and then blast its way out into space, cooling the surface fairly rapidly down to something much cooler. The lapse rate might still cause the surface to be warmer than Earth, but I’m not sure about that. I think we need an actual scientist to do modeling to arrive at an answer. My thinking is that the surface would have to be radiatively in balance with incoming sunlight, which would be pretty low radiation. I suspect there would still be some lapse rate from the surface to the cloud bottoms, but it would have to be much smaller than it is today in order to match the low surface radiation balance in an IR-transparent atmosphere. Of course, the clouds 50km up might reflect infrared back down, but nowhere near as much as a dense CO2 atmosphere.
We know it is the gases on Venus today that reflect the bulk of the radiation back down to the ground and not the clouds, because if Venus’s atmosphere were transparent to infrared, the cloud bottoms should then be almost as hot as the ground, and data says that is not true. As Steve’s graph shows, temperature falls gradually from the surface until it reaches about 60C at the cloud bottoms at 48km altitude. Therefore something is preventing 8000 – 12000 W/m2 of radiation from bouncing off the cloud bottoms and that something is the CO2 greenhouse effect.

George Turner
May 10, 2010 3:13 pm

dr. Bill,
Pressure doesn’t cause temperature, but a change in pressure does cause a change in temperature, and the temperature change is precisely defined and commonly used in the the enginnering of all heat engines and refrigeration equipment, as related in Nullius in Verba’s comment above.
If parcels of atmosphere are circulated vertically in a gravitational field they must undergo changes in pressure, and those changes in pressure cause changes in temperature. The gas at higher pressures will be hotter than the gas at lower pressures as long as the pressure changes continue (the gas is transported between regions of different pressures).
So if you spin up a jet engine, even without burning any fuel, the combustor inlet temperature (and without fuel the combustor inlet temperature would equal the turbine inlet temperature) would still be hundreds of degrees hotter than the compressor inlet temperature or the turbine exit temperature. The higher pressure in the compressor makes a hot spot that will remain hot for as long as the turbine spins. If you connected the exhaust to the intake in a closed loop you could use it as a refrigerator or a heat pump, with a hot side and a cold side.
The temperature rise in the compressor (due to ideal gas laws) ultimately limits how many compression stages we can use before the added heat from combustion sends the turbine inlet temperature past the limits of the turbine materials, destroying the engine.
As long as the sun keeps the atmosphere of Venus circulating vertically, the temperature difference will exist. As a side note, no military or commercial jet engines compress the air anywhere near the pressures on Venus (92 atmospheres) because they’d melt.

May 10, 2010 3:19 pm

dr.bill
What I am saying is that because of the high pressure on the surface of Venus, adiabatic heating causes the very high temperatures. Those kinds of temperatures aren’t seen on earth because of the lower pressure.

Keith Minto
May 10, 2010 3:34 pm

Steve, thanks for the topic, it has been a great discussion.

Gail Combs
May 10, 2010 3:38 pm

dr.bill, I have more or less been trying to follow the discussion here. I will admit that my thermo-dynomics education was very weak, the prof had a heart attack and we got stuck with an organic chemistry teacher whose opening statement to the class was “I hate thermodynamics and you are going to pay…” nough said.
I remember PV=nRT as the ideal gas law.
Are you stating that since Venus is an “open” system the gas law does not apply? And if so what about the effects of gravity? What about the chart that shows that about 1 earth’s atmosphere the temp on Venus is similar to that of earth? Is the problem that the atmospheric gas on Venus is not being actively compressed or expanded?

May 10, 2010 3:46 pm

Jeremy – yes I understand that g and Cp are bound to be different. My question was why is Cp for Venus be 85% of Earth’s. The point is that we cannot asser that the lapse rate is independent from composition.

Bob_FJ
May 10, 2010 3:56 pm

Stevengoddard, Reur May 10, 2010 at 12:39 pm

I think the Real Climate explanation is a good start. The atmosphere of Venus has a very large thermal mass and doesn’t change temperature quickly.

I’ve not read the RC explanation but cannot see that the concept is a prime cause of uniform average surface temperature, (although it would help), for several reasons, but broadly:
Whilst the mass of the Venus atmosphere is ~92x more than the Earth’s, the nighttime cooling period is 117x longer. It does not seem possible that there is no heat loss over such a long period, even if we were to consider simple conduction alone. Because of the huge temperature and pressure range, it seems probable that there is very strong convection. Certainly, the observed upper winds that circulate the planet in ~4 earth days, and the polar vortices are violent, and not understood. (a lot of ‘work done’ energy BTW) There are inadequate data for what happens below. It seems to me that the (unknown) atmospheric dynamics are a likely candidate. Convection, (which validates your pressure hypothesis), may even be stronger at nighttime; see discussion below that I’ve extracted from my earlier comment:
[1] When facing the sun, she is said to receive at the surface about 10% of sunlight. Whether this is a midday or total facing area average, or the effects of scattering, I don’t know, but whatever, the surface receives SOME solar energy. This energy amount must be lost back to space because the planet is apparently in thermal equilibrium. The fundamental process for this should be convection and conduction. Additionally, since infrared photography etc of the surface has been accomplished there is at least one window for some infrared to escape directly to space.
[2] At nighttime she no longer receives any solar energy, but has capability to lose heat in the same way as on the daylight side, over a period 117 times longer than on Earth. What is more, because the upper atmosphere is no longer heated by solar infrared, (~40% of sunlight), the temperature gradient of the atmosphere should increase, inferring increased conductive/convective cooling. Yet, there is no change in surface temperature! Such a condition would require an impossible perfect insulation layer, [with] no geothermal energy, but clearly, this is not the case.

GaryW
May 10, 2010 4:21 pm

#
stevengoddard says:
May 10, 2010 at 8:39 am
GaryW
A much simpler empirical exercise would be to answer qualitatively this simple question.
What would happen to temperatures on Venus if you could strip away 99% of it’s atmosphere (making it like earth?)
Hint:
http://www.astro.wisc.edu/~townsend/resource/teaching/diploma/venus-t.gif
Steve, I’m not at all sure how a graph of atmospheric temperature versus altitude either simplifies or settles the discussion. The question at hand is not whether temperature varies with altitude but rather why it does. The strawman you present is that pressure is the determining factor in atmospheric temperature. Where I have a problem is that appears you are saying that the temperature of a gas is entirely determined by its pressure. That, in my mind, would mean that you would assign a specific temperature to a specific pressure in the senses that 100 PSIA CO2 would always exhibit some specific temperature X under all possible situations. That is what is coming across to me in your text.
Let me have a shot at this again. I think what you are trying to say that the adiabatic lapse rate of an atmosphere determines the basic slope of the environmental lapse rate. Other factors such as phase and chemical changes is atmospheric makeup modify that rate but a mixing atmosphere will always thermally settle to something close to the adiabatic lapse rate.
Further more, I believe you are saying that Venus’s surface level atmospheric temperature is higher than Earth’s because of the difference in depth of their atmospheres. The inherent adiabatic lapse rate of the mostly CO2 atmosphere on Venus provides its base temperature profile and since its atmosphere is deeper (and thus at a higher pressure) that lapse rate must force temperatures higher as the depth increases.
Also, I can see an argument that says the low pressure end of the linear part of the atmospheric lapse curve is determined by something like the temperature of the tropopause. The temperature at lower altitudes tends to follow the adiabatic lapse rate and if tropopause temperatures are similar then planets with deeper atmospheres will have higher surface temperatures.
Actually, I can pretty much allow that much. What appears to be missing from your descriptions is what factor causes atmospheric lapse rate to fairly well (but not exactly) follow adiabatic lapse rate. When you use PV=nRT as your basis, my problem is that in an unrestrained situation like a planetary atmosphere, it is entirely likely that a situation can be found in which nRT is constant but P varies. That is -40 degrees C on a mountain top and -40 degrees C in the arctic at sea level. Clearly the pressures will be different with constant temperature with the same mass of air but what will be different is the volume so that PV equals nRT. The total potential energy available from the arctic air mass will be a bit greater because of the higher pressure but the kinetic energy of each molecule will be the same.
All I can see that might cause an atmospheric temperature profile to follow the adiabatic lapse curve is vertical movement within the atmosphere. Somehow, that causes the PV=nRT thing to take over.
I may be getting closer to your point of view but still haven’t been able completely justify that last little bit.

Ralph
May 10, 2010 4:26 pm

>>> Dr Bill
>>>Nevertheless, the pressure at sea level NEVER changes in
>>>any SIGNIFICANT way from 1000mb, no matter where you
>>>are on Earth, or what time of the day or time of the year it
>>>happens to be.
Errrm. The International Standard Pressure is 1013.25 mb, not 1000. And since I have seen 936 mb, your claim that the pressure does not change significantly is rather wide of the mark.
.

Stephen Wilde
May 10, 2010 4:34 pm

Despite Steve’s best efforts this thread is going off track into unnecessary levels of detail and thereby causing confusion.
Here’s a renewed link to my 2008 article which supports Steve’s contentions in simple (ish) terms. I hope it helps some:
http://climaterealists.com/index.php?id=1562&linkbox=true&position=4

Bob_FJ
May 10, 2010 4:40 pm

Dr.Bill
Since you are an expert in the field, could you please help me with my suggestion/enquiry here, concerning S-B emission levels of a body when it is immersed in a fluid? (and as to how it might affect claims here of 8,000 and 12,000 W/^2 from the surface).

Ralph
May 10, 2010 4:42 pm

>>>Spector.
>>If there are no rising or falling parcels of air then there can be no
>>>adiabatic cooling or warming.
Nonsense.
On a fine stable 1040mb day with zero convection and no downward flows there will still be an adiabatic lapse rate in the atmosphere (a dry one in this case). The lapse rate is always present, courtesy of gravity. The reference to ‘adiabatic temperature’ change is because we assume a theoretical parcel of air rising or falling in the calculations, even if the atmosphere is totally stable.
The temperature of the air at 35,000ft does not suddenly warm to 20oc, just because the atmosphere is not convective that day.
.

Ralph
May 10, 2010 4:53 pm

>>>Steve Goddard
>>>If the atmosphere (of Venus) were thinner (like earth) the
>>>temperature would be much lower. Does anyone actually doubt this?
>>>This isn’t theory. It is observational data.
But not if Venus had the orbit of Mercury – the thinner Venusian atmosphere would actually be much hotter than Earth.
You cannot leave the Sun out of this completely – a thick atmosphere does not equal a high temperature all by itself. Like the diver’s air tank, a thick atmosphere can cool down, if Venus had the orbit of Saturn, for instance.
What you need to concentrate on, Steve, is the fact that the presence of high levels of CO2 on Venus does not end up with a significantly higher lapse rate than Earth. If CO2 was the driver of temperature, the lapse rate on Venus should be much higher.
The thickness of the atmosphere on Venus is a complete distraction. This should be so whatever the atmosphere density on Venus. Even if Venus only had the same atmosphere density as the Earth, if CO2 was the driver of temperature and the Venusian atmosphere was still 99% CO2, then the lapse rate on Venus should be much higher than Earth – but it is not, and so CO2 is not the main driver of surface temperature.
Q.E.D.
.

May 10, 2010 5:20 pm

GaryW
The reason that graph is important is because if you truncate off the bottom 50km, you have something similar to the earth – in spite of the fact that the composition of Earth’s atmosphere is very different.
http://www.astro.wisc.edu/~townsend/resource/teaching/diploma/venus-t.gif

Keith Minto
May 10, 2010 5:21 pm

Air that is dense transfers its energy more efficiently to a temperature sensor. The sensor will then read higher at a lower altitude than at a higher altitude as per the wet/dry lapse rate. The temperature is lower at higher altitude because of a lower efficiency of transfer of molecular heat energy NOT necessarily because of a lower temperature.
If you standardize pressure and then read temperature at different altitudes then you have eliminated one variable. I live at 800m and the summers are cooler than the plains because the thinner air is less efficient at transferring its heat energy too me. So the lapse rate is measuring the rate of energy transfer (collision rate) from air to sensor as well as air molecular energy.
Makes it confusing doesn’t it ?
My suggestion would eliminate one variable by eliminating the apparent temperature rise with increasing air density and revealing a truer reading of gas transfer energy.

dr.bill
May 10, 2010 5:22 pm

Pardon me if this sounds like a lecture, but here goes anyway:
George Turner: May 10, 2010 at 3:13 pm
The 1st Law of Thermodynamics can be expressed as dU = dQ + dW, where U is the internal energy of the gas. Part of this is in the form of Kinetic Energy of Translation of the molecules (i.e. their ordinary speed as whole objects). That, and only that, is what determines the temperature of the gas. There are other “degrees of freedom” by which the molecules “store” energy, namely in their Rotational and Vibrational motion. These contribute to the heat capacity of the gas, but not to its temperature. If a molecule has a lot of internal degrees of freedom, it can “suck up” a lot of energy without getting hotter. That’s why some things are harder to heat up than others.
To change the internal energy of a gas, you have to either heat it (the dQ part) or do work on it (the dW part). When you spin up your jet engine, you are doing work on the gas by compressing it, and it will get hotter, because you are making the molecules mover faster by doing work on them, and the gas will stay hot as long as you keep spinning. I have no quarrel with that.
In the atmosphere, however, there is no external agent doing work on the gases. It is only the dQ part that matters, and that heating must occur at the bottom. If you heated some layer higher up in the atmosphere (which happens, for example, in the upper part of the stratosphere where the ozone layer occurs, and which is heated by the Solar Wind (high energy particles) coming in from above), it will NOT start heating the air layers BELOW it, because its density will decrease as it heats up, and it will thus tend to RISE because of the buoyancy effect of our gravitational field.
By the time you get to the top of the stratosphere, however, all but a tiny fraction of the mass of the atmosphere is below you, there is precious little buoyancy left anyway, and thus there is simply a cooling gradient (negative lapse rate, if you wish) downward from the tropopause. In any case, the stratosphere is hotter at the top because that is where the heating occurs, and the troposphere is hotter at the bottom because THAT is where the heating occurs.
Gail Combs: May 10, 2010 at 3:38 pm
The Gas Law DOES apply to our atmosphere, and to that of Venus, and I don’t see that there is anything wrong with your understanding of it. They are both, however, open systems, in the sense that they have no fixed volumes. They can expand and contract, depending on how much heating is done on them. That’s why the tropopause at the Poles is much lower than at the Equator. There is also, as I noted in my response to George Turner, no external agent applying forces to either atmosphere, as there is in the case of jet engines or compressors of any other sort, so it all depends on the heating.
To All
For what it’s worth, I am not trying to be a nuisance to Steve or anyone else, and I am sorry if my comments have created that effect. In my time being a reader of WUWT, and disregarding the trolls, it has been my impression that people want to know the reality of the Science involved in these issues. Many of these issues involve Thermodynamics, which I happen to know something about, partly because I teach it. I am not trying to pontificate. Everything that I have said here, and in earlier posts, can be found in standard textbooks. My purpose is mainly to point out that such knowledge exists, and that people can study it themselves once they are aware of its existence.
/dr.bill

dr.bill
May 10, 2010 5:26 pm

stevengoddard: May 10, 2010 at 3:19 pm
dr.bill
What I am saying is that because of the high pressure on the surface of Venus, adiabatic heating causes the very high temperatures. Those kinds of temperatures aren’t seen on earth because of the lower pressure.

Steve, I do understand what you are saying. You are saying that the troposphere is hotter at the bottom because of adiabatic heating, whereas I am saying that the troposphere is cooler at the top because of adiabatic cooling. I don’t see where we can go from here without a referee – or maybe a few beers. 🙂
/dr.bill

Dave
May 10, 2010 5:26 pm

“The weak bands are linear, but with a small slope. That is why they call them “weak bands.””
Did you even look at the Figure in that paper? Did you even try to think about this? This is about the 4th time I’m going to try to get through to you on this one: at current concentrations, the strong bands are saturated at the middle, so increasing concentration causes the radiative forcing due to those bands to increase only logarithmically. The weak bands, as you point out, while increasing linearly are weak, so the logarithmic relationship of the strong bands dominates.
However, every time you double the concentration, you reduce the slope of the logarithmic relationship, but the slope of the linear relationship stays constant. If you had looked at Figure 1 in the linked paper (http://journals.ametsoc.org/doi/pdf/10.1175/1520-0469%281977%29034%3C0448%3AARCMSO%3E2.0.CO%3B2) you’d see that the weak bands contribute about 1/5th of the additional radiative forcing from a doubling of CO2, but about 1/3rd of the additional radiative forcing from increasing concentrations by a factor of 10, and, eyeballing the chart, it looks like once concentrations increase by a factor of 20 the weak bands could be contributing about as much to the increase as the strong bands did. A factor of 20 is puny compared to a factor of 2000 for increasing the CO2 in Earth’s atmosphere to 95%, and a factor of 200,000 for the CO2 in Venus’ atmosphere.
So once CO2 exceeds about 1% of the atmosphere, the linear weak band is dominating, and radiative forcing increases _much_ faster than if you assume the logarithmic relationship holds to infinity. (eventually the weak bands saturate too, so the linear regime is limited, but you only need to be linear for a few doublings before returning to log-land to double or triple the final temperature change.
Therefore, your estimate of temperature increase from a 100% CO2 earth is a very large underestimate – “Earth already has plenty of greenhouse gases, so my point is that even if earth went to 100% CO2, the maximum temperature increase would be less than 36C – nothing like Venus. (In reality, it would get cooler because of the loss of water vapour.)” And your estimate of the effects of a reduction of CO2 from 95% of the atmosphere of Venus to a couple % are a large underestimate.
(I’ll also note that the assumption that the climate sensitivity remains constant at 3 over orders of magnitude CO2 concentration changes is also crazy).

dr.bill
May 10, 2010 6:30 pm

Bob_FJ: May 10, 2010 at 4:40 pm
Dr.Bill
Since you are an expert in the field, could you please help me with my suggestion/enquiry here, concerning S-B emission levels of a body when it is immersed in a fluid? (and as to how it might affect claims here of 8,000 and 12,000 W/^2 from the surface).

First of all Bob, I do not claim to be an expert in anything – except perhaps fly-fishing and getting into trouble with redheads, but I may be deluding myself there too (about the fly-fishing, that is – I definitely get into trouble with the redheads). 🙂 Like everyone else, however, I do know a few things about the topics I make my living at.
Your question is rather complicated, and I’m not sure how to reply to some parts of it. What I can say, however, is that the Stefan-Boltzmann and blackbody issues in general are all about surfaces. Sometimes, however, it is hard to define where the surface of an object is located. The Sun, for example, radiates as if it had an effective radiative temperature of a bit less than 6000K, but it is clearly much hotter than that on the inside, and the light it emits doesn’t come from just one thin layer. The same is true of the Earth. Some of our emissions come straight from the surface, but others come from the molecules in the atmosphere. In essence, very few objects can be classified as true single-temperature blackbodies, or even true greybodies. If we add up all the radiation coming from all levels of something like the Sun or the Earth, or better still – measure them, we can “fit” a curve to it all and thus obtain some kind of effective average temperature. That’s about as good as it gets.
In the case of your block of dry ice, you should note that solid CO2 generally sublimates directly to the gas phase when it melts, and thus you wouldn’t have much of a liquid coating to work with. If you had a block of regular ice, however, both the remaining ice and the liquid skin would emit radiation. If the liquid stayed the same temperature as the solid, they would radiate the same profile. If the liquid part warmed up a bit, it would have a different distribution, but the solid part would also radiate right through the liquid, and one would thus have a “composite” distribution.
It’s little wonder that this has given you headaches! 🙂
/dr.bill

wayne
May 10, 2010 6:58 pm

Dr. Bill, excuse me but I am going to jump in here if I can.
I see your point but I also see Stevens point. That is why I’m now confused. Answer a question since you are TD versed and I’m a bit limited there. Take some molecules with a given, constant velocity. Every molecule has the same velocity, always. Take one cubic cm near Venus’s surface and take a one cubic centimeter from 50km up in the atmosphere where Earth like properties exist and the density is some 90 times less. Put a thermometer in each cubic centimeter. What are the two temperatures. Equal? Does the definition of temperature itself depend on density (number of molecules per volume) or only on the average velocity of the molecules.
I’ll reserve another question later depending on your answer.

GeoFlynx
May 10, 2010 7:27 pm

I withdraw my comment

May 10, 2010 7:50 pm

dr. bill,
If this is all standard science as you claim, why are we subjected to so much misinformation about Venus from the scientific community?
http://www.is.wayne.edu/mnissani/a&s/GREENHOU.htm

It is impossible to predict with certainty anything as complex as major climate shifts; we can only make educated guesses. Still, the most reasonable guess leads most scientists to worry about the long term effects of greenhouse gases on Earth’s climate. In fifty years, if we continue the unbridled release of greenhouse gases into the atmosphere, temperatures might be high enough to melt the polar ice caps, raise sea levels, and submerge low lying areas such as Louisiana. Indeed, if all the icecaps completely melted, sea level might rise by as much as 200 feet, drowning the coastal areas of all continents, including many of the world’s largest cities. Climates may shift dramatically, perhaps turning once prosperous agricultural areas into deserts. If the process continues unchecked, it could reach a point of no return, as it apparently had on Venus. As the Earth heats up, water would convert into steam, which is a greenhouse gas. The Earth would get hotter and hotter. As the atmosphere heats up, more CO2 might escape from its present location in ocean rocks and shells. Beyond a certain point, the process may be self-sustaining. Venus tells us how far such a process can go—hellish temperatures, a cloud cover that lets less sunlight reach the earth but helps trap the heat, enormous pressures at ground levels weighing heavily upon everything and distorting the landscape. Venus provides us a timely warning. If we pay no attention, in some 800 years life on Earth might perish.

dr.bill
May 10, 2010 7:58 pm

wayne: May 10, 2010 at 6:58 pm
Hi Wayne,
Temperature is defined in terms of the average Translational Kinetic Energy of a molecule, nothing more. An individual molecule doesn’t have a temperature – it only has mass and speed. Temperature is an “emergent property” that is valid only with large numbers of things. Pressure is a similar concept. Even in a cubic centimeter, however, there are generally way more than enough particles to satisfy this condition.
In any case, the expression is (1/2)mv² = (3/2)kT. Note that it isn’t just the mass or the speed that matters, but the combination. One consequence of this is that Oxygen molecules in the air are (on average) moving a bit slower than the Nitrogens. If there were some Hydrogens around, at the same temperature, they would by “flying” in comparison with the others. Some molecules, however, are always moving faster than average and others slower, even in a single-component gas. (Look up the Maxwell-Boltzmann velocity distribution.)
If we take your two identical cubic centimeters of molecules and put them anywhere, and keep them segregated, and don’t do anything to them, they will (initially at least) have exactly the same temperature. I’d actually go for a liter myself, unless you have a really tiny thermometer. Measuring changes the result. 🙂
/dr.bill

Nick
May 10, 2010 8:08 pm

Steve, that’s from the website of a Liberal Arts College’s Interdisciplinary Studies department. It was written by someone with a bachelor’s in philosophy and a Ph.D. in genetics. It’s also from 1990.
If it was from an atmospheric sciences, or even earth and space sciences department, you might get away with a block quote like that as being a sign of scientists lying to people.

Keith Minto
May 10, 2010 8:12 pm

On a different aspect of this topic, it would be welcome if a geologist could comment on these surface images provided by Colin. Have a look at

http://www.solarviews.com/raw/venus/v13corr.jpg – the surface looks black like lava.
Further discussion about the color of light on Venus is available at Don Mitchell’s excellent site: http://www.mentallandscape.com/C_CatalogVenus.htm

Is this old lava flow?, why are the surface rocks flattened?

dr.bill
May 10, 2010 8:17 pm

stevengoddard: May 10, 2010 at 7:50 pm
dr. bill,
If this is all standard science as you claim, why are we subjected to so much misinformation about Venus from the scientific community?
http://www.is.wayne.edu/mnissani/a&s/GREENHOU.htm

Hi Steve,
Personally, I’d call BS on all of that, and have been doing so on this particular issue for over a decade. As time goes by, however, I find that not as many of my colleagues are spouting such nonsense all the time. Those who do are generally on the receiving end of some kind of “benefits”. It’s possible, of course, that some people, scientists or otherwise, actually believe all that bull[snip] (save you the job CTM), but anyone who has actually looked at the matter realizes that it’s all a money and power-grabbing hoax.
I have lived through the “Coming Ice Age”, the “Population Bomb”, and about a dozen other major ones since becoming an adult. All they have in common, in my opinion, is the desire to funnel large amounts of money and resources into a small number of pockets, and shape the world into a certain mould, preferably without the presence of much of the 3rd world. Perhaps I’m just cynical, but that’s what my gut (and a lot of reading) tell me.
/dr.bill

May 10, 2010 8:17 pm

Speaking of Venus, it was incredibly bright tonight – due to that very high albedo.

May 10, 2010 8:41 pm

Keith Minto
Basaltic lava is pretty much the standard fare for planets besides earth. In order to form granites (continental crust) you need to have water in abundance.

May 10, 2010 8:44 pm

Nick
The reason people believe that kind of nonsense is because that is what they have been told.
http://en.wikipedia.org/wiki/Carl_Sagan
Sagan established that the atmosphere of Venus is extremely hot and dense with pressures increasing steadily all the way down to the surface. He also perceived global warming as a growing, man-made danger and likened it to the natural development of Venus into a hot, life-hostile planet through a kind of runaway greenhouse effect.

May 10, 2010 8:51 pm

Here is one from the American Institute of Physics :
http://www.aip.org/history/climate/Venus.htm

Finally in 1960 a young doctoral student, Carl Sagan, took up the problem and got a solution that made his name known among astronomers. Using what he later recalled as “embarrassingly crude” methods, taking data from tables designed for steam boiler engineering, he confirmed that Venus could indeed be a greenhouse effect furnace.(5) The atmosphere would have to be almost totally opaque, and this “very efficient greenhouse effect” couldn’t all be due to CO2. He pointed to absorption by water vapor as the likely culprit.
by the late 1970s, NASA climate modeler James Hansen stated confidently that the sulfates together with CO2 “are responsible for the basic climatic state on Venus.” Hansen had originally become interested in the greenhouse effect when, in response to Sagan’s primitive calculations, he tried to derive a better explanation of why the planet’s atmosphere was so hot. Now Hansen’s findings about sulfate aerosols strengthened his belief that these particles could make a serious difference for the Earth’s climate as well. Sulfates were emitted by volcanoes and, increasingly, by human industry, so Venus had things to tell us about climate change at home.(13*) CO2 was by far the largest factor in warming Venus

dr.bill
May 10, 2010 8:55 pm

George Turner: May 10, 2010 at 3:13 pm
re dr.bill: May 10, 2010 at 5:22 pm
I made a typo in my response to you.
Here’s what I said:
By the time you get to the top of the stratosphere, however, all but a tiny fraction of the mass of the atmosphere is below you, there is precious little buoyancy left anyway, and thus there is simply a cooling gradient (negative lapse rate, if you wish) downward from the tropopause. In any case, the stratosphere is hotter at the top because that is where the heating occurs, and the troposphere is hotter at the bottom because THAT is where the heating occurs.
I should have said: “downward from the stratopause“, which is roughly at a height of 50km. That negative lapse rate persists downward for about 30km from there, and then there is an isothermal region about 10km high that has the same termperature as the tropopause.
Sorry for any confusion,
/dr.bill

Keith Minto
May 10, 2010 8:55 pm

stevengoddard says:
May 10, 2010 at 8:17 pm
Speaking of Venus, it was incredibly bright tonight – due to that very high albedo.

Yes, very prominent in the WNW sky in Canberra, very little light pollution where I live. I was checking it out two nights ago in my 10 power binoculars when a meteor passed just under Venus, within my field of view. Just amazing !
My ‘Astronomy Australia 2010’ tells me that Venus will be 2deg above the thin crescent moon (1deg in WA) on the 16th May and an occultation will be observed across North Africa, the Middle East, China and Indonesia.
By the 15th of May, Venus will have a diameter of 12.o” and a magnitude of -3.9.

wayne
May 10, 2010 9:19 pm

Steven, I had hoped to get a response from dr.bill in my post at wayne says: May 10, 2010 at 6:58 pm. I have given much thought to you two articles on Venus. Very intriguing. See if this aligns with your thoughts on the lapse rate at a more molecular level.
Every planet with an atmosphere also has a lapse rate. Earth has one, Venus & Mars too, and even though I have never read it, I assume the major gas planets have one also. I had to stop here and think. Why? Much of what I am going to state below you will just say, I know that, but I have to state it so we are parallel and you know exactly what I am aware of.
As you have been saying of Venus, it is the pressure. The atmospheres of planets create a very special case in physics. You cannot create experiments in the lab that would ever exactly mimic the physics driving the atmosphere in this one aspect, gravity holds the gases in place with increasing pressure as the altitude descends, no work being performed on the gases. That property is unique to atmospheres, because of the gravity. But why would temperature naturally increase with the pressure?
It took me a while thinking on that point before the light bulb went off… all velocities of the molecules must equalize naturally throughout the entire atmosphere. Of course they are never the same but that is the tendency. That is how photonic heat absorbed in the upper atmosphere is transferred to lower (and hotter) levels.
This is very anti-logic and anti-normal-thermodynamics! Thermodynamics states that heat can only move from hotter to colder, never the opposite direction, but in the special case of atmospheres held by gravity, that is exactly what happens and it is because the molecular velocities are striving to equalize.
Isolate this in your mind to one single molecule for simplicity. If that molecule accepts a photon and is accelerated to a higher velocity, it’s increased velocity will be distributed to the neighboring molecules in ALL directions, even though lower molecules happen to be, because of gravity, at a slightly greater pressure. If all molecules tend to have equal velocities throughout the entire atmosphere, the surface WILL be hotter than higher in the atmosphere.
Steven, on a molecular level is that not what you are saying, that is why lower levels are always hotter and that is how radiation in the upper atmosphere makes it way to the surface at a higher temperature? I never had thought so deep on the definition of temperature itself, never needed to before. If all of the above is true, most definitions for temperature you find on the internet are somewhat mis-stated.
For dr.bill:
I am assuming you would have written back to my question above that the temperature of the cubic cm from the surface would read much higher than the cubic cm taken high in the atmosphere even though the molecular velocities in each case were equal. That makes sense, it contains more kinetic energy per volume than the one up high at lower pressure.

wayne
May 10, 2010 9:55 pm

dr.bill says:
May 10, 2010 at 7:58 pm
Just noticed you did respond. So you are saying temperature has nothing to do with the total energy within a given volume which matters directly with pressure (density)? In case one, taken at the surface, with 90 times the molecules at ½mv² it has 90 times the energy as case two taken at a high altitude, all molecular velocities being the same. Are you sure of that?
I would have said that for the two samples, to have the same temperature (OK, too small for the thermometers, make them liters), the one at high pressure would have to have √90 of the velocity so the energy per volume would be the same. Like just grabbing some numbers: ½·10·500² = ½·(10*90)·(500/√90)².
I thought temperature measured the average kinetic energy, not simply average molecular velocity, one ignores mass. Are you sure of your statement? After all of these years following physics I find the EXACT definiotion of temperature still somewhat eludes me.

George Turner
May 10, 2010 10:24 pm

Dr. Bill,
Regarding one of your recent above posts (I’m working backwards and a little tipsy from watching “Hancock”), I think one of the problems with Stefen-Boltzmann’s law is that we use it to reduce infinitely variable and complex spectral emissions into a simple number, thus destroying information in return for a handy-dandy yardstick more useful for marketing fluorescent bulbs than doing physics.
As a side note on “Hancock”, I’m upset that Charlize Theron went out with Keanu Reeves last week when we all know she deserves a sceptical climatologist.

George Turner
May 10, 2010 10:31 pm

Bill,
Regarding your comment at 5:22?, (ie. no work is done on descending air parcels.)
Yep, that’s external work done on the gas as it trades off potential energy (from PE=mgh).
To compress a gas is to do work on it, as it requires a force applied through a distance, which is the definition of work (W=F*d).
It may seem like no work is being done on a descending air column, but believe me, the work is enormous (though largely undocumented until we figure out how to tax it).

dr.bill
May 10, 2010 11:01 pm

re: wayne: May 10, 2010 at 9:19 pm
and wayne: May 10, 2010 at 9:55 pm
Hi Again Wayne,
You said a couple of contradictory things there, perhaps accidentally, but your belief that temperature is equivalent to kinetic energy, and not just velocity, is correct. The exact definition of temperature is the one I gave in my previous note to you. Here it is again: (1/2)mv² = (3/2)kT, where k is the Boltzmann constant. This is just equal to the Gas Constant divided by Avogadro’s Number, so it is like a “gas constant per molecule”. You can replace m by M (the molar mass), and k by R (the usual gas constant), and the equation still holds, as long as v still refers to the average speed of a molecule.
You are also not wrong about your belief that heating only occurs from hotter to colder. There are three mechanisms by which one object can heat another. They are Conduction, Convection, and Radiation. If you turn on the burner of your stove, for example, there are three ways to tell that it is hot. You can touch it (conduction), or hold your hand above it (convection), or just look at it (radiation).
For the first two, only a trivial amount of energy will generally be transferred from a cold object to a hot one, but radiation is “exempt” from that, in a certain sense, but not entirely. A cold object can emit lots of photons that can be absorbed by a hotter object, thus increasing its internal energy. The hot object will, at the same time be emitting even more photons toward the colder object, more than making up for what it absorbed. In the end, the hotter object heats the colder one, but there can still be a back-and-forth during the process.
The hotter to colder rule is with respect to the NET transfer between the objects, and the hot one always “wins”. On the other hand, you need to keep in mind that all Thermodynamic processes are statistical averages over time, and at the microscopic level there are fluctuations in both directions.
In addition to these three mechanisms, there is also Mechanical Work, which is what is being done on a gas when you compress it. This, however, requires an external agent of some kind that exerts a force over a distance interval. You might find my note to George Turner (dr.bill: May 10, 2010 at 5:22 pm) useful in that respect.
Hope that helps,
/dr.bill

May 10, 2010 11:28 pm

Gail Combs says:
May 10, 2010 at 3:38 pm
I remember PV=nRT as the ideal gas law.
Are you stating that since Venus is an “open” system the gas law does not apply? And if so what about the effects of gravity?

P=ρRT/M where ρ is the density (n/V)
hydrostatic equation dP=-ρgdh where h is altitude
So dP/P=-Mρgdh/ρRT = -Mgdh/RT
Integrating from the surface to the altitude h: P(h)=P(0)exp(-Mgh/RT)
I.e. the barometric formula.

dr.bill
May 10, 2010 11:29 pm

re: George Turner: May 10, 2010 at 10:24 pm
and George Turner: May 10, 2010 at 10:31 pm
Hi George,
Sorry to hear about your troubles with that unappreciative Charlize.
Perhaps in time she’ll see the light. 🙂
Regarding your other comment about work being done by gravity, I didn’t actually say that, nor did I intend it. You are perfectly correct in saying that gravity does work on the descending gas molecules.
There are, however, a couple of caveats. For a variety of reasons, and even in the absence of atmospheric friction or drag, the descending molecules will have less kinetic energy when they arrive back at the ground than they had when they headed upward. If this weren’t the case, we would have unlimited energy due to a perpetual motion machine.
And then, of course, THERE IS the atmospheric friction, which is just as real for air molecules moving through each other as it is for your hand moving through the air if you stick it out the window of your car while driving at highway speeds (and molecules move around much faster than that). This drag slows the molecules down, and transfers much of the energy they picked up from gravity to the other molecules through which they are moving. By the time they get to the bottom, they’re pretty well tuckered out and ready for a recharge from the ground.
As it always seems in atmospheric phenomena, there are partial exceptions to this as well. The case of Chinooks, Foehn winds, and other descending adiabatic processes that involve movement of entire air masses will avoid this energy loss to some extent my “moving as a whole”, so that the drag is limited to the surface of the air mass, and not to its interior.
Lots and lots of book-keeping! No wonder those GCM’s don’t work!
/dr.bill

Spector
May 11, 2010 12:07 am

RE:dr.bill says (May 10, 2010 at 5:26 pm) “… You are saying that the troposphere is hotter at the bottom because of adiabatic heating, whereas I am saying that the troposphere is cooler at the top because of adiabatic cooling.”
Just a thought: Is it possible that the reason that temperatures at the tropopause are so cold is because the heat [energy] there is being radiated by a distributed volume of trace gases such that any given surface is only emitting a fraction of the total thermal energy required to balance the thermal input from the sun?
I believe in the case of the Earth the ‘nominal’ non-reflected earthshine is supposed to be 235 W/m2 which requires a black-body radiation temperature of about 253.7 deg K. I understand that the temperature at the tropopause is may be as low as 217 deg K which can only force a black-body energy flux of about 126 W/m2. While one might say the other 109 W/m2 comes directly from the surface, I wonder if we really do know what is actually driving the temperature and altitude of the tropopause.
Perhaps the narrow band nature of the radiation spectra of trace gases distributed in a transparent volume makes this region a more effective energy radiator than we now expect. [Pure Speculation.]

Ralph
May 11, 2010 12:38 am

Dr Bill
Quote:
If you heated some layer higher up in the atmosphere, it will NOT start heating the air layers BELOW it, because its density will decrease as it heats up, and it will thus tend to RISE because of the buoyancy effect of our gravitational field.

Rubbish.
As long as you have differential heating, then you will get convection, which will eventually encompass the entire air mass.
A hotter column of air, heated half way up, will expand and find itself at a higher pressure at altitude, and thus spread outwards. This will lower the pressure AT THE SURFACE (not half way up). This will induce convection at the surface, as well as half way up.
(This is a standard atmospheric convection explanation)
.

Keith Minto
May 11, 2010 12:42 am

dr.bill,

…. the descending molecules will have less kinetic energy when they arrive back at the ground than they had when they headed upward. If this weren’t the case, we would have unlimited energy……..

and

By the time they get to the bottom, they’re pretty well tuckered out and ready for a recharge from the ground.

I know that the molecules will have less potential energy when they arrive back at the ground and I guess that that energy has been spent in downward movement due to friction under gravity. But can this solid textbook information be applied to Venus?. Can you say then that the Venusian surface is heating the air? That is the only outcome from this reasoning.

wayne
May 11, 2010 1:07 am

Hi again dr.bill –
Hate to press a point but the figures do jive. You speak to me as if I were a young student and that’s OK but you do need to realize I have loved and followed physics to great depths for most of my life so even though we are by no means equals, I don’t need to be led. My questions are much, much deeper.
Figured the best way to show you is by figures which, since you teach thermodynamics, you will understand. This is using the equations you stated above of which I am familiar. Some assumptions apply: 100% co2 atmosphere, in this example all of the atmosphere is of the same constant temperature, all output figures are SI (internally converted).

Precision(4);                  = {4}
k = ‹R/NA›;                    = {1.381E-23}
// This is at 1 atm
molPerL = (1.562‹g/mL›/44.010‹g/mol›)→‹mol/L›;
                               = {35.49}
/* CASE ONE AT SURFACE */
P = 90‹atm›;                   = {9.119E+06}
V = 1‹L›;                      = {0.001}
n = 90‹atm›•molPerL;           = {3.237E+08}
T = P•V/(n•‹R›);               = {3.389E-06}
m = 90•1.562‹kg/L›;            = {1.406E+05}
v1 = √(3•k•T/m);               = {3.16E-17}
/* CASE TWO AT ~52 km */
P = 1‹atm›;                    = {1.013E+05}
n = 1‹atm›•molPerL;            = {3.596E+06}
T = P•V/(n•‹R›);               = {3.389E-06}
m = 1.562‹kg/L›;               = {1562}
v2 = √(3•k•T/m);               = {2.998E-16}
v2/v1;                         = {9.487}
√90;                           = {9.487}

Notice especially the bottom two figures. That is the √90 reduction in the molecular velocity I spoke of above. You see, this is why I think Steven is onto something. This is the key physics question excluding other physical effects on purpose: can an atmosphere as a whole exist with a huge difference in the velocity of molecules across pressure gradients? That is one good question!
I think basically no. Once again, ignoring convection, conduction rules over the radiative transfer, partially due too the short path length. Of course the hot gas near the surface is going to be radiating a great amount of LW but at such pressures the free path length of the photons will be very short, radiatively skipping from molecule to molecule, but even this I want to make that a separate issue. Home in on the velocity of the molecules. Therefore the velocities will always tend to equalize and therefore lower layers will always be hotter per the lapse rate, radiation and convection attempting to counter this to a certain point in a actual atmosphere.
Steven, I think you’ve got something there! Great hypothesis. One of the best new thoughts I have come across in a long time.
Oh, and by the way, the work done on the gases by gravity occured when Venus came to be, except for packets of rising convection and such, there is no work done on the atmosphere, in gravity case anyway. Right?

wayne
May 11, 2010 1:11 am

Mods—
Oh, brother! Why is “PRE” not using a font with UNICODE such as Lucida Console?
It sure didn’t like the tiny brackets!

Spector
May 11, 2010 1:29 am

Ralph: (May 10, 2010 at 4:42 pm) “Nonsense… The temperature of the air at 35,000ft does not suddenly warm to 20oc, just because the atmosphere is not convective that day.”
Not that day, but perhaps 100,000 years later if that condition (no convection anywhere in the atmosphere) persists from then on.

Ralph
May 11, 2010 1:44 am

Further to my last, you would also need an unstable lapse rate to continue the convection. I am presuming that the composition of this atmosphere can lead to an unstable lapse rate – perhaps Steve could enlighten us on that.
.

DavidB
May 11, 2010 2:56 am

Steven Goddard says, in reply to me:
“If you push a piston down into a cylinder, you are exerting a force which directly increases the pressure. That is why the temperature increases. Your claim that “everything else is held equal” is incorrect.”
First, on a point of accuracy which anyone can check, I did not use the words “everything else is held equal”. I used the phrase “other things being equal” as part of a hypothetical fallacious argument which I then showed to be fallacious precisely because other things were not equal!
Second, and more important, I said that the increase of temperature when a gas is compressed is due to the transfer of kinetic energy from the moving piston to the molecules of the gas. Is Steven Goddard disputing this? Surely it is basic physics. The temperature of a gas is the average kinetic energy of its molecules. If the kinetic energy of the gas molecules is increased, that energy must come from somewhere. The only source of energy directly impinging on the gas from outside is the movement (i.e. kinetic energy) of the piston. Does Goddard disagree with this? If so, where else does he think the energy is coming from?
Goddard also says that the temperature increases because the pressure increases. I think that is a misleading way of putting things. The pressure of a gas on the walls of a container is due to the impact of gas molecules. The amount of pressure depends on (a) the number of impacts in a given time and (b) the kinetic energy of each impact. The first of these factors depends partly on the number of particles per unit volume and partly on the velocity of the particles, since fast-moving particles will hit the walls more frequently in a given time than slow-moving ones. An increase in temperature increases both the velocity of the particles and the average kinetic energy of each impact. Other things being equal, this will result in an increase in pressure. To say that the temperature increases because the pressure increases is inverting the direction of causation. It is also possible to increase pressure without an increase in temperature, by putting more gas into the container, or by reducing the volume of the container while maintaining it at a constant temperature by cooling.
In any case, whether we describe the increase in temperature under compression as due to a transfer of kinetic energy or to an increase in pressure, in practice the increase of temperature is always only temporary, as the extra heat is eventually conducted, convected, or radiated away.

May 11, 2010 3:14 am

Spector says:
RE: Ralph: (May 9, 2010 at 11:59 pm) “You don’t need convection for an adiabatic lapse rate . . .”
Apples and Oranges . . . If there are no rising or falling parcels of air then there can be no adiabatic cooling or warming.

Ralph is correct. You get the lapse rate with or without. Deviations from the lapse rate occur, but that is what forecasters used to look for 50 years ago when they plotted temps on skew-T diagrams to determine atmospheric stability and the likelihood of getting your picnic ruined.
stevengoddard says:
The ideal gas law works just fine, and gravity has nothing, nada, zippo to do with the accuracy of the equation.

stevengoddard says:. . .
Gas pressure P = nRT/V . There is no term for either mass or gravity

stevengoddard says:
Mike M [different Mike]
As I have stated about 14 times now, the atmospheric pressure (in temperature ranges where it is a gas) is set by the weight of the atmosphere above it. P is fixed by the weight of the column of air above.

Weight requires gravity.

Jbar
May 11, 2010 3:55 am

Nullius:
“During the Hadean period, an atmospheric pressure of 200 atm has been suggested.”
Sure, I could have suggested it myself. Do you have a source? I’d love to read it. From a scientific paper or book? Not from the blogosphere.
Unfortunately even if there were a proxy for atmospheric pressure in rocks, and I haven’t read that there is, no rocks exist from earlier than about 3 billion years ago, so there’s no way to find data to confirm it.

Jbar
May 11, 2010 4:04 am

Steve G: “On earth water vapor saturates between 0 and 100mbar”
So what’s your point?
The maximum vapor pressure of water in an atmosphere depends on the temperature. If the surface of a planet were sustained at 99C by radiation or GH effect, then the vapor pressure of water at the surface would be 1 bar. I wasn’t talking about the surface of OUR Earth.

Jbar
May 11, 2010 4:13 am

Steve: Your May 10 7:50PM comment about misinformation from the scientific community.
This post is by one person. I’m sure the climate science community consensus does not support the idea of AGW turning the Earth into Venus. Individuals can make mistakes. That’s why we need conferences, peer review, journals, etc. to separate the wheat from the chaff.
There is not enough fossil fuel in the ground to raise atmospheric CO2 more than maybe a couple thousand ppm, and this give nowhere near enough GHE to create a Venus-style runaway GH.

Mike M
May 11, 2010 4:18 am

(May 10, 2010 at 8:12 pm) Keith Minto says: Is this old lava flow?, why are the surface rocks flattened?

Because they aren’t rocks, it is actually an ancient Venusian road. They kept cutting down trees, building structures and roadway turning the whole planet into one big city.
So eventually the entire planet became one big urban disaster, (imagine you keep ending up in Detroit no matter which direction you go). The Venusians starved to death long ago waiting for their welfare checks and all that remains to remind us of them is how they turned UHI into UHP, (Urban Heat Planet).
During the building spree, (financed mostly by carbon credit speculation), a Venusian prophet allegedly said, “This whole damn planet’s going to Hell!”
I have an idea though … let’s send a few gigaplethora pounds of finely ground titanium oxide to Venus and explode it in its upper atmosphere to float down and coat the whole planet. Then more heat heat will be reflected and maybe the planet will cool off enough for us to visit in person some day?

May 11, 2010 4:28 am

Dr bill and George,
I think the energy exchange goes like this. At the exact dry adiabat, rising or falling air is always at the same temperature as ambient, so there is no buoyant force. But if the gradient is less than the adiabat (closer to isothermal) then rising air becomes cooler than ambient. Work is done to make it rise. Falling air becomes warmer than ambient. It too needs a push. Both motions transport heat downward. It’s a heat pump, and the energy comes from the random motions of the air. That’s why air below the dry adiabat is stable to convection. It tends to damp these motions.
The random motions of the air arise from turbulence, driven by the sun’s energy creating temperature differences and hence wind. Pumping heat downward causes the gradient to move towards the neutral dry adiabat.
If the gradient exceeds the adiabat, the air is convectively unstable. The relations are reversed. Air that rises becomes warmer than ambient, and acquires a buoyant upthrust. It now carries heat upward. Motion is created, and heat is moved from warm to cooler. It’s a heat engine.
But again the effect is to move the gradient back towards the dry adiabat.
I’ve been putting these arguments on my blog.

TomVonk
May 11, 2010 5:41 am

Dr. Bill,
As I have been teaching physics too, I hope that you don’t teach that:
Temperature is defined in terms of the average Translational Kinetic Energy of a molecule, and the expression is (1/2)mv² = (3/2)kT, because the former is wrong and the latter a particular case.
The corect definition is that under conditions for the energy equipartition theorem to hold true, the average of kinetic energy per degree of freedom = (1/2) k.T.
For a particular case of a monoatomic molecule (f.ex He) that has only 3 translationnal degrees of freedom we have average (1/2m.v²) = 3.(1/2 k.T).
However for polyatomic molecules, rotational and vibrational degrees of freedom add to the translational and average (KE) = (N/2).k.T where N > 3 that is the general case.
This shows readily in the fact that when one adds the same amount of energy to He and to H2, the translational kinetic energy of H2 will increase much less than the one of He because the additional non-translational degrees of freedom of the H2 molecule will use up a portion of the supplied energy, what He can’t do.
Because details indeed matter, one has to add that even that is no longer true when the energy equipartition theorem doesn’t apply like f.ex in the stratosphere.

May 11, 2010 5:42 am

On Venus, the perfect gas law is only applicable near the top of the atmosphere. In most of the lower atmosphere, temperatures and pressures exceed the critical points for CO2 and SO3 reaching a critical density. SO3 reacts with what little water vapor exists to form H2SO4 aerosol leaving none to react with or absorb CO2 as it does on earth. CO2 on Venus and Mars is not being recycled into oceans.

Spector
May 11, 2010 6:23 am

RE: Mike McMillan says: (May 11, 2010 at 3:14 am) “Ralph is correct.”
My comments did not apply to short-term conditions on earth. If convection *never* happens at any level in a given planetary atmosphere then adiabatic heat transfer cannot happen. Without convection, I believe that this hypothetical and probably unrealistic planet would most likely have something equivalent to the stratosphere going all the way down to the surface. Heat exchange by thermal conduction is, by definition, non-adiabatic.

May 11, 2010 7:04 am

Fred H. Haynie
The ideal gas law doesn’t work perfectly mathematically in any range of temperatures and pressures, because molecules have a finite size. It is always an approximation.
Nevertheless, the sign of the relationships is accurate. When CO2 freezes, the volume can’t decrease any more, so the pressure drops with falling temperature.
P = cT/V

May 11, 2010 9:11 am

DesertYote has it backwards . In a static situation the temperature in a gas , even with a gradient in density maintained by gravity , will come to a uniform temperature ; there will be less energy per unit volume where there is less mass per unit volume .
I just posted this on Lubos Motl’s blog :
I continue to fail to see how the interior of an externally radiantly heated sphere can come to a higher mean temperature than that calculated by StefanBoltzmann and Kirchhoff for its externally observed spectrum given that all heat flow equations are from hot to cold .
If someone knows how to accomplish this trick , please show me the equations . We should be able to duplicate the phenomenon here on earth and solve all our energy problems forever .
The surface of Venus is over twice the ~327k , as calculated at my website , – on both its 244earthDay long day and night sides – calculated for a gray ball in its orbit .
I see no alternative than that Venus’s extreme surface temperature is due to its known vulcanism , perhaps due to solar tidal forces , held in by a very insulative atmosphere .

Mike M
May 11, 2010 11:07 am

Bob Armstrong says: May 11, 2010 at 9:11 am I see no alternative than that Venus’s extreme surface temperature is due to its known vulcanism , perhaps due to solar tidal forces , held in by a very insulative atmosphere .

Those plus maybe a significant amount of radioactive material generating heat. I found this 1982 paper by Sean Solomon and James Head quite interesting.
As I understand the formation of our solar system, the heavier the element – the closer it tended to end up being WRT the Sun. (We got a lot of iron; maybe Venus got a lot of actinium?)

Nullius in Verba
May 11, 2010 11:15 am

Steven Goddard 3:19
“What I am saying is that because of the high pressure on the surface of Venus, adiabatic heating causes the very high temperatures. Those kinds of temperatures aren’t seen on earth because of the lower pressure.”
The surface temperature is a function of the input power, the average height of emission to space, and the adiabatic lapse rate between this height and the surface. It depends on all three. The difference between Earth and Venus is the thickness of the atmosphere, but the mechanism isn’t solely due to pressure. I’m sure you know this, but statements like that above confuse.
8:44/8:51
Regarding Carl Sagan, I already gave a link to a paper by Sagan in which he sets out the adiabatic lapse rate theory of Venusian surface temperature. Evidently he wasn’t always consistent, but it’s only fair to point out that he did know about this.
Ralph, 4:42
“On a fine stable 1040mb day with zero convection and no downward flows there will still be an adiabatic lapse rate in the atmosphere (a dry one in this case). The lapse rate is always present, courtesy of gravity.”
At night, or during a polar winter, there is no solar heating at the ground, the warmer air rises and is not replaced, and you get an inversion layer, in which the adiabatic lapse rate is not followed. Likewise, in the stratosphere the temperature increases with altitude (i.e. the opposite of the adiabatic situation), and there is little to no convection. The atmosphere is stratified – hence the name.
Dr Bill,
5:22
“In the atmosphere, however, there is no external agent doing work on the gases. It is only the dQ part that matters, and that heating must occur at the bottom.”
“Adiabatic” means dQ = 0. Work is done on the gas by its surroundings through the compression as it descends in altitude. It’s just PdV.
“For what it’s worth, I am not trying to be a nuisance to Steve or anyone else, and I am sorry if my comments have created that effect.”
I shouldn’t worry about it. Nobody else does.
Argue for as long as you don’t understand or don’t agree (and feel like it). It’s how blog discussion works.
11:29
“This drag slows the molecules down, and transfers much of the energy they picked up from gravity to the other molecules through which they are moving.”
Are you arguing that gas loses internal heat energy by friction?! Think about it.
Jbar, 3:55
“Sure, I could have suggested it myself. Do you have a source? I’d love to read it. From a scientific paper or book? Not from the blogosphere.”
I assumed you’d just Google for the term. It’s standard scientific terminology.

dr.bill
May 11, 2010 11:18 am

wayne: May 11, 2010 at 1:07 am
Hi Wayne,
Sorry to make you wait so long, but my job thing keeps interfering with my recreation. 🙂
You appear to be a bit miffed by what you interpret as my “tone”. Please keep in mind that I have no idea how old anyone is (not that it matters), nor how much knowledge they possess (which does sometimes matter), so I try to start from a place that seems accessible to everyone, including other readers who might be interested. I wasn’t being condescending.
Anyway, your original question (way back) was about two identical samples of gas where all molecules had identical speeds. What you presented in your table, however, doesn’t seem to be the same thing. You are now (as far as I can tell) allowing things to change, and I can’t see where you have specified a temperature, so I’ll use some sample numbers, and you can have a look at them. Here they are:
T=288K, P=90atm, V=1 liter, M=44g/mol. This gives n=3.74mol, and v=404m/s.
If you now want to keep the volume at 1 liter, the moles at 3.74, and the temperature at 288K, you can’t allow the pressure to change either, because P=nRT/V, and everything on the right-hand side is fixed. The only way to accomplish this is to make your container an insulated rigid box, and in that case, it doesn’t matter whether the new outside pressure is higher or lower, and the molecular speed is still 404m/s. It’s pretty much like being inside of a perfectly designed submarine.
If you want to reduce the pressure to 1atm, but keep the moles and volume fixed, then you have to actively cool the gas to reduce its temperature, and the pressure will decrease to match it. For the numbers I used above, the new temperature would be 3.2K (and we’ll ignore liquefaction), and the new molecular speed would be 42.6m/s. It is certainly true that 404/42.6 = sqrt(90), as you concluded, but in order to get that, you have to allow the temperature to change, and I think that during your substitutions, you didn’t account for that.
/dr.bill

dr.bill
May 11, 2010 11:20 am

Keith Minto: May 11, 2010 at 12:42 am
Hi Keith,
As I said (perhaps flippantly) in an earlier post, it could be the Borg doing something on the surface of Venus for all I know, but from Steve’s graph, it appears that the temperature decreases linearly with altitude in the troposphere of Venus. This signifies convection, and thus I conclude that the energy for this process comes from some kind of heating at ground level, just as it does on Earth. The thing that gets cooled is the Earth’s surface, and the energy has to go someplace, so it ends up in the atmosphere. Other amounts of energy are “dumped” to space by direct radiation from the ground and from the atmosphere, but that’s not the process we’ve been discussing here. In the end, radiation is the only way to get the energy “off-planet”. The convective processes don’t do that. They just redistribute things around the Earth while that process is taking place. It also, helpfully, makes life at the surface tolerable.
/dr.bill

dr.bill
May 11, 2010 11:21 am

TomVonk: May 11, 2010 at 5:41 am
Everything you objected to is included – correctly – in my earlier post to which you refer.
The flaw in your reasoning is in the following phrase:
what He can’t do
In actual fact, Hecan use up just as much energy as H2. The only difference is that the He will convert all of it into translational motion, and thus increased temperture, while H2 will “store” some of it in the rotational modes, and will not increase its temperature as rapidly. Heat capacity – remember?
/dr.bill

dr.bill
May 11, 2010 11:22 am

There were several other recent posts oriented in my direction that I don’t know how to respond to, or which would require more time to think about than I currently have available. My apologies for that, but it can’t be helped.
There was one other note which (as they say) “wasn’t even wrong”. I’m sure you can sort them out.
/dr.bill

Nullius in Verba
May 11, 2010 11:30 am

Here’s a paper on the Hadean atmosphere. “The inception of the oceans and CO2-atmosphere in the early history of the Earth”

Mike M
May 11, 2010 11:36 am

Earth’s core is circulating molten iron that generates our magnetic field. Venus has no magnetic field. That means either that its core is not nearly as electrically conductive as ours OR … the planet has been cooling, and continues to cool, a lot more quickly than earth because of a much higher thermal conductivity between the core and the surface thus explaining its high temperature atmosphere.

Barry Kearns
May 11, 2010 11:47 am

Steven, I have a few questions. I would appreciate it if you would consider each of them deeply and thoroughly before answering.
1. Earlier, you wrote: At temperatures where the atmosphere is a gas, the pressure is fixed by the weight of the column of air. The temperature controls the volume (i.e height of the atmosphere.) That is why the cold poles have a thinner atmosphere than the tropics.
P = T *c/V
This is the most fundamental physics. When the temperature decreases, the volume has to decrease. The volume is area * height. Area is fixed, so the only thing which can change at the cold poles is the height of the atmosphere.

Is it your contention that the poles being cold is what CAUSES the height of the atmosphere to be lower at the poles… because the lower temperature causes that column of air above the surface to become more dense, and thus shorter for the same mass of particles above it?
In short: What do you think is the driving force behind the total height of the atmosphere at the poles being what it is?
What I see as a critical question is this: Why do you think the poles are cold?
2. Earlier, you wrote: With a constant lapse rate, an atmosphere twice as thick would be twice as warm. Three times as thick would be three times as warm. etc.
Do you believe that it would be possible to have two different planets in the same orbit, where the planet with the much thicker atmosphere has a much colder surface temperature? What do you believe would tend to govern the temperatures at the surfaces of each planet?
3. Earlier, you wrote: If you changed the composition of Venus atmosphere to 100% nitrogen, the planet would be much colder due to the lack of absorption of IR. The atmosphere would also necessarily be much thinner (lower volume.) P = nRT/V
However, if you changed the composition of Venus atmosphere to 95% nitrogen and 5% CO2, the temperature and height of the atmosphere would not be hugely different than it is at present. Why? Because the greenhouse effect is logarithmic. After the first few percent, additional CO2 makes much less difference to the temperature.

Do you believe that it is purely a function of the PERCENTAGE concentration of CO2 or other GHG which matters with respect to thermal transfers from the surface of a planet to space, or is it more a question of the ABSOLUTE amount of these gases between the surface and space?
Modifying one of your previous thought experiments, consider the following:
Experiment #5: Take Venus and modify its atmosphere in the following fashion: Incrementally remove CO2 from the atmosphere and substitute N2 such that the total atmospheric mass remains identical. Continue the modification process until the total mass of airborne CO2 on Venus is identical to the mass of current airborne CO2 on Earth. Post-modification, what do you believe would happen to the surface temperature of Venus?

UK John
May 11, 2010 11:48 am

Lets see if I have understood this!
Venus is a different planet to Earth, and on different planets, different things happen!

dr.bill
May 11, 2010 12:15 pm

Nullius in Verba: May 11, 2010 at 11:15 am

Dr Bill,
5:22
“In the atmosphere, however, there is no external agent doing work on the gases. It is only the dQ part that matters, and that heating must occur at the bottom.”
“Adiabatic” means dQ = 0. Work is done on the gas by its surroundings through the compression as it descends in altitude. It’s just PdV.
11:29
“This drag slows the molecules down, and transfers much of the energy they picked up from gravity to the other molecules through which they are moving.”
Are you arguing that gas loses internal heat energy by friction?! Think about it.

Hi Nullius (if I may be so informal!),
Yes, I understand what adiabatic means, but what I was talking about was what happens in the rising gas, which is what’s doing work on the things it displaces as it rises, but without heating them or being heated by them. That’s what determines the lapse rate. The rising “package” does, though, have to get heated by the ground in the first place or it will not start rising to begin with. The adiabatic part is for everything that happens above the surface skin.
The return trip is another story. For one thing, the rising gas doesn’t come back down in the same place. There are lateral movements, turbulence, entrainments, and a lot of bouncing and mixing going on, which serve to homogenize things. This isn’t friction in the usual macroscopic sense, but it does siphon off energy from anything that gets going “too fast” and redistributes it to other molecules via collisions. In most cases, but not all, it eventually settles into a “sort of” Maxwell-Boltzmann distribution with a superimposed downward drift. If this were not the case, we would have high-velocity winds coming straight down at us all the time.
And thanks for the comment about “thickening my skin”. I’ll work on that, but I don’t like to be abrasive unless absolutely necessary. When I get to that point, though, I sometimes have “gasket issues”. 🙂

George Turner
May 11, 2010 12:27 pm

Barry Kearns, in experiment #5 I anticipate a surface temperature increase of about 100 to 150 kelvins because the lower molecular weight of N2 compared to CO2 means replacement on a mass basis results in 1.57 times as many molecules, which increases atmospheric volume (and thus height).
So even though the dry adiabatic lapse rate of N2 is slightly lower than CO2, the taller atmosphere would have about a 40% greater temperature difference between the surface and the cloud tops, giving a surface temperature of about 850 K.
It would still have the clouds of sulfuric acid and other bizarre chemicals, plus the dust, so I’m not sure how the IR picture would change.

George Turner
May 11, 2010 12:31 pm

dr. Bill
You said:
The return trip is another story. For one thing, the rising gas doesn’t come back down in the same place. There are lateral movements, turbulence, entrainments, and a lot of bouncing and mixing going on, which serve to homogenize things.
That’s why the Marines hate riding the dropship down to a terraforming planet for a bug hunt, man! It’s the express elevator to …

May 11, 2010 12:34 pm

@ Mike M
Interesting comments & link . I just mentioned the tidal forces as something which has brought Venus to slow retrograde motion . ( Wikipedia seems to give a rational description which it certainly fails to do for the basic calculation of a planet’s temperature based on StefanBoltzmann & Kirchhoff . )
Perhaps Venus’s lack of a magnetic field has more to do with its lack of rotation rather than a solid core .
Good , but no longer surprising , to see a number of people recognize that the only way the interior of a sphere can remain hotter than its surface is if it has an internal heat source . Adiabatic heating explains nothing because it’s transitory . The profile of atmospheric pressure caused by a planet’s gravity is essentially static .
I actually have been censored from “warmist” blogs because of my “obsession” with understanding the quantitative physics and proposing the “absurd” notion that Venus might have an internal heat source . Easier , I guess to just claim that heat can be made to go up hill and stay there .

Nullius in Verba
May 11, 2010 12:55 pm

Dr Bill,
The informality is fine.
If you look at the diagram for Hadley cells, you can see the air coming down over the desert latitudes. But yes, turbulence is more complicated. It siphons off the bulk motion of convection into heat. But the way you had written it, it looked like you was talking about the heat energy itself – the adiabatic temperature increase resulting from gravitational compression of descending air.
It would be fair to say that it would be moving slower and in a more disorganised fashion, but the kinetic energy of the molecules could be just as high, and it wouldn’t need reheating. In fact, since it has just moved to where the ground is cooler, it would tend to transfer heat to the surface, before flowing over the surface back to the tropics. Heat is transferred from the hot tropics to the cooler temperate zones, where it is dumped; the temperature difference driving the heat engine.
In fact, it does cool overall on the cycle, but by radiation to space from higher altitudes, rather than by friction.
I was pretty sure it wasn’t what you meant, but most of this conversation appears to be two people in violent agreement, both misunderstanding what the other intends.
Like you, I like to get the details right, even when I agree in general.

May 11, 2010 1:11 pm

I found a link to the 30-year old Dragon of Eden that Steve seeks to slay:
Per the request of the website owner, I’m not quoting,
but I urge the reader to consider each paragraph fully
http://users.tpg.com.au/users/mpaine/Cosmos_greenhouse.html
Also, Charlie Rose in his last interview with Sagan,
http://www.charlierose.com/view/interview/5787

George Turner
May 11, 2010 3:16 pm

I have a question on the spectral widening of CO2 absorption bands under the conditions found on Venus, which several people here have pointed to as explaining the potent CO2 GHG affect on that planet.
Doppler widening (due to temperature) at the surface of Venus should make the spectral lines only about 50% wider than CO2 at Earth’s sea level temperature, which means pressure widening of the spectrum must be the explaination.
CO2 has absorption peaks at 2.6 and 4 microns, yet the VIRTIS imaging spectrometer on Venus Express can see all the way to the surface at 2.3 microns, just 0.3 microns away from a CO2 absorption peak.
So to me it looks like there can’t be much widening of CO2’s spectrum.
Also, VIRTIS can see pretty well from 2.2 to 2.5 microns, just 0.1 microns from the CO2 peak.

dr.bill
May 11, 2010 3:38 pm

Nullius in Verba: May 11, 2010 at 12:55 pm
Hi Nullius,
What you say perfectly valid, and I was, in fact, being a bit sloppy and simplistic in an effort to keep my original point from getting lost in tangential issues. Mea culpa ex animo.
I also liked the following:
I was pretty sure it wasn’t what you meant, but most of this conversation appears to be two people in violent agreement, both misunderstanding what the other intends.
The curse of our species sometimes! 🙂
/dr.bill

Bob_FJ
May 11, 2010 3:45 pm

dr.bill Reur May 10, 2010 at 6:30 pm
Concerning S-B as it might apply to a body immersed in a fluid. (such as the Venus atmosphere);
Thankyou for responding to the first of several considerations I sequentially listed.
It would be good if you could also advise on step b} that is repeated below, in order to try to explain what might be happening on the surface of Venus.
b} If a dry body is immersed in say an Earthly thin gas, [it] having a lower temperature, there will not only be EMR emission resulting in net HEAT loss, but also HEAT loss via conduction and convection. Where does this “extra energy” come from without upsetting the balance of the EMR? Or, to put it another way, the surface molecules are busy losing energy as EMR, (photons), and as HEAT, (KE). Can they do that without affecting each other?
~~~~~~~~~~~~~~~~~~~~~~~~~~~~~~~~~~~~~~~~~~~~~~~~~~~~~~~
BTW, you appear to be saying that wet water ice will emit twice as much EMR as dry water ice. (at its freezing point). I find that to be surprising because there is intimate contact between the water and ice such that I would have thought that the water would become the radiating surface of the body. Consider for instance painted concrete. Does the concrete still send photons through the paint laminate? From my limited reading, it would be phonons from the concrete, (which cannot radiate through space), not photons. I ponder too; dry, moist, (meaning water has penetrated), and wet concrete.
You’re right, it’s quite a headache for me.
Can you recommend any text books or studies that consider immersion of bodies in a fluid, WRT S-B?

George Turner
May 11, 2010 4:14 pm

And a follow up on the clear IR window from 2.2 to 2.5 microns.
VIRTIS for those interested.
I calculated that a 740K blackbody would emit 799 W/m^2 in the open window from 2.2 to 2.5 microns. That’s more than the solar energy Venus receives at the surface.
check please?

Bob_FJ
May 11, 2010 4:30 pm

GEOTHERMAL activity; Here is an interesting article:
The Baffling Geology of Venus (TWO PAGES)
http://geology.about.com/od/venus/a/aa_venus.htm
It links to some essential reading stuff from the ESA where there is speculation that she is active.
(BTW, reportedly, one of the Soviet probes may have passed through a volcano plume, according to its “sniffer“)

dr.bill
May 11, 2010 5:37 pm

Bob_FJ: May 11, 2010 at 3:45 pm
Hi Bob,
I’ll do things in reverse order:
– I don’t know of any reference that explicitly deals with the radiation phenomena of objects immersed in fluids. I’ll hunt around and see if I can find something, but I’m not all that hopeful about it.
– I wasn’t saying that the (ice + melted water) emits twice as much radiation, just that each one emits some, and that the result would be the total of the two. I ignored the complication that the fluid might not be transparent to the radiation from the internal solid. If that is important, then the emission, absorption, and re-emission process needs to be included. On Earth, and treating the atmosphere as your fluid, that does need to be taken into account because the atmosphere initially absorbs some of the outgoing radiation from the solid surface.
– In the case of the painted concrete, the photons don’t enter into the picture until you get to the paint layer. Before that, the energy transfers are mainly by conduction, and since phonons are a way to describe group vibrations of the atoms in the solid, you could use that approach to deal with things inside the concrete, but photons don’t get emitted “through” the paint. They are, in essence, created “by” the paint. The difference between paint on the one hand, and atmosphere or fluid on the other, is the matter of transparency. If you had a thick layer of see-through paint, you’d be back to something like the solid plus fluid situation. If not, then only the surface temperature of the paint is important for the radiation distribution pattern.
– Regarding your item (b), keep in mind that conduction requires physical contact, and convection requires “large-scale” movement. If we’re still talking about things like planets, neither process can result in energy loss to space. Only radiation can do that. The conduction and convection processes move energy around inside of the system, mostly conduction in solids, and convection in gases. Liquids are somewhere in between.
– When “thermal photons” are emitted, some energy of motion is lost by the molecules, thus reducing their Kinetic Energy. I think the glitch in your thinking is not noticing that these are not two different processes. One is simply the result of the other, and I believe that you might be “double-counting”.
– One more point, and this is perhaps just me being picky, is your use of the word HEAT. Strictly speaking, there is no such thing as heat, even though it is perfectly fine for everyday use, and generally causes no confusion. In Physics, however, using this term causes lots of unintended errors. Properly speaking, heating and cooling are processes, verbs, not nouns, that involve conduction, convection and radiation as means of transferring energy from one place or thing to another. The word heat (as a noun) is something left over from our earlier misinterpretations of energy and temperature and related things. (Look up ‘caloric’, ‘frigoric’, ‘phlogiston’ and ‘anti-phlogiston’.)
/dr.bill

May 11, 2010 5:56 pm

George Turner,
Are you reading that Virtis link right? I couldn’t see a reference to that window range. From the spectrum (Fig 1), that seems to be the range of CO2 absorption (confirming the greenhouse effect). I think the window they refer to is the narrow peak actually at 2.35 micron.

George Turner
May 11, 2010 7:21 pm

I think I’m reading it right. The window peaks at 2.3 microns but looks to average about 50% of that across the 2.2 to 2.5 micron range. This was apparently unexpected and discovered by accident during Earth-based observations that showed structures at 2.2 microns.
However, its normalized units.
*Digs deeper*
More on VIRTIS including a spectrum with actual units, but I’m inclined to think those are the units for the detector (receiving about 13 microwatts on a 1 cm^2 cell.
If CO2 was active (and broad spectrum) in this region there’s no way they could detect the trace gases.
I also ran across a VIRTIS link on vulcanism that indicated the conditions on Venus’ surface produces rocks with very low IR emissivity, which is another interesting avenue to investigate.

wayne
May 11, 2010 7:27 pm

Hi dr.bill (Steven, you too if you can help answer this):
Didn’t want to put you on the spot, I know you have no idea who or what age you are conversing with initially. (I too have had troubles with a couple of redheads in the far past! ;))
Back on Venusian atmosphere, I don’t know how to convey this thought to you but to ask the question and have you ponder on it for a while.
Take a quiet and stable atmosphere on any planet with any one type of molecule as its gas, no convection, no rotation, no winds, purely hypothetically quiet but with proper physics. Looking at the atmosphere as concentric shells of thickness dz, can the molecular velocities vary from shell to shell to shell, and if they can vary, what force is maintaining the velocity differential. The gravitational pressure gradient is going to be there no matter what. Also, it’s quiet, there is no ‘work’ being done on the gas.
That was all I was trying to get to above. One part of me says no, they (the velocities) would equalize over time and stay in a equilibrium. That would mean it would always be hotter the deeper you go, the lapse rate, getting just enough radiation input to maintain the temperature. The other part of me says yes, by TD gas equations, the velocities must be somewhat different to satisfy the temperature and pressure differentials seen on Venus but I cannot seem to find why, what force would be doing that. That all ties into why there is a lapse rate in the first place. Get my gist now? It’s a rather deep question. To properly answer it I think you have to go a bit underneath to the basic properties supporting the statistical nature of thermodynamics (½mv²) which underlie temperature and pressure themselves.

May 11, 2010 8:25 pm

George Turner says:
May 11, 2010 at 7:21 pm
I think I’m reading it right. The window peaks at 2.3 microns but looks to average about 50% of that across the 2.2 to 2.5 micron range. This was apparently unexpected and discovered by accident during Earth-based observations that showed structures at 2.2 microns.
However, its normalized units.
*Digs deeper*
More on VIRTIS including a spectrum with actual units, but I’m inclined to think those are the units for the detector (receiving about 13 microwatts on a 1 cm^2 cell.
If CO2 was active (and broad spectrum) in this region there’s no way they could detect the trace gases.

I ran CO2 spectra at ~90 atm and there’s a transmission window from 3840^cm-1 to 4600^cm-1, so yes there is a narrow window.

Keith Minto
May 11, 2010 8:27 pm

wayne says:
May 11, 2010 at 7:27 pm
That all ties into why there is a lapse rate in the first place.?

Wayne, that is the crux of the whole discussion and this has puzzled me even before this thread started. The question pondered is, when temperatures are being measured, what exactly are we measuring?. In space, in view of the sun, radiative photons on the sensor, in shadow, photons from background cosmic radiation. In water, an intimate contact between the sensor and the fluid. But what about air ? the ability of the sensor to measure temperature accurately is a function of the efficiency of kinetic energy to be transferred to the sensor, so, the more dense the air the more efficient this transfer is, and the temperature reading is higher. There is a gradient of efficiency as the sensor goes higher, less molecules hitting the sensor. Standardize the pressure, then read temp. at different altitudes and see what the difference is. My guess is that the gradient is nowhere as steep as the standard wet/dry lapse rate. The lapse rate increase in temperature with decreasing altitude is a artefact of increasing air density.
Until your question is answered “why is there a lapse rate in the first place” this argument is circular.
My proposal is not in any textbook that I know, and I welcome feedback if it is, but it is testable.

Spector
May 11, 2010 9:40 pm

Understanding the Tropopause
The tropopause is that altitude where convective heat transfer stops and the lapse-rate changes from a typical adiabatic decreasing temperature slope, typical of the troposphere, to the increasing temperature slope, typical of the stratosphere.
One thing, I think, that appears to be missing from our thermal radiation knowledge base is a good understanding or theory for radiation from a transparent medium that contains a sprinkling of internal radiation sources. I would assume the sources were in general thermal equilibrium with their containing medium. I note that if the active radiation layer of the Earth’s tropopause were 3 km thick, it would only take an average volume radiation power of 78.33 milliwatts per cubic meter to achieve the nominal 235 W/m2 radiated power level that some sources say is required balance the Earth’s radiation budget.
From one source I see that the typical temperature of the tropopause may be on the order of minus 55 degrees C. That is almost like a negative greenhouse effect [or an ice-locker effect] of 37 degrees relative to the minus 18 degrees C required to produce a surface flux of 235 W/m2 for the Earth.
Perhaps with a better understanding of how the tropopause unloads convected heat from below, we could more readily determine if the surface temperatures on the Earth and Venus are, as I suspect, controlled or regulated by the tropopause temperature (plus the lapse-rate span) or if the tropopause temperature is controlled by the surface temperature (minus the lapse-rate span) or if, perhaps, it were some intermediate average of these two alternatives.
I note that the Venus tropopause temperature bottoms out around -112 degrees C at an altitude of 100 km and on Earth the tropopause ranges between 11 km at the poles to 17 km at the equator where it might get as cold as minus 80 degrees C. As temperatures of these two planets are more similar at their tropopause altitudes than at their surface levels, I am more inclined to believe that surface temperatures are controlled or at least limited by tropopause conditions. Perhaps, in the case of the Earth, the higher tropopause temperatures are a result of more direct radiation from the surface.

John A. Marr
May 11, 2010 10:00 pm

I really don’t see why Mr Goddard’s ideas should be so controversial except, of course, because they question the doctrines of the AGW true believers.
Anyone who has experienced the sudden arrival of the warm Chinook wind on an otherwise frosty day east of the Rockies can attest to the great power of adiabatic warming.
They have the same kind of wind north of the Alps called the Föhn which is also the German word for hairdryer. As I explained to my grandchildren just the other day, hairdryers, like Chinooks, work on adiabatic principles, with the fan compressing and thereby heating the air, rather like the atmosphere of Venus.

George Turner
May 11, 2010 10:44 pm

Actually hairdryers don’t compress the air, they just accelerate it like a propellor or fan. To compress the air they’d need stator vanes, and to compress it significantly they’d need lots of paired rotors and stators. Hair dryers generate hot air with electrical heating coils in the direct flow of a high speed fan, which both cools the coils and blows the heated air toward a scalp.
🙂 <— understands many common household appliances.

May 12, 2010 12:11 am

Spector
“That is almost like a negative greenhouse effect [or an ice-locker effect] of 37 degrees relative to the minus 18 degrees C required to produce a surface flux of 235 W/m2 for the Earth.”
The arithmetic goes like this. About 40 W/m2 outward IR comes from ground through the atmospheric window (unabsorbed frequencies), at about 288K. Another 60W/m2 approx goes out from cloud tops, also fairly warm. The rest comes from varying altitudes, with about half from the tropopause at about 225 K. That all adds up to an average 255K.
You can see this reflected in the integral under the OLR spectrum.

just me
May 12, 2010 12:59 am

A. Marr
“Anyone who has experienced the sudden arrival of the warm Chinook wind on an otherwise frosty day east of the Rockies can attest to the great power of adiabatic warming.”
but, that is another effect. Moist air is going up the mountains, cooling on its way, but water is condensing, thus, the lapse rate is a wet lapse rate, around 6.5K/km, now, on the other side of the mountains the now cool, dry air is sinking with dry lapse rate, which is around 10K/km. Thus, you convert latent heat into sensible heat, IMHO. Of course, there are also other explanations, but this is the common one.
Example: you have west wind. Moist air from Pacific or so is going up the Rockies, water is condensing, snow, rain, etc. The now cool,dry air reaches the top of the mountains, and is going down to Calgary. Chinook.

dr.bill
May 12, 2010 1:49 am

Hi Wayne,
I think we had better knock off with the redhead stuff before we get clobbered by Pamela Grey or someone! Yeah, I know, I’m a wuss… 🙂
Anyway, I’ve got your model for you (Wayne’s World ?? – sorry!), but you have to stop working me so hard! On the other hand, I might get a bonus from the exercise. I’m giving a TD test on Friday, and I’m thinking of putting this on as a question in the atmospherics part. Hope none of them are reading this thread. 🙂
So then, if you don’t mind, along with your no-convection stipulation, I simplified things a bit further by irradiating your planet uniformly from all directions, and I gave it a transparent atmosphere. That just leaves conduction processes acting, and you can’t avoid those. What you get is essentially an atmosphere that is one big troposphere, right up to the “top” (which isn’t, of course, precisely defined – it just fades away into space).
Under these conditions, the TD processes are pretty much the same as keeping one end of an iron bar at a high temperature and the other end cold, except that the bar doesn’t have a finite length, and the low temperature is the background temperature of space, or pretty much 0 kelvins for our purposes. Because there’s a hot end and a cold end, however, you do get a temperature gradient, just like with an iron bar, so the question is how to get a fix on what that should be. We could do it by working with the thermal conductivity of air or other gases, but I decided to “cheat” a little bit for the sake of expediency.
In regular analysis of the troposphere, we use a linear function T = To – Ly, where L is the lapse rate (which is just a regular thermal gradient, regardless of what it’s caused by, or what its value might be). An alternative is the Isothermal Approximation. This leads to what people call the Barometric Equation (for pressure), which is a decreasing exponential. It works surprisingly well, despite the rather bad assumption that the surface temperature persists right up to the tropopause. The lapse rate version is much better, but we can’t use either of them for your model because they give “stupid results” once you get a bit above the tropopause.
What has to happen is that the gradient will be largest when close to the hot end, and will stay pretty much constant for some distance, just like the iron bar and our own troposphere, but will eventually diminish because of the “infinite length”. It turns out that there’s an easy way to get something just like that, by re-writing the temperature equation given above, and then treating it as the first two terms in the expansion of a decreasing exponential. In other words: T/To = (1 – Ly/To) becomes T/To = exp(-Ly/To). The quantity L/To is quite small (about 1/30 or 1/40 km or so), which makes this a good mathematical approximation, and it guarantees that we will also match “normal behaviour” for things like iron bars and tropospheres.
All you need after that are the Ideal Gas Law and the Hydrostatic Equation: dP/dy = -ρg, and the rest is just a matter of solving one differential equation. Here are the results, which you can stick into a spreadsheet and do some plotting. I’ve normalized everything, so all the values either start at 1 and end at 0, or the other way around in the case of the cumulative mass function (the total mass M below a given altitude).
T/To = exp(-Ly/To)
P/Po = exp(-(mg/RL)(exp(Ly/To) – 1))
ρ/ρo = (P/Po)exp(Ly/To)
M/Mtot = 1 – P/Po
For “play purposes”, I’d advise putting To, L, and m (the molar mass) in separate cells somewhere, and use them as parameters in your cell formulas instead of hard-coding them. Then you just change any one of them and all the values and the graphs will adjust by themselves.
Just for reference, here some of the values you should get if y is measured in km:
L = whatever you like (5 to 10 K/km is typical in our atmosphere)
To = whatever you like (but 288K is “average” for Earth)
mg/R = 34.157 K/km for air, 51.91 for CO2, etc…
By the way, those results above give a dead match to the International Standard Atmosphere profile for the troposphere if you use 6.5 for the lapse rate and 28.96 for the molar mass, so in that sense, you can use them for “semi real world” stuff as well as just playing with them. Let me know how it works out, but you may run the risk of getting an overtime slip if you keep slave-driving me like this. 🙂
/dr.bill

Jbar
May 12, 2010 3:33 am

I gotta hand it to ya, Steve, you sure do generate lots of comments.
What’s the duration record for a thread on WUWT. How long do we have to keep going to beat it?
Reply: You’ve got another 1228 comments to go to beat this post. ~ctm
[Actually, the this article is within less than 400 posts of the Climategate article. -Moderator]

dr.bill
May 12, 2010 4:44 am

Hi Again Wayne,
I forgot one formula in my previous post, and actually it was the one that you seemed most interested in, namely the molecular speeds. My bad! So here’s the addendum:
v/vo = exp(-Ly/2To), where vo = sqrt(3RTo/m)
vo is the speed at ground level, and as you can see, because the temperature decreases with altitude, then so does the speed of the molecules, but not nearly as quickly because of the square root.
Using O2, for example, with To = 288K, gives vo = 474m/s. At an altitude of 10km, the speed has decreased, but only to 423m/s (using L = 6.5K/km), which is a much smaller decrease than for T or P or ρ.
/dr.bill

Jon-Anders Grannes
May 12, 2010 7:08 am

Well i had some comments here earlier, have they been removed?
Venus atmosphere I guess absorb the short wave light from the Sun, like the Earths oceans do. And when Venus atmosphere is isolated/insulated 90 times more than Earths atmosphere, the long wave I.R. has a much much longer way to go and thus stays longer in Venus atmosphere. In addition there is no or little water in Venus Atmosphere to make clouds to give negative feedback on the climate so Venus atmosphere is therefore mostly extra crispy?
And this is what makes Venus atmosphere warmer?

wayne
May 12, 2010 10:28 am

dr.bill says:
May 12, 2010 at 4:44 am
I forgot one formula in my previous post, and actually it was the one that you seemed most interested in, namely the molecular speeds. ..
Thanks Bill for the info. That’s might help. I also found the proper equations which bring density into the temperature calculations, I just knew it was there somewhere. Take a look at my question at May 11, 2010 at 7:27 pm later and see if you come up with the same. I can only account for about half of the dT at molecular level. With equal velocities the temp should be much higher at surface, or conversely, lower at high altitudes. That is why it seems to be top driven, not from the surface.
My wild guess is that what is being shown is most of the incoming radiation energy is being absorbed by high levels in the atmosphere and re-radiated back out to space, never making it to the surface keeping velocities higher always greater. This would be in a smooth distribution vertically of coarse. Look closely at Venus’s lapse curve, its convex while Earth’s is concave. Is this because our atmosphere is much more transparent?
(And the numbers I posted near the top, not only was the formatting trashed but most of the calculations are trash also. Never focused on the central values before pressing submit, was only focusing on the √90 factor. But I caught and fixed them. Oops! Patience would help. That is why I said earlier that I am not totally versed in TD, I still make such mistakes.).
Later.

wayne
May 12, 2010 10:34 am

dr.bill, to be clearer you might change:
Is this because our atmosphere is much more transparent?
to read
Is this because our atmosphere is much more transparent and therefore mostly bottom (or surface) driven in contrast to Venus?

wayne
May 12, 2010 1:35 pm

dr.bill,
Going call this an end for a while. You have given enough for me to digest for some time! Thank again.
Also, I will adjust the 3 factor in the equations depending on the atom or molecule being assumed as the gas as degrees of freedom vary, I was following that all along. Now just need to numerically integrate to see if I can get a little clarity if possible.
Still, I will always wonder on the avg. molecular velocities in a gradient density environment, as a gravity held atmosphere, how could they possibly be different in a mostly opaque, dense, and smoothly radiated atmosphere such as Venus, intriguing.

dr.bill
May 12, 2010 2:41 pm

re: wayne: May 12, 2010 at 10:28 am
and wayne: May 12, 2010 at 10:34 am
Hi Wayne,
One thing you should note is that I (personally) don’t think Venus is “top-driven” – except in the sense that the energy which heats the surface of the planet likely comes in at the top in the same way that sunlight coming into the Earth’s atmosphere heats the surface of our planet. After you get a warm surface, however, no matter how you get it, and as long as it is sustained in its warm condition, there isn’t a whole lot of mystery about the generalities of what follows. Details, sure. Big picture, no.
For a column of gas in a gravitational field, more weight will be pressing down on the molecules at the bottom than on those higher up, so there will be a pressure gradient. This is adequately described in a normal atmosphere by the equation: dP/dy = -(mg/RT)P. In order to solve this equation, you need to know how T depends on y. In the tropospheres of Earth and Venus, temperature decreases with altitude. Once you know the details of that variation, the rest is just Math, and what you get are decreasing values for Temperature, Pressure, Particle Density, and Particle Velocity as you move upwards.
Those statements hold true for all the predominant normal conditions that prevail in the atmosphere, and I don’t want to get side-tracked onto ephermeral things like Chinooks and temperature inversions, ash clouds, or any other short-term and geographically-isolated phenomena. They occur. There are reasons for them, but they are not the normal day-to-day behaviour of the atmosphere, which is what we, or at least I, have been focused on. The force holding up the molecules isn’t mysterious either. It’s the same kind of buoyancy described by Archimedes Principle. It’s what holds up the Goodyear blimp.
The model that I worked out for you is the simplest one that I could think of, since the only active ingredients are gravity and a warm surface, with only contact conduction for energy transfer, and everything else removed. Even under such simplified and minimialistic conditions, however, all of the general variations that I described above hold true. Plot the graphs. Look at them. The details would change if a slightly different variation of T with y were used, but not the big picture.
In short then, I have a feeling that you might be looking for the answer to a couple on “non-questions” in the matter of particle velocities and forces. I’m not saying that these aren’t involved in the processes, but I also don’t think there’s anything odd about them. When temperature decreases, particles slow down (the other way around, actually, but I won’t get picky). When particles sit on top of each other, buoyancy supports the higher ones. I don’t see any mysteries in this.
/dr.bill

dr.bill
May 12, 2010 2:51 pm

re: wayne: May 12, 2010 at 1:35 pm
Hi Wayne,
Our last posts seem to have passed in transit.
One final item regarding the “model”: in addition to letting the lapse rate, surface temperature, and molar mass change, you can also change gravity, so it would be convenient to leave those as “settable” parameters. I hope you sort things out in your contemplations, and if I have been of any small help, it has been my pleasure.
All the best.
/dr.bill

Bob_FJ
May 12, 2010 3:46 pm

dr.bill, Reur May 11, 2010 at 5:37 pm
Concerning S-B emission from a body immersed in a fluid, (maybe such as the Venus atmosphere); thanks for your comments Bill.
In the analogy of painted concrete, you say that there would be emission ONLY from the paint, assuming the paint to be opaque. On the other hand, if the paint were hypothetically transparent, the concrete would then emit through the paint unhindered. It is thus apparent that if immersed in a fluid, the amount of radiation from a body is dependent on the level of opacity in the fluid. (considering also your earlier comments on wet versus dry water ice)
On Venus, at 92 bars, and with mostly CO2 in intimate contact with the basaltic rocks, that fluid is obviously very opaque to infrared, apart from some small windows in the shorter wavelengths.
Thus, whilst S-B may be “near enough” on planet Earth, with only its trace levels of GHG’s, it would seem to me that its application on Venus’s surface is badly compromised. (without addition of some other theory)
Furthermore, I would think that most of the surface HEAT loss is via conduction/ convection/ advection. Whereupon, at the higher altitudes, that HEAT is more able to convert to EMR and escape to space.
~~~~~~~~~~~~~~~~~~~~~~~~~~~~~~~~~~~~~~~~~~~~~~~~~~
There are other points in your comments to discuss, but right now, I don’t want to distract from the important considerations above

George Turner
May 12, 2010 4:15 pm

re Phil: May 11, 8:25
I ran CO2 spectra at ~90 atm and there’s a transmission window from 3840^cm-1 to 4600^cm-1, so yes there is a narrow window.
I calculated that 799 W/m^2 could be emitted into that window (blackbody), and since parts of the window are used to image the surface from orbit, some of the radiation is making it directly into space. Other parts of the window are being used to look at constituents of the lower atmosphere, so CO2 emissions (and thus absorption) aren’t occuring there.
Even is the surface emissivity is 0.25, that would still be 200 W/m^2 radiating up from Venus surface, which is more radiation than the surface receives from the sun. If the surface has this negative radiative flux then its temperature is being maintained by atmospheric convection/conduction (or it requires that as supplemental energy).
If this is the case then the surface radiation exchange (ingoing and outgoing) is cooling the atmosphere, not warming it, and I would think the surface temperature is being maintained by the rest of the atmospheric dynamics.
However, I don’t have hard numbers on the fluxes.
As a thought experiment I could imagine that the sunlight shifted totally to the blue end of the spectrum (which doesn’t reach Venus surface) but maintained the same overall radiative flux, being totally absorbed or reflected by the cloud layers. Then there would be no downward flux in the visible spectrum at the surface, ensuring a net negative radiative flux at the surface, but I’d bet the overall atmospheric temperatures would stay largely unchanged.

paul in UK
May 12, 2010 4:24 pm

So many comments it’s difficult to follow the scientific discussion in just a few minutes, so I’ve probably missed something, but I’m struggling with the basic theory here regarding pressure and temperature. I probably covered some of the relevant physics at school/uni, but that was a long time ago now and I’ve not had to apply it to this sort of scenario so presumably I’m missing something or got my logic wrong?
I’m imagining trying to model a hypothetical simple atmosphere where all we need for the model is PV=nRT. So, if air is rising, the pressure (P) drops, but I don’t understand why in this simple model temperature (T) drops. Would not P1xV1 = P2xV2? i.e. the gas is free to expand as it rises and the pressure drops; i.e. the volume (V) increases? Isn’t PV a constant; as P drops, V increases, therefore P1V2 = P2V2 therefore no change in T? If V did not change, then I presume T would drop, but presumably V is increasing, that’s why the air gets thinner with altitude?
I haven’t studied any of this properly, so probably wrong, but (and as a frequent visitor to mountains I’ve often pondered climate and temperature differences at different locations and altitudes) I was thinking the reason why temperature drops with altitude is not due to pressure, but (assuming absorption of radiation is relatively small in comparison) imagined that the main heating effect came from the sun heating the planet’s surface (rather than the atmosphere directly), and this in turn heating the atmosphere by conduction and convection, so the further away (higher up) from the surface (i.e. distance not pressure), the less heat the atmosphere has picked up. Thus explaining why when I go to Norway in June I can camp at 3000ft on one of the large plateaus and sit outside my tent in shorts anytime it’s sunny (with temperatures subjectively comparable to or warmer than the valleys 3000ft below), but if I camp at 3000ft on a Scottish Munro it’s far too cold!! ie. when on a plateau the breeze I’m getting passed close to lots of ground heated by the Sun, even though I’m at 3000ft, but on a Scottish peak, surrounded by land and valleys 3000ft lower the air passing me has not been close to much sun warmed ground.

wayne
May 12, 2010 6:43 pm

re: dr.bill: May 12, 2010 at 2:41 pm
Hi Bill, one last post.
We did pass in the posts. I looked later and said, that wasn’t here a while ago. Well, that happens in blogs.
I did notice on one of your first post you mentioned you understanding that radiation was heating the surface and the heat moves upward in the atmosphere. That is where I diverged; because of a new thought that had entered my mind.
Due to a lapse rate, the lower one km is warmer ‘by a thermometer’ than the km layer above it. I asked myself (and I’m not speaking of radiation, that is another separate topic), does the heat energy in the lower and hotter layer tend to warm the layer above by conduction. After much thought I think not. Why? It’s hotter by a thermometer. TD says the heat will move but it doesn’t. Why? The molecules must have the same mean velocity. That is where I started speaking of the molecular velocity and the density differences. You can’t even get that to happen in a lab here on Earth. Only a gravity held atmosphere has this aspect. In a lab gases at different densities will instantly equalize, in an atmosphere, they won’t, vertically that is.
Whoa was my response to myself. I had never thought of temperature, pressure, density and thermometers in that manner. If that heat will not and cannot migrate upward by conduction, then the molecules, on an average (actually a Boltzmann distribution I think, maybe Maxwell), must have the same mean velocity, that is why the ‘heat’ does not move by conduction. Whoa, that IS a lapse rate. But I try to doubt myself a first, and second, and a third time. But that one aspect seemed correct even though that basically goes against normal thermodynamic principles when loosely passed in conversations about normal substances here on the ground. It must have to do with the density gradient by gravity.
The big thought is that if the above is possible and actually true then in an atmosphere, due to the density gradient of the gases, energy can pass from a cooler to warmer area by conduction as long as the warmer area is also at a greater density and only to a certain degree. Solving for v using (½ ρ • equal_vol • v_rms2 = 3RT / 2, I think 5 not 3 for co2) on each area will tell you if heat can move by conduction alone. The molecules of the cooler layer must actually have greater avg. velocity than the molecules in the warmer layer, density differential does the counter-intuitive magic. That is why I said you would have to go underneath normal TD to the equations underlying. New thought, right? So Venus type of atmosphere could be totally or partially top driven, heat moving down even though that is initially thought impossible by common TD jargon.
I could be totally wrong on this and I have before. After while with the relationships you have supplied above I’ll probably prove myself wrong, but that is science, right, exploring new thoughts, new ideas, new ways to look at the same thing from a different aspect.
If I try a model, I am well versed in gravitational aspects and will leave it variable, in fact, integrated as the height in the atmosphere varies. Have programmed for decades and one area was ephemeris software for near-exact positions of the planets and asteroids. (solar system simulation). I have always had a deep interest in astronomy and later physics.
There is another aspect in Venus’s atmosphere that has a big factor in internal re-radiation and scattering since it is so very thick at >60km. The dip of the horizon. At 50 km a much greater portion of the radiation, randomly directed, is re-radiated back to space than downward to the surface, but that is separate factor for me to handle.
If you should ever see my view on the density gradients and temperature, either way, let me know here. Once again, thanks for the help and an open ear!

dr.bill
May 12, 2010 9:47 pm

wayne: May 12, 2010 at 6:43 pm
Hi Wayne,
The conduction effect is there, in our atmosphere and anywhere else that things are in contact, but it isn’t the dominant effect on Earth; the convection and radiation processes are, as I’m sure you know.
Regarding temperature gradients and your thoughts about molecular speeds, think again about that iron bar. One end is hot. One end is cold. Once it stabilizes, every little slice along the bar is at a different, but stable, temperature, and energy is being transported along the bar from the hot end to the cold end. Each little slice stays at a fixed temperature as long as the hot end stays hot and the energy is “dumped” at the cold end. That means that the velocity of the molecules in each adjacent slice will be different, and the faster ones transfer energy to the colder ones, but they don’t actually go very far. They just sit there, move back and forth, and give their extra energy to the next ones in line, and so on until the end of the bar. Each slice gets continually “re-heated” and they keep on doing that, which is why it is possible to have adjacent slices with different speeds (and thus temperatures) for the molecules in those slices. In the atmosphere, when energy gets transported upward in this manner, it is also dumped at the cold end (the top) in the form of outward radiation. In the bar, you could dump it into a big ice-bucket or something.
It’s a lot like an electrical circuit with a battery connected to a resistor in the form of a long wire. The energy stored in the battery creates an electric field (a voltage gradient) that drives the current along the wire. As long as the battery stays topped up, and maintains that voltage gradient, the process will continue moving energy along the wire. The voltage gradient in that case serves the same purpose as the temperature gradient for thermal processes.
Regarding the 5 and 3 business, it’s only the 3 that matters for temperature (and thus speed). The 5 is what’s needed for heat capacity because such molecules can have 5x(1/2)kT units of energy at ordinary temperatures. Two of those units go into making them spin, but not actually making them go anywhere. At much higher temperatures, there are extra degrees of freedom due to bending (or vibrational) modes, that can also suck up energy (but again, don’t increase molecular speed).
Here’s a short slideshow you might find useful about all that:
Introduction to Molecular Physical Chemistry
Keep thinking (as will I)! 🙂
/dr.bill

Spector
May 12, 2010 11:29 pm

RE: Nick Stokes (May 12, 2010 at 12:11 am) “The arithmetic goes like this. About 40 W/m2 outward IR comes from ground through the atmospheric window (unabsorbed frequencies), at about 288K. Another 60W/m2 approx goes out from cloud tops, also fairly warm. The rest comes from varying altitudes, with about half from the tropopause at about 225 K. That all adds up to an average 255K.”
Thanks. I am still not sure that I have ever heard a good explanation of why the tropopause regions of the Earth and especially Venus have such strong ice-locker (negative greenhouse) effects.
It seems to me that there might be some special factor perhaps related to thermal radiation from a distributed volume that might be making tropopause regions of these planets so effective in expelling convected heat that they cool far below the nominal thermal radiation temperature. I would expect that in the case of Venus, most of the non-reflected heat returned to space is from the tropopause on up. I note that the tropopause on Venus is nominally at the end of the normal adiabatic lapse-rate at 60 km.
It seems that many educational sites I see on the web are describing Venus as the result of a run-away CO2 greenhouse effect, yet I also find a statement saying that if the Earth were ever moved to the orbit of Venus, there would be a run-away water-vapor driven greenhouse effect that would eventually boil away the oceans. This would allow UV to disassociate water molecules in the upper atmosphere, where the hydrogen would escape to space. Then the freed oxygen would be able to combine with all the carbon on the Venus-like surface and anything else it took a liking to.

George Turner
May 13, 2010 12:07 am

Spector,
There is much confusion over the term “runaway greenhouse effect.” As oroiginally postulated it was a hypothetical scenario involving water that may have occured in Venus’ distant past, but not in effect today. From there it morphed into whatever maintains the current temperature on Venus. From there it morphed into a passion play about discarded deodorant cans, or something.

Spector
May 13, 2010 12:52 am

RE: George Turner (May 13, 2010 at 12:07 am) “…it was a hypothetical scenario involving water that may have occured in Venus’ distant past, but not in effect today. “
I am not sure that CO2 alone is responsible for the greenhouse effect on Venus as there are many gaps in its absorption spectrum, but with an atmosphere that dense, perhaps almost all gases are ‘greenhouse gases.’
The scenario I was referring to implied that moving the Earth to orbit of Venus would cause the Earth to eventually acquire a Venus-like atmosphere. I do not want to try that experiment.

Jbar
May 13, 2010 3:43 am

George Turner, Steve, and all –
There is a website called SpectralCalc.com with a free public tool (pay for unlimited usage, all outputs) for calculating transmittance spectra of a gas or gas mixture. With this I calculated the CO2 transmittance in Venus’s atmosphere and found that the CO2 spectral lines broaden DRAMATICALLY when you go from 380ppm on Earth to “88,000,000 ppm” equivalent on Venus. Most of the IR spectrum from 1.4 – 28 microns is blocked, and water vapor blocks everything longer. There are narrow IR “windows” at
1.70 – 1.73 microns (max 58% transmittance)
1.82 – 1.84 microns (max 34% transmittance)
2.21– 2.42 microns (>90% fr0m 2.27 to 2.41)
3.28 – 3.56 microns (>90% from 3.36 – 3.47)
5.85 – 6.68 microns, but blocked by water vapor
>0% at 27 microns and higher, but blocked by water vapor
So SpectralCalc gives you George your VIRTIS window at 2.3 microns, but leaves most of the IR-B and C bands with 0% transmittance.
The IR-A band is pretty clear except for some water vapor peaks (and maybe other gases), but I’m guessing even a 733K surface doesn’t radiate much IR-A.(? Don’t have tool.)
But how can the absorption lines broaden so much when as Steve says “the greenhouse effect is logarithmic. After the first few percent, additional CO2 makes much less difference to the temperature”?!
The reason – you don’t stop at 100% CO2 in the atmosphere. When you go from Earth to Venus, it’s like having 88,000% CO2 in the atmosphere!!
It is because the transmittance of radiation is multiplicative based on the mass of CO2 through which the radiation must pass.
If 0.6 grams CO2/cm2 (the amount in Earth’s atmosphere) transmits 99.99% of radiation, when you add another 0.6 grams below that, it transmits 99.99% of what’s left (at constant T & P), and so on. So if you have 138,000 grams of CO2/cm2 (Venus), the transmittance is 99.99% raised to the (138,000/0.6)th power.
So, take the CO2 transmittance spectrum on Earth, with 380ppm of gas.
On Venus, you have 88 MILLION ppm of the gas, relative to Earth
Or 231,579 times the mass/m2.
If the data point on your CO2 transmittance graph for Earth is 99.99% at some wavelength “x”, you won’t even see it on your Wikipedia graph for Earth, it’s basically “100%”, but raise 99.99% to the 231,578th power, and your transmittance goes from virtually transparent on Earth to virtually zero transmittance on Venus. Let’s do the numbers:
380 ppm –> 99.99% transmittance at “x” microns
3800 ppm –> 99.90% – So far so good.
38,000 ppm –> 99.00% – Hangin’ in there.
380,000 ppm –>90.48%
3.8 atmos. –> 36.79% transmittance Uh oh!
38 atmos. –> 0.005% transmittance
88 atmos. –> 9 E-11 transmittance. Complete blackout!
Yes, on Earth the logarithmic effect makes little difference, but when you increase the CO2 by a very large number, you see that the effect is quite dramatic. The absorption lines can broaden a great deal when you are changing by many orders of magnitude.
These numbers also illustrate why wildly extrapolating the “1-3 deg C per doubling of CO2” rule of thumb does not apply. This rule was estimated for Earth for up to several thousand ppm of CO2. You can infer from above that CO2 behavior is not going to change dramatically over that narrow range. The rule is based on calculations and geological data including the ice ages and going back to the Paleocene/ Eocene period when Earth had several thousand ppm of CO2 and northern Canada was warm enough for crocodiles to survive. However, there is no data for dozens of atmospheres of CO2. Also, the rule ONLY applies for 1 atmosphere surface pressure, AND it includes all the feedbacks inherent in Earth’s climate system – clouds, ice sheets, humidity changes, etc. Thus, not only should this “rule” NOT be extrapolated by orders of magnitude, it applies only to Earth and has nothing whatsoever to do with Venus, Mars, Jupiter, or any other planet!
Thus two arguments are demolished: the “it can’t be CO2 because the greenhouse is logarithmic” one and the “it can’t be CO2 because of the 1 – 3 deg C per doubling rule” argument. Both fail because they don’t apply for 5 orders of magnitude of change. All we need now is a calculation of Venus’s prodigious radiative energy balance kilometer by kilometer in the lower atmosphere to prove conclusively that it is CO2 that is responsible for Venus’s temperature profile and that the lapse rate is merely a symptom of this layer-by-layer absorption and re-radiation of a large flux of energy.

May 13, 2010 4:11 am

Jbar:
“If the data point on your CO2 transmittance graph for Earth is 99.99% at some wavelength “x”, you won’t even see it on your Wikipedia graph for Earth, it’s basically “100%”, but raise 99.99% to the 231,578th power, and your transmittance goes from virtually transparent on Earth to virtually zero transmittance on Venus.”
99.99% to the 231,578th power is for all practical purposes completely opaque. And if the spectral lines “broaden DRAMATICALLY,” all wavelengths would essentially be captured by CO2.
So why is there plenty of observed daylight on Venus?
When there is a discrepancy between models and observation, the models are wrong.

Mike M
May 13, 2010 6:06 am

A simple experiment can put all this to rest quite easily. Take a long insulated cylinder of some gaseous mix at some pressure and temperature with temperature and pressure sensors at each end and … put it in a centrifuge cylinder long axis parallel to the radius of of the centrifuge. I doubt there is a way to generate enough G’s to get the inside cylinder end pressure to drop to zero but we really only need to generate a measurable pressure difference between the two ends not simulate an entire column out to space.

May 13, 2010 6:55 am

paul in UK says:
May 12, 2010 at 4:24 pm
I’m imagining trying to model a hypothetical simple atmosphere where all we need for the model is PV=nRT. So, if air is rising, the pressure (P) drops, but I don’t understand why in this simple model temperature (T) drops. Would not P1xV1 = P2xV2? i.e. the gas is free to expand as it rises and the pressure drops; i.e. the volume (V) increases? Isn’t PV a constant; as P drops, V increases, therefore P1V2 = P2V2 therefore no change in T? If V did not change, then I presume T would drop, but presumably V is increasing, that’s why the air gets thinner with altitude?

That’s why it’s more convenient to use this form of the Gas Law: P=ρRT where ρ is density and =n/V. Then as Bill showed above all you need is the Hydrostatic equation, dP/dy = -ρg.

May 13, 2010 7:05 am

George Turner says:
May 12, 2010 at 4:15 pm
re Phil: May 11, 8:25
I ran CO2 spectra at ~90 atm and there’s a transmission window from 3840^cm-1 to 4600^cm-1, so yes there is a narrow window.
I calculated that 799 W/m^2 could be emitted into that window (blackbody), and since parts of the window are used to image the surface from orbit, some of the radiation is making it directly into space. Other parts of the window are being used to look at constituents of the lower atmosphere, so CO2 emissions (and thus absorption) aren’t occuring there.
Even is the surface emissivity is 0.25, that would still be 200 W/m^2 radiating up from Venus surface, which is more radiation than the surface receives from the sun. If the surface has this negative radiative flux then its temperature is being maintained by atmospheric convection/conduction (or it requires that as supplemental energy).

Or the clouds and other GHGs (H2O, COS, SO2, CO etc), are reducing the losses (refer to Table1 in the VIRTIS site you linked).

GaryW
May 13, 2010 10:36 am

Well, like that little choo-choo train in the children’s book my wife read to our kids when they were very little, I’ve been watching this discussion, telling myself “I think I can! I think I can!” gain an understanding of Venus versus Earth environmental lapse rates and surface temperatures.
The first stumbling block I had to overcome was to figure out why adiabatic lapse rate would be the base factor for environmental lapse rate. The first is a calculable quantity based in physics and unrelated to specific environments, only the chemistry of the gases in question. Environmental lapse rate, however, is an observed value. The two are similar but not exactly the same for the atmospheric gases encountered. Furthermore, environmental lapse rate is specified for still air conditions, not moving masses.
Justifying the similarity of the two lapse rates was not an obvious step for me. I guess that I’ve always dealt with fluids or gasses in situations where pressure gradients due gravity are not measurable with available instrumentation. I had to sit back and think a bit. What I came up with is looking at what might happen in a theoretical situation where adiabatic lapse rate might observable and guess what is happening as the distance the gas moves is reduced in steps until it approaches that due simply to thermal energy. I can come up with no reason, thinking of ideal gases at least, why adiabatic lapse rate should not be consistent down to that level. Sensible up and down drafts may not be necessary for gas temperatures to follow an adiabatic lapse rate. OK, maybe this was obvious to everyone else but I don’t remember seeing it mentioned above.
Adiabatic lapse rate is a dynamic value. A delta for delta kind of calculation with no absolute value specified. Environmental lapse rate though is dealing with real temperatures at specified elevations allowing us to say: At elevation X we will experience temperature Y. What establishes an “anchor” pressure/temperature point from which we may then use environmental lapse rate to calculate that Y from X values? So far, no specific single factor has stood out for me.
I suppose what I’d like to see is that there is some convenient black body temperature were incoming solar radiation plus internal planetary heat source energies are balanced by radiation to space. That then would be the base from which all other temperatures are derived. Of course, a straight line dry adiabatic lapse rate is not a good model for the full depth of real Venus and Earth atmospheres. Energy movement through real atmospheres encompasses radiation, convection, and conduction. Clouds can impede radiative energy transfer, molecular thermal energy levels effect both absorption and radiation wavelength bandwidths. Phase and chemical changes introduce interesting and often non-linear variations in lapse rate. There is probably a very large number of things that effect environmental lapse rate.
Now, as to the question of “runaway greenhouse effect.” Assuming my previous statements are rational, I think it is necessary to separate out the specific impact of a CO2 rich atmosphere on environmental lapse rate to make a definitive statement. However, after working my way through this long run of comments, I have come to the conclusion that “runaway” not a scientific description. For it to have been a runaway in the past, the atmospheric temperature would have to have been much cooler than today. I see no descriptions of what we have divined about Venus’ history to believe that there was a time when Venus was as cool as Earth is today.
As for the greenhouse part. I’m not going to worry at this point about a 90% CO2 atmosphere having nothing in common with a Earthly greenhouse. I’ll just assume greenhouse means warming specifically due to CO2’s presence versus other possible atmospheric gasses. Since the CO2 that is there is at both high pressure and high temperature and is nearly opaque to infrared, it does seem like it should have some effect, and probably warming at that. Of course, that poor transmission of infrared is both upward and downward. I does not appear that using the effect of CO2 in Earth’s atmosphere to describe that gas’s effect within the Venus atmosphere is rational. “Runaway Greenhouse Effect” may make a fun sound bite but is far from accurate from an engineering perspective.
Anyway, that is how it plays out for me, looking at the subject as an engineer.

Barry Kearns
May 13, 2010 10:47 am

Jbar says:
May 13, 2010 at 3:43 am
The reason – you don’t stop at 100% CO2 in the atmosphere. When you go from Earth to Venus, it’s like having 88,000% CO2 in the atmosphere!!

This is the critical issue which was the basis of my Question #3 to Steven in my post of May 11, 2010 at 11:47 am.

Barry Kearns
May 13, 2010 10:50 am

Smokey says:
May 13, 2010 at 4:11 am
So why is there plenty of observed daylight on Venus?

Because major components of “observed daylight” are not infrared?

Correy Dukes
May 13, 2010 1:11 pm

Frustrated.
Venus’s atmosphere most likely started out like ours. However, on Earth much of the secondary atmosphere became part of the planet surface, (carbon dioxide and sulfur dioxide dissolved in the oceans or were absorbed with surface rock). If all of the dissolved or chemically combined carbon dioxide on the planet were released back into our atmosphere, its new comp would be 98% carbon dioxide and 2% Nitrogen and the pressure would be about 70 times its current value. A lot like Venus, huh? The difference is that Venus’s greenhouse gases never left the atmosphere the way ours did.
Venus’s runaway greenhouse effect, (and that is actually a scientific term), results from it’s composition. Period. It’s atmosphere consists of 96.5% carbon dioxide. Radiation enters the atmosphere, strikes the surface, and is reflected back towards space where the thick blanket of atmosphere (remember its comp – 96.5% of those nasty co2 molecules) absorbs 99% of all the infrared radiation. This, in turn, rewarms the planet with its own radiated heat (hence the term greenhouse effect). This simple explanation should also explain the tag “runaway”.
And, yes, there is temperature equilibrium. The reason we can enjoy our balmy 280K surface temp is a direct result of this. Our planet needs greenhouse effect in order to maintain a comfy temp, but it also needs to stay in balance. Without the greenhouse effect, average surface temp of the planet would be 40K cooler. So, maintaining a balance of incoming radiation and re-radiated absorbed energy is extremely important. Long wavelength infrared radiation is partially blocked by Earth’s atmosphere, mainly because of carbon dioxide and water vapor, both of which absorb very efficiently in the infrared portion of the spectrum. Even though these two gases account for only a tiny fraction of the atmosphere, they manage to absorb a large fraction of all the infrared radiation emitted from the surface. Consequently, only some of that radiation escapes back into space; the rest is radiated back to the surface, causing the surface temp to rise. With this info, you can quickly realize that even a small difference in the number of co2 emissions we contribute to the atmosphere can have large consequences.
My question is this: Why take the chance? Why not make major changes in our ways that make our planet a healthier place to live? What’s the harm? If I’m wrong (I’m not, trust me) in the end, we still have a healthy planet to give our children. If the conservatives are wrong and nothing is done to turn global warming around, we’re screwed. Period. I understand that the conservatives think that there are more important things to spend money on, but the funny thing about it is that they won’t give a shit about those things when they’re dealing with a major global climate revolution.

dr.bill
May 13, 2010 2:23 pm

Correy Dukes: May 13, 2010 at 1:11 pm
Give it a rest Correy. You’re just wasting bandwidth with that “think of the children” crap, and your “book of revelations” needs a rewrite.
/dr.bill

Barry Kearns
May 13, 2010 2:57 pm

My question is this: Why take the chance? Why not make major changes in our ways that make our planet a healthier place to live? What’s the harm? If I’m wrong (I’m not, trust me) in the end, we still have a healthy planet to give our children.
Economics is about trade-offs. It’s about recognizing opportunity costs, and all of the knock-on effects of a proposed policy.
If I could recommend only one thing to you, it would be to read “Economics in One Lesson“.
The “harm” in such an approach could, in fact, be staggeringly large. If you’re “wrong” (and trust me, you most definitely could be), there could be fewer children to leave the “healthier world” to.
You might think that trade is “worth it”. Somehow, I doubt that they would.

Bob_FJ
May 13, 2010 5:18 pm

Returning to the baffling topic of why is the average temperature on Venus reportedly uniform all over, when there is such a long nighttime to lose heat:
I mentioned that the cooling period was ~117x longer than on Earth, but have since realized that because the year length is of similar order to the day length for Venus, its presentation pointing away from the sun is ~122 earth days, not ~59 Earth days.
It is of course impossible for there to be no loss of heat over such a long period. For a start, there are some small windows for shorter wavelength infrared to escape, and there is no such thing as a perfect insulator.
Ref: Sidereal rotation period (hrs) -5832.5 (Length of day (hrs) 2802.0)

dr.bill
May 13, 2010 8:45 pm

Bob_FJ: May 13, 2010 at 5:18 pm
Jbar had a comment higher up on this thread (May 9, 2010 at 5:26 pm) which might have some relevance. It’s of “sketchy provenance”, but that wouldn’t necessarily make it un-factual. I’m not sure how truly uniform the surface temperature of Venus is, but with a very large mass and not much happening at ground level, it might be possible to get a fairly “sluggish” lower layer.
I don’t know if there is a data set giving the pressure and temperature profile of Venus right down to the surface, or if what Steve showed on his graphs is all that exists. If the whole range is available, the density profile can be found from that, and then maybe some viscosity calculations could shed some light. Anyone know about a complete datat set?
/dr.bill

May 13, 2010 10:49 pm

Correy Dukes ,
I won’t claim to understand the vertical structure of atmospheric temperature , but one thing I know is that your 40k number for earth’s “greenhouse effect” is a BS number based on a misleading , mathematically intractable assumption confounding albedo with spectrum positing a body which has a non-zero reflectance yet perfect emissivity . Far more relevant is that we are about 9k warmer than a gray ( flat spectrum ) ball in our orbit , how dark or light does not matter .
As Barry Kearns points out , you similarly appear to assign zero cost to trying to suppress the amount of available carbon in the biosphere , ignoring the significant greening of the planet promised by increasing the substance which combined with H2O by sunlight forms over 90% of every bite of food , and therefore every warmist and realist on the planet .
Even with all the discussion both here and on counter-blogs , I still see no alternative to internal heating being the cause of Venus’s steady state surface temperature , more than twice that of a ball totally absorbing facing the sun and reflective facing away , on both its , as Bob_FJ just reiterated , extremely long day and night sides .

Bob_FJ
May 14, 2010 12:57 am

Dr.bill, Reur May 13, 2010 at 8:45 pm

[1] Jbar had a comment higher up on this thread (May 9, 2010 at 5:26 pm) which might have some relevance. It’s of “sketchy provenance”, but that wouldn’t necessarily make it un-factual.
[2] I’m not sure how truly uniform the surface temperature of Venus is, but with a very large mass and not much happening at ground level, it might be possible to get a fairly “sluggish” lower layer.
[3] I don’t know if there is a data set giving the pressure and temperature profile of Venus right down to the surface, or if what Steve showed on his graphs is all that exists. If the whole range is available, the density profile can be found from that, and then maybe some viscosity calculations could shed some light. Anyone know about a complete datat set?

[1] I would describe some of Jbar’s comments as shaky rather than sketchy. If he would respond to my earlier comment here, I might debate his May 9, of 5:26PM that you cite.
[2] Let me put it this way; in my past life, when signing-off any draft report from my engineers, I would apply to their data or whatnot what I call: A test of reasonableness. If by analogy they had submitted analyses like that reported from the ESA here, I would have “politely” told them it had failed the “reasonableness test” and to go away and do it again.
The point is, that in the applied sciences, such as in engineering, we cannot make ideological assumptions, because like for example; people might get killed, or there could be bad infrastructure failures. (although mistakes do still happen). However it is a much more flippant approach in academia, where securing future research funding may well be a primary consideration.
Here is an interesting extract from the link immediately above:
The [temperature] map comprises over a thousand individual images, recorded between May 2006 and December 2007. Because Venus is covered in clouds, normal cameras cannot see the surface, but Venus Express used a particular infrared wavelength that can see through them.
Well that is interesting; over such a long time! How were the individual 1,000+ images cross calibrated? Were they all taken at the same time of day? ……. No ‘limb darkening’ then? (are some things springing to mind)
[3] I would think that the only T versus altitude and atmospheric pressure (also surface wind speed) data were derived from short survival time Soviet probes that obviously only had single non vertical descent paths and landing points.
~~~~~~~~~~~~~~~~~~~~~~~~~~~~~~~~~~~~~~~~~~~~~~~~~~
BTW Bill, I look forward to your response to my comment here.

dr.bill
May 14, 2010 3:51 am

Bob Armstrong: May 13, 2010 at 10:49 pm
Hi Bob,
I just had a browse around your website, and found your plot of planetary temperatures vs distance from the Sun very interesting. Pictures can make you think, and that graph is a good example. Venus sure does stick up there like a sore thumb. Thanks.
/dr.bill

Mike M
May 14, 2010 4:24 am

Correy Dukes: :absorb a large fraction of all the infrared radiation emitted from the surface”…. “only some of that radiation escapes back into space; the rest is radiated back to the surface”

Can you show us any other instance where nature magically violates the 2nd law of thermodynamics?

dr.bill
May 14, 2010 4:44 am

Bob_FJ: May 14, 2010 at 12:57 am
Hi Bob,
I missed that earlier message from you, and am off to work in a bit, so I’ll get back to you later about that. Regarding your other comments, I certainly have to agree with your “rubber hits the road” attitude. I beat that into my students until they’re numb from hearing about it. Too bad it isn’t a more prevalent requirement in ‘climate science’.
/dr.bill

dr.bill
May 14, 2010 5:26 am

Bob_FJ: May 14, 2010 at 12:57 am
Actually, I do have a few more minutes till I’m out the door.
For the opaque paint on the concrete, what I was getting at was that ‘things going on inside’ will surely affect what happens at the paint layer, but the paint layer is where the energy emerges, and the Planck distribution and S-B results are expressed in terms of what happens at surfaces. I think of it like the nozzle of a fire-hose. Any number of things may be happening ‘back the line’, but the nozzle is where the water finally gets out. The pressure behind it will affect what happens at the nozzle, but the characteristics of the nozzle also shape the effect of the pressure.
That analogy is a bit weak, however, in the case one object (say your ice cube) emitting ‘from inside’ of another object (the water layer outside of it), since water in a fire-hose, being ‘materially substantial’, can’t pass through other water molecules the way that radiation from one place can pass through radiation from another, and then combine to give the composite emission pattern. I’m glad you brought up this example. I’ve never given much thought to the idea of a ‘two-layer’ or ‘multi-layer’ Planck emitter before, and it’s a very interesting question that I’ll explore further.
/dr.bill

May 14, 2010 5:36 am

Mike M says:
May 14, 2010 at 4:24 am
Correy Dukes: :absorb a large fraction of all the infrared radiation emitted from the surface”…. “only some of that radiation escapes back into space; the rest is radiated back to the surface”
Can you show us any other instance where nature magically violates the 2nd law of thermodynamics?

No it doesn’t in this case either! You appear to misunderstand the 2nd Law, your version appears to hold that a photon knows what the temperature of its ultimate target is before it is emitted and only leaves in that direction if the target is cooler. The 2nd law as far as radiation is concerned says that net transfer is from hotter to cooler.

Nullius in Verba
May 14, 2010 1:32 pm

MikeM,
“Can you show us any other instance where nature magically violates the 2nd law of thermodynamics?”
There’s no violation, for two reasons – firstly, the 2nd law only talks about the net flow and that’s still directed upwards, and secondly, because heat can indeed go from colder to hotter if there is an input of work. A refrigerator transfers heat from the cold interior to the warm exterior, without breaking the second law. In the case of the atmosphere, convection supplies this work.
The model of heat being radiated back is a description appropriate to a non-convective atmosphere, and if it wasn’t for convection, the atmosphere would indeed operate this way. (See here for more detail.) However, because the atmosphere does have convection, the pure radiative model is incorrect, and grossly misleading. It’s also contradicted by experiment.
Correy is wrong, but not because there’s a problem with radiation shining from a cold sky to a warm surface. He’s wrong because convection prevents any such surface heat build-up before it can occur.

dr.bill
May 14, 2010 5:28 pm

Nullius (if you happen by):
I don’t want to get you blushing or anything, but that was a nicely put together explanation you posted on Science Of Doom website. Admirable clarity.
/dr.bill

Jbar
May 14, 2010 6:11 pm

Smokey:
Why is there daylight seen on the surface of Venus?
Well, because daylight is VISIBLE LIGHT, NOT INFRARED! CO2 does not absorb visible light.
When a blogger’s shaky knowledge conflicts with actual data, it’s the blogger that is wrong.
Hey, you started it!

Jbar
May 14, 2010 6:17 pm

Mike M:
Great. Anybody got some 50km long cylinders?
Well, actually, yes. You can model long cylinders of gas at constant temp and pressure on
http://www.SpectralCalc.com
For free if you just want the graph and not the text data file.
Please excuse me if I’m showing attitude, cuz I’ve enjoyed half a bottle of white tonite.

Jbar
May 14, 2010 6:22 pm

George Turner said
“I ran CO2 spectra at ~90 atm and there’s a transmission window from 3840^cm-1 to 4600^cm-1, so yes there is a narrow window.
I calculated that 799 W/m^2 could be emitted into that window (blackbody), ”
Yo, George, how are you doing this calculation to get the 799W/m2? Is there an online tool?
Not doubting you or nothin’. Just want to check it out and “Favorite” it.
BTW, I think Spectralcalc.com lets you specify a radiation source to use with your gas transmittance case??? Been a long time so not sure. But I think you must pay for that.

Jbar
May 14, 2010 6:32 pm

Gary W:
As you suggest, the “runaway” narrative is entirely speculative. It “could” have happened, but there is no historical proxy data to back it up, and maybe never will be anything more than theoretical models.
It assumes Venus started like Earth, with oceans.
Once the oceans reach 100 deg C, thanks to proximity to the sun, water vapor steadily enters the atmosphere, increasing the greenhouse effect all the more and boiling more water which leads to more greenhouse until the oceans are boiled away. That’s the “runaway”. Once it starts boiling, it can’t stop until the oceans are gone.
However this is entirely fantasy because there is no data to specify Venus’s original conditions. Well, except for the high deuterium/ hydrogen ratio in Venus’s atmosphere. But that doesn’t prove Venus was ever like Earth.

Jbar
May 14, 2010 6:51 pm

Barry Kearns:
In re your May 11 11:47 AM Q#3 to Steve:
Yes, transmittance/ absorbance is not a function of percent of a gas in the atmosphere, but rather a function of the mass or moles of gas that the radiation must pass through. Each molecule of gas has a tiny chance to hit a photon of IR, reducing the IR by a certain very small percentage. The more molecules of gas you have, the more chance of hitting photons. But it’s not a linear function.
If 1kg of gas absorbs 1%, 99% comes out the end.
The next kg of gas absorbs 1% of 99%, leaving 98.01%
3rd kg absorbs 1% of 98.01%, leaving 97.03%
10th kg leaves 90.4%, 100th kg leaves 36.6%, 200th kg leaves 13.4%, 500th leaves 0.66%
1000th leaves 0.0043%. And so on. It is easily calculated by spreadsheet if you know the absorbance/transmittance factor.

Jbar
May 14, 2010 7:36 pm

Correy Dukes:
Ran across an article in Science today (a March issue, a little behind on my reading) apropos to Earth’s primordial atmosphere:
Recent thinking is that after Earth accreted from asteroids, the sun was rotating much faster than today. Over time, its angular momentum or energy of rotation got transferred to its atmosphere, the corona, creating a solar wind much more powerful than today’s, strong enough to strip all the “terrestrial” (i.e., “non-giant”) planets of their atmospheres, especially the ones without magnetic fields, but even Earth’s WITH a mag field. Then the planets re-acquired atmospheres courtesy of being pelted by asteroids and especially soggy comets.
Venus being closer to the sun and therefore hotter to start out, there is just no way of knowing what kind of atmosphere it had after the comet-pelting. It may NEVER have been like Earth. Any rocks that would testify to its “initial conditions” have all been cooked. Therefore all we have to go on are computer models and any obscure clues from its current atmosphere, and even these have a healthy degree of speculation and assumptions.
Sadly, a Venus-like fate IS envisioned for the Earth, but only because the sun is eating its hydrogen and slowly becoming denser and therefore reacting hotter and faster over the eons as a result, but even this fate will take a billion years or so, and anthropogenic global warming CANNOT turn Earth into Venus.
So where is Earth? Evidence is convincing that humans are increasing CO2 in the atmosphere (this blog notwithstanding) and that this will result in a temperature increase of some degrees C by 2100 (this blog…). IF we want to try and stop CO2 from exceeding some value, it will take a BIG pile of money. The $3 trillion question is why would we do that?
A: To prevent FUTURE human suffering and
B: To prevent FUTURE mass extinctions.
Because the REAL temperature increases and ice melt and sea level rise won’t come until 2050 and later, and despite DOCTOR James Hansen’s wild extrapolations, the Greenland ice sheet probably will not melt for at least 500 – 1000 years.
BUT: There are billions of people suffering today AS WE TYPE. Their suffering can be reduced much more “easily” for much less money than it takes to stop climate change (if only anyone cared enough to actually DO anything about it, like Bill Gates is doing WRT fighting debilitating tropical diseases – YAY BILL!!… even though you double-charged us for 28 years for your bug-eaten software!).
AND: The economic benefits from “stopping” climate change won’t be seen for many decades, and the BIG benefits not even for centuries. If we invest a trillion dollars today, is it worth getting a 1% return on our investment (as the Stern report claims it is) compared to a typical business investment return of 15% to 30%, or gov’t spending return of 5% when we could continue to pour less money into energy research and get the technology costs down and then deploy the cheaper technologies at a later date, like 2020 (My “drop dead” date)?
It is a pickle. We much choose a path, and if we choose the wrong path we will either be screwed, as you say, or “shot in the foot”.

May 14, 2010 7:42 pm

Barry Kearns, May 13, 2010 at 2:57 pm, quoting Correy Dukes:
“Why take the chance? Why not make major changes in our ways that make our planet a healthier place to live? What’s the harm? If I’m wrong (I’m not, trust me)…”
You first, Correy. Start hoofing it from your mud hut down to the river to wash your loincloth on the rocks. Enjoy those mud grubs for dinner.
Anyone who says ‘trust me, I’m not wrong,’ must be about 15 years old, at least emotionally. It’s an attempt to end the debate, and it won’t work here. This isn’t the censoring realclimate, climate progress or tamino. This is the “Best Science” site. All points of view are welcome, and are hotly debated. The ones that remain standing are accepted. That’s why the alarmist crowd hates WUWT: Their arguments generally fail due to lack of hard facts.
Also, Barry, I second your recommendation of Henry Hazlitt’s Economics In One Lesson, which I still have after forty years. The essence of the book, demonstrated through numerous examples, is that when the free market is abandoned, then every dollar that one person gets is a dollar forfeited from another. That is what we’re seeing today. And it is causing enormous harm.
Mass transference of wealth from those who earned it to those who did not cheats the honest, hard working citizens, and hands unearned wealth to the grifters and the government’s subsidized class. It leads to the rule of law becoming an insider’s joke, by taking more and more from those who earned it by actually producing goods and services, and rewarding fast talking scam artists who hide out from any honest debate, and who sell their ethics to whoever provides their constant, all expenses paid trips to Bali, Copenhagen, Mexico City, Hawaii, etc., in addition to the lucrative grants regularly handed out to them with little oversight, and with no rigorous, honestly peer reviewed science either required or performed. It’s simply a back-scratching clique pushing an agenda at the public’s expense.
People aren’t stupid when it comes to knowing what is honest, after tax compensation for their work. They see what is happening. When dishonesty is financially rewarded, it promotes widespread dishonesty — as we see in the Climategate emails, and throughout the stacked deck of the clearly gamed climate peer review process.
The same wealth transference from the productive members of society to the grifters is everywhere we look, from the totally corrupt UN, to universities and their pet grant rainmakers, down to city councils and their developer and union benefactors. Like ravenous hyenas, they all have their eyes on our wallets, intent on confiscating money they never earned, all
based on emotional hand-wringing guilt projection and bogus scare stories unsupported by any real scientific evidence.
Zimbabwe, here we come.
.
</rant>

Jbar
May 14, 2010 7:44 pm

Dr. Bill, May 13, 8:45 PM
Did a search on “Venus” on ScienceDirect yesterday.
Recall running across an abstract that said there was convection near the surface of Venus [presumably to move the surface energy from visible lite absorption around to the dark side of the planet] and again in the cloud layers (i.e., above 50 km.), but not in between.

Jbar
May 14, 2010 7:50 pm

Bob Armstrong, May 13, 10:49 (was that AM or PM?)
“still see no alternative to internal heating being the cause of Venus’s steady state surface ”
Literature suggests that Venus was resurfaced 500 million years ago, surely enough time to cool down since then.
Venus can’t have a huge amount more of uranium than Earth, but then again because plate tectonics has stopped since GKW (“God knows when”), energy has gotten bottled up. So WHO the hell knows!?

Jbar
May 14, 2010 8:18 pm

Hi Bob_FJ
Kicked bottle 3 blind moose. Opening Grand Marnier. Wife away for weekend!
[1] Measurements indicate that Venus’s upper atmosphere takes 5 days to circle the planet, much faster than the Earth, at one of the 3 cloud layers, all of which are above 49km altitude. Undoubtedly it carries energy from the day side to the night side. Below that altitude, the temperature profile around the planet is “constant” – no “diurnal” variation.
Temperature at that altitude around 60C. High conc CO2 means some IR from surface reflected back toward surfacefrom above 49 km.
Surface absorbs measly 100W/m2 visible light TOPS.
However, surface is 733K, radiating “12,000 W/m2” IR – or some high number.
Dense CO2 atmosphere is SO absorptive, 12,000 W/m2 goes up, 11,900 comes back down at surface. Based on some radiative energy-balance profile yet to be calculated by person in this blog (but probably already modeled by some scientist), temperature decreases with increasing altitude, IR energy radiated upward decreases with altitude, IR energy reflected back down decreases with altitude, until at 49km it reaches 333K, circa 600W/m2 radiation.
Have to allow some reflectance of IR at cloud bottoms at 49km too.
This detailed calc is beyond my ability, or at least beyond my time available. UNLESS you pay me!
[2] You referring to Steve’s photos? Shadows appear to be illuminated from directly above. Let me ask you… what kind of shadows would you see on Earth in a cloudy sky with hidden sun? Would not rocks on Earth still cast some shadows?

Jbar
May 14, 2010 8:27 pm

Nullius [cool name] and Mike M,
May 14, 1:32 Pm
At any altitude from zero to 49 km on Venus, more IR is directed upward than downward, and atmospheric temperature decreases with altitude. In theory, at 49 km altitude, 100w/m2 (or thereabouts) of visible lite goes downward, and a net excess of 100W/m2 (or thereabouts) of IR goes upward. Therefore the Laws of Thermodynamics are not violated.
However, the details of this phenomenon are beyond my available time to analyze.

Jbar
May 14, 2010 8:38 pm

It’s 11:35PM. Think I should leave some Grand Marnier for tomorrow night!

Spector
May 14, 2010 8:48 pm

RE: High steady-state surface temperatures on Venus
Given the extremely dense atmosphere of Venus, I think that errors in the estimation of low-temperature thermal conductivity and heat flow are more believable causes of that planet’s ‘unexplained’ stygian surface temperatures than any mysterious internal heat source.

Spector
May 14, 2010 9:44 pm

I have found a study that provides a series graphs similar to the ones provided by Steve Goddard in this article except that have been extended down to the surface.
These were provided as a first step in a study to determine if there are chemical, temperature, pressure environments in the Venusian atmosphere that are suitable for the replication of viruses.
http://www.datasync.com/~rsf1/vel/1918vpt.htm

Bob_FJ
May 15, 2010 1:02 am

George Turner, Reur May 11, 2010 at 4:14 pm

And a follow up on the clear IR window from 2.2 to 2.5 microns. VIRTIS for those interested.
I calculated that a 740K blackbody would emit 799 W/m^2 in the open window from 2.2 to 2.5 microns. That’s more than the solar energy Venus receives at the surface.
check please?

Sorry George to take so long to look at this, but I was delayed trying to find a Planck curve at 740K.
According to my reading of your ESA “Venus Express” VIRTIS citation, the open window that you refer to does not occur below about 35 Km altitude, (at ~570K), and thus will not be seen from the surface. VIRTIS uses different wavelengths to view different levels in the atmosphere. In order to view the surface, (or in other words, where up-welling surface EMR/ infrared can escape directly to space), VIRTIS utilizes a narrow window at 1 micron.
If you look at the Planck curve for 750K that I found here, you will see that the proportion of EMR energy that can escape through that 1 micron window is tiny, even if the surface were a black body.
Thus, the mechanism for most HEAT loss from the surface would appear to be conduction/ convection/ advection. (converting to EMR escape at progressively higher altitudes)

dr.bill
May 15, 2010 1:57 am

Spector: May 14, 2010 at 9:44 pm
Thanks for the links to the extended Magellan graphs, Spector. It’s not the same as actual data, but I can at least check my own ballpark extrapolations against someone else’s. Much appreciated. I tracked the source data back to a JPL site that hasn’t been updated very recently (still talking about selling physical slide-sets and videotapes). Given that the numbers don’t go lower than 34km, though, having tha raw data might not be much better than digitizing the gif’s, but I’ll dig some more.
Your original site you pointed to is a bit ‘different’. Oedipus, Akhnaton [sic], Velikovsky, The Virgin (ha’Almah), and Flu Viruses ?? Eclectic, to say the least!
/dr.bill

Jbar
May 15, 2010 9:11 am

CO2 is mostly transparent at wavelengths shorter than 1.2 microns, and your 750K curve shows a not insignificant amount of radiation below 1.2 microns. If there’s only a narrow window at 1 micron, some other gas must be blocking the shorter wavelengths.

GaryW
May 15, 2010 12:40 pm

Jbar : May 14, 2010 at 6:32 pm
Just a quick note: Water does not boil at 100 degrees C at 90 atmospheres. It takes a higher temperature at that pressure to get bulk boiling of water. I’d have to dig out my steam table to even guess what the behavior of water would be. I’ve never had occasion to deal with a high temperature, high pressure CO2 gas atmosphere and water together. I’m not even sure my standard steam table is valid for that situation.

Bob_FJ
May 15, 2010 4:26 pm

Jbar, Reur May 14, 2010 at 8:18 pm , in part:

[1] However, surface is 733K, radiating “12,000 W/m2″ IR – or some high number.
[2] Dense CO2 atmosphere is SO absorptive, 12,000 W/m2 goes up, 11,900 comes back down at surface.

[1] See my ongoing discussion with Dr. Bill. I don’t think S-B blackbody radiation alone applies when immersed in a highly opaque fluid like the Venus atmosphere. (and Dr. Bill has yet to dispute this)
[2] Regardless of how much radiation there may be from the surface, (hemispherically) when it is absorbed in any particular layer of the atmosphere, that layer radiates in all directions. (spherically). Crudely speaking, the same amount goes up as what goes down, and a great deal of it goes sideways, which does nothing in itself.

You referring to Steve’s photos? Shadows appear to be illuminated from directly above. Let me ask you… what kind of shadows would you see on Earth in a cloudy sky with hidden sun? Would not rocks on Earth still cast some shadows?

Yes, shadows can be seen on overcast days on Earth, but it is a matter of context here. The shadows evident in the photos from the various Soviet landers, (both enhanced and raw), do not support the hypothesis of immense scattering of sunlight around to the dark side, as a reason for uniform temperature. (not visible under the clouds)
For more surface imaging and processing, see:
http://exoplanet.as.arizona.edu/~lclose/teaching/a202/Venus_surface.jpg
http://nineplanets.org/thumb/lander.jpg
http://www.mentallandscape.com/C_CatalogVenus.htm
http://www.mentallandscape.com/V_DigitalImages.htm

dr.bill
May 15, 2010 10:25 pm

Bob_FJ: May 15, 2010 at 4:26 pm
[1] See my ongoing discussion with Dr. Bill. I don’t think S-B blackbody radiation alone applies when immersed in a highly opaque fluid like the Venus atmosphere. (and Dr. Bill has yet to dispute this)

And he won’t, Bob, because you’re right. ☺
Most of the radiation emitted by the Earth comes right from the surface and goes straight out into space. The S-B temperature associated with that would be the surface temperature of the Earth (if the Earth actually had a single surface temperature, which is another matter altogether).
The rest of the radiation is ‘impeded in progress’ (which is not the same as being ‘trapped’) by the gases through which it passes. Some of it, as you say, is re-radiated down to the surface, where it has some effect on the total striking the ground. That bumps up the surface emissions a little bit from what they would be if the atmosphere were completely transparent, and this process iterates itself until an equilibrium is attained (if all other things were magically kept fixed). On each iteration, however, the added effect of the bits coming back down decreases significantly, like a small fraction multiplied by itself over and over (one-tenth of one-tenth of … etc). To get the total, you’d have to do an infinite sum, but the first one or two bits would be the bulk of the effect.
Then, of course, there’s the matter of what temperature to associate with those extra bits that are going up and down. That’s actually rather tricky. One of the things that’s sometimes forgotten about blackbody stuff is that it’s a ‘whole molecule’ effect, not just a matter of electron transitions, whereas most of the absorption of radiation in the atmosphere simply involves the excitation of electrons in the molecules, and the re-emitted photons will thus have the same energy as the originals. That energy in turn will have been determined by the temperature of the Earth’s surface, which is where they came from in the first place. The ones sent back down will thus be readily re-absorbed by the surface since anything that emits a given frequency will also absorb that same frequency. (Again, of course, they might not end up in the same patch of land or sea that they came from, and that’s part of what makes all this stuff so damned difficult to process with actual numbers.)
There is, nonetheless, a small ‘atmospheric thermalization’ effect involved in these processes, so the molecules higher up that are absorbing and re-emitting the upwelling radiation will end up a little bit hotter than if they were left alone. I’m not sure that anyone knows how to quantify that. I certainly don’t, but most of the radiation they absorb won’t make them move faster. It just ‘messes with’ their electrons, which then re-emit in short order.
On a tangential note, there’s an interesting example of this ‘back and forth’ process that occurs in the construction of anti-glare coatings for camera lenses and eyeglasses. It’s based on thin-film interference among the multiple reflections and transmissions that take place in the thin layer of the coating, which is sandwiched between the air on the outside and the glass of the lens itself (which is considered ‘thick’ for such purposes). Again, you figure out the total effect by adding up the series of weakening rays of light. Then you apply the condition that all of the reflections add up to destructive interference, and that determines the required thickness and index of refraction needed for the coating. In this case too, there is no ‘energy buildup’ of any significance in the coating. It just takes a little bit of time (less than nanoseconds) for the first round of multiple reflections to work their way through, and then the steady state conditions set in. The coatings don’t keep getting hotter and hotter. Maybe we should get the optical people involved in atmospherics. 🙂
/dr.bill

May 15, 2010 11:39 pm

The fact that the atmospheric pressure is the same in the Sahara desert in midsummer and the Antarctic in midwinter is the same shows that surface temperature is not proportional to pressure.

Bob_FJ
May 16, 2010 1:24 am

Jbar, Reur May 15, 2010 at 9:11 am

CO2 is mostly transparent at wavelengths shorter than 1.2 microns, and your 750K curve shows a not insignificant amount of radiation below 1.2 microns. If there’s only a narrow window at 1 micron, some other gas must be blocking the shorter wavelengths.

The Planck curve gives the distribution of energy flux at quantum level for a given black body temperature, and it is the area under the curve that gives total flux.
Per my eyeball, I would guess that the area to the left of 1.2 microns, is less than 5% of the total flux.
BTW, did you notice that the Y axis of the graph of three sample Planckies does not have any units? Have a play with the applet on this web page, and you can see that there is a scaling difficulty even between 190K and 360K, (the range of the applet), if units were shown
Don’t worry about the blue Wein curve; it is irrelevant to this discussion.

wayne
May 16, 2010 5:27 am

Hi dr.bill, you sill around? Watch out, it can be addictive. This has turned into a hearty discussion here.
Update: my realization that by ρ = MP/RT it’s the pressure to density ratio that rather dictates the temperature and that put a complexity in my simple example, but I’ll give it some more attention later when time allows.
If you should have the time I’ve got a quirky question.
Got into a heated discussion on albedo in space’s case. Take two identical planets similar to Mercury in identical perfectly circular orbits, rotating to equalize the radiation heat from the sun and with no atmosphere, cold to the core, stable irradiation from the sun. One is white and the other is black.
My argument was that after enough millions or billions of years both would equalize to have identical core temperatures because conduction to space could not occur in a vacuum to help remove heat as air would in a lab experiment, it would just take the white one much longer to reach radiation equilibrium. A good reflector being assumed an equally poor radiator. The white was accepting less radiation per unit of time but the incoming radiation intensity was a constant and would win the battle in the end in both cases. But that would only hold in a vacuum as space.
The other person swore that by physics the white one would always be much colder, it’s albedo was so much lower, end of discussion, as if albedo controls temperature, not just reflection.
Any thoughts? No problem if I’m missing something, just want a second opinion from someone who seems to understand physics.

evilincandescentbulb
May 16, 2010 9:03 am

So, if I understand correctly, many global warming alarmists seem to dismiss the relevance of the difference between the atmospheric pressures of Mars v. Venus to explain differences in their respective temperatures. Does that make these folks atmospheric pressure deniers? And, does that help explain why these deniers also think MBH98 (aka, the ‘hockey stick’ graph) is good science instead of 10% reality and 90% scientific fraud?

dr.bill
May 16, 2010 1:00 pm

re wayne: May 16, 2010 at 5:27 am
Hi Wayne,
Not to worry about me trashing my life. I’ve been online since the Arpanet days, and I’m very selective about where I spend my cyber-time. WUWT is a rare departure from my habit of ‘reading and moving on’. This site feels more like a place to learn and share, rather than a vehicle used to ramrod an agenda (not to say that there isn’t a bit of that sometimes, but it’s not the dominating feature). I learn things from what I read, and sometimes I can add things that may be of help.
———
Now, back to your gases…. I think that the ρ = MP/RT relationship is the best one to keep in mind when thinking of gases, but there are a couple of caveats. Firstly, that particular equation is very general, but it is based on the Ideal Gas Law, so be careful when things get ‘extreme’, in which case you have to use a more extended equation of state. Secondly, I would advise being careful when attributing cause and effect. All three variables can be changed separately.
– For example, if you do work on the gas, its volume will decrease, its density will increase, and its temperature will tend to increase, but you can inhibit that by siphoning off energy if you cool it while compressing it. It’s common to do this when filling various kinds of gas cylinders.
– On the other hand, if you heat the gas, its temperature will be the first thing to increase, followed by the pressure. The density will tend to stay fixed, but you can regulate that too if you let the gas expand as you are heating it. In that case, the gas won’t increase its temperature by as much as it would in a fixed volume. That’s why heat capacity at constant pressure is greater than at constant volume.
– In short, the ρ = MP/RT relationship connects the variables at any instant, but doesn’t tell you what’s driving the process, nor which way it’s headed. That’s the basic point on which I disagreed with Steve way back at the start of this thread.
———
Regarding your black and white bodies, if I could go back in time and smack a few people up the back of the head for picking bad terminology, whoever came up with the ‘blackbody’ term (Kirchhoff, I think) would be high on my list of candidates. What it comes down to is that ‘blackbody’ doesn’t mean the same thing as ‘black body’. Ditto for ‘bluebodies’ and ‘blue bodies’.
In the case of your scenario with a white planet and a black planet, both of them ‘cold to the core’ to begin with (which I’ll take to be absolute zero or thereabouts), I’m afraid that I would side with your friend on the outcome. If these planets are just ‘popped into place’ and start receiving radiation, the one with the higher albedo will absorb less energy, and will heat up less. Keep in mind that it is emitting radiation at the same time as it is receiving it, and that all these blackbody things relate to equilibrium conditions, not to the transient process that gets them to that equilibrium. In the absence of any other energy input or output, the white one doesn’t need to get as hot in order re-radiate the amount of energy it receives.
An analogy might be those ‘heat lamps’ that you see in cafeterias. If you put two identical food trays under two identical lamps, they’ll both eventually settle to the same temperature, even if one of the trays was warmer than the other at the start. If you cover one of them with aluminum foil, however, that one won’t get as hot as the one exposed to the full effect of the lamp.
It’s also a bit like driving a car. If you start from rest and ‘ratchet’ the gas pedal to some fixed position, the car will pick up speed, quickly at first, and then more slowly until it reaches ‘terminal velocity’. That’s the speed at which the propulsive force of the engine just balances the drag forces on the car (and thus most of the fuel we use in our vehicles is ‘wasted’ in the process of pushing air molecules out of the way.) Anyway, the pedal position is equivalent to the amount of incoming radiation that isn’t reflected in the first place, and your speed is analogous to temperature. If you want to go faster (or get hotter), you have to press harder on the ‘pedal’.
Sorry for rambling on so much.
/dr.bill

wayne
May 16, 2010 3:34 pm

dr.bill says:
May 16, 2010 at 1:00 pm
I had an inkling I might be somewhat or totally wrong, leaning too much on the pure theoretical principles, not what actually happens, because nothing really fits into many of TD ideal equations. However, you’re trays mentioned do have air touching them so conduction enters into the example, that was one huge difference in my point, it was in a vacuum of space. My viewpoint would require a more hypothetically property that emissivity to be exactly equal to ‘absorbability’ of the matter which gave them their color, never more and probably never occurs in reality.
Just found an open course offered at MIT on Thermodynamics & Kinetics, Chem 5.60 I think that I going to go through it to get some answers instead of leaning so much on others, but I’ll be around. I agree, WUWT is a great site if you like to interact in science. Just my cup-of-tea. I’ve spent years reading scientific papers but that manner of learning can leave some gaping holes. Time to hit the books again.
-w

Bob_FJ
May 16, 2010 4:57 pm

There has been speculation by the ESA with VIRTIS images that lava flows are “recent” and that Venus may be currently active.
It’s interesting to study this geothermal gradient graph for Earth, and to ponder what the effect would be if the surface temperature were similar to Venus, or what might cause it to be similar, with different geology. Thoughts anyone?
Here’s another graph that might be handy:
http://geolor.com/GEOLOR/Core/Earth-Interior-Temperature.gif

dr.bill
May 16, 2010 5:52 pm

wayne: May 16, 2010 at 3:34 pm
Hi Wayne,
I’d say that taking a course is a good idea (even without my teacher’s bias!). A well-constructed course has the benefit of including all the relevant pieces, in the right order, and maybe leaving out just enough to make you struggle with it a bit. And, of course, there’s nothing like sweat equity to help with learning – for teachers as well as students.
I bet you’re gonna work that teacher’s butt off, though! 🙂
/dr.bill

May 17, 2010 3:00 pm

( Boy I wish there were a way to preview posts . )
Wayne , your question @ May 16, 2010 , 05:27 highlights why I continue to judge the quality of understanding of the basic physics apparent on both sides of the conflict is pathetic . Certainly , so called “climate science” has become detached from the fundamental quantitative constraints well understood a 100 years ago .
I keep being given lists of $100 textbooks to slog thru to understand the problem . But if any of them had the definitive quantitative , ie , equation by equation , experimentally confirmed , explication of planet/atmosphere temperature profile , we would not be having this controversy . There certainly has been no Chandrasekhar in the field and that strikes me as bizarre given that these issues appear much simpler than those he analyzed .
I’ll be so bold as to assert that my own implementation , in several modern Array Programming Languages , of the StefanBoltzmann & Kirchhoff relationships for non-uniform gray ( flat spectrum ) spheres with arbitrary surrounding heat sources and sinks is the clearest and most general explication of the first-cut calculations which explain all but about 3% ( 9 kelvin ) of our temperature .
I’ll attempt to insert here the graph of the “Calculated & Observed Temperatures of Inner Planets” which I thank Dr Bill for his complement about the other day . If it didn’t show , you can find it on my website .
You will see how enormously higher the surface temperature of Venus is compared to a gray ball in its orbit simply heated by the sun . Given the StrfanBoltzmann P = sb * T ^ 4 relationship , where the sb constant drops out of any comparisons of temperatures between bodies , the surface of Venus is trying to radiate more than 16 times the amount of power it is receiving from the sun .
Sorry dr.bill , Wayne ‘s right on his intuition that “black” and “white” balls will come to the same temperature . That was Kirchhoff’s incredible insight 151 years ago . But it applies to flat spectra so that the correlation between the ball’s spectrum and its heat sources and sinks will be the same . That’s why the graybody temperature is the first term to be extracted . It is orthogonal to spectrum , thus makes no difference whether dark or light .
I have never found a quantitative definition of “the greenhouse effect” on the web — which is all that matters . So I’ll give one which makes sense :

GHE is the ratio of the correlations between the spectra of an object and its radiant sources and sinks

since this is the ratio between absorptivity from sources versus emissivity  towards sinks . Given any set of object and source and sink spectra , the equilibrium temperature can be calculated .
I’ll leave as a homework assignment for those who understand vector geometry what spectrum of an object will maximize its temperature for a simple situation like our sun’s spectrum versus the 3k cosmic background .
If the mean surface temperature of a planet exceeds that value , then it must have an internal source of heat .
Since I don’t have the time , you will find on my website that I am offering $300 to any student who elaborates the handful of lines of my algorithms to calculate values for full spectra – and calculates the equilibrium temperature for a bunch of spectra of interest such as pure CO2 . That’s well below minimum wage but the answers should be most interesting .

Keith Minto
May 17, 2010 4:06 pm

For dr.bill, from Bob’s website http://www.youtube.com/watch?v=X-swoy2WKm0 .

dr.bill
May 17, 2010 4:21 pm

Bob Armstrong: May 17, 2010 at 3:00 pm
Sorry dr.bill , Wayne ‘s right on his intuition that “black” and “white” balls will come to the same temperature . That was Kirchhoff’s incredible insight 151 years ago . But it applies to flat spectra so that the correlation between the ball’s spectrum and its heat sources and sinks will be the same . That’s why the graybody temperature is the first term to be extracted . It is orthogonal to spectrum , thus makes no difference whether dark or light .

Hi Bob,
I think our lack of agreement might just be a matter of interpreting the question in different ways. There’s also the issue of dealing with hypothetical scenarios that have a few ‘not-actually-possible-in-real-life’ aspects to them. Some gedanken experiments work out better than others. ☺
On a more substantial note, however, have you worked out the results for the Moon? From what I have understood of your calculations (and my grasp of those might not be complete), I would guess that the values should be quite similar to those for the Earth itself.
/dr.bill

dr.bill
May 17, 2010 4:29 pm

Keith Minto: May 17, 2010 at 4:06 pm
For dr.bill, from Bob’s website

Hi Keith,
I’ve actually seen that before, and have no issues with it. I didn’t, however, know that it was from Bob. A good demo, and based on observable effects in the real world. My compliments again, Bob.
/dr.bill

Bob_FJ
May 17, 2010 5:07 pm

Bob Armstrong Reur May 17, 2010 at 3:00 pm
I’ve had a quick look at your website, and notice that you refer to Venus radiating energy at ~735K
There seems to be a dearth of literature on how S-B radiation is affected when a body is immersed in highly opaque fluid. However, Dr. Bill and I have been discussing this e.g. here, and have concluded that simple logic dictates that the Venus atmosphere is so opaque that the surface will not comply with S-B alone. One simple analogy is painted concrete, where the concrete loses HEAT into the opaque paint, and it is only the paint that then radiates EMR.
This means that most heat loss from the Venus surface is probably via conduction/ convection/ advection, except through some small windows.
(and not the same as it would be via EMR at 735K in a transparent or Earthly atmosphere)
There has been no dispute from Phil, or Tom Vonk, et al. (if they are around)

wayne
May 17, 2010 5:11 pm

Bob Armstrong says:
May 17, 2010 at 3:00 pm
Wayne , your question @ May 16, 2010 , 05:27 highlights why I continue to judge the quality of understanding of the basic physics apparent on both sides of the conflict is pathetic . Certainly , so called “climate science” has become detached from the fundamental quantitative constraints well understood a 100 years ago .

Bob, you have some very, very good insight. Halleluiah, some are really thinking science! Normally this type of comment I made would fall on deft ears.
I have been posting numerous scenarios, as the one above, for almost a year and you are one of the vary first persons of science which has seen through my post to the core. Bravo! I’m going to give dr.bill a pat on the back too that he is one of the first posting here willing to actually converse on actual underlying science. That is a rarity. Maybe the tide is turning. Two science minds found in one post, fantastic.
That example given above about the white and black planet, by the way, it was just a test but an important one, is what I would like to see occur here along with the normal present concentration on climate science and it’s political and social consequences. I mean real science conversations looking under the hood at the physic laws and their unpinning principles which govern the earth system, even at any planet’s system. So far that has only been a dream, but you two have done it for the first time here. I hope it continues as the months go by, there are so many incredibly simple questions as the one above that lay there concretely unanswered so everyone visiting WUWT to know. It’s a start.
I am not positively sure that my answer is true, but like Bob, I think it is. Dr.bill might help in answering that basically simple question, same temperature or not. If it’s incorrect, exactly why. What I see is if we cannot totally answer a question like that you can never take the next step, like for instance, now and an atmosphere, any atmosphere to that example. Does the added atmosphere make null the physics that we just proved to apply to the bare rock? That was where I was heading.
Bob, just visited your site, interesting, I will explore some later.

wayne
May 17, 2010 5:29 pm

dr.bill says:
May 17, 2010 at 4:21 pm
I think our lack of agreement might just be a matter of interpreting the question in different ways.

If you need to further constain that example so that a firm answer might be found, do so.

dr.bill
May 17, 2010 6:07 pm

wayne: May 17, 2010 at 5:11 pm
Hi Wayne,
Many thanks for the kind words, but it’s just a trait I inherited from my parents (along with their penchant for taking apart everything that came into the house in order to see how it actually worked), and that got reinforced by some other good people along the way. I’ve been “programmed”. ☺ In any case, I’ll have another go at your black and white planets.
The white one: I assumed that you intended it to have an albedo of exactly 1.00, so in my way of thinking, it will reflect 100% of the radiation impinging upon it. If it does that, then none of the energy gets transimitted down into the planet, so I don’t see how the planet will get any warmer.
The black one: I assumed that you wanted this one to have an albedo of exactly 0.00, in which case it will absorb everything that comes at it, and will indeed get warmer – up to the point where it is re-radiating at a temperature that allows it to put out the same amount of energy as it is receiving.
Is that any improvement?
/dr.bill

Bob_FJ
May 17, 2010 6:59 pm

Dr.bill, Reur May 17, 2010 at 4:21 pm to Bob Armstrong:

On a more substantial note, however, have you worked out the results for the Moon? From what I have understood of your calculations (and my grasp of those might not be complete), I would guess that the values should be quite similar to those for the Earth itself.

Pardon me for butting in, but I find the argument for an effective radiation T for planets/ moons to be a bit of a stretch.
In the case of the moon, the surface T is said to range from very cold to very hot; some 350C. How this can be sensibly integrated into an average T seems to me to be rather simplistic. One difficulty is that EMR loss to space is proportional to the fourth power of T. Thus heat loss will be relatively rapid in the hot spot area under the sun, and much slower elsewhere. (whereas, this is not so on a planet with atmosphere and oceans shuffling the HEAT and EMR around) Other difficulties include the length of exposure to sunlight and darkness, and the thermal characteristics/ inertia of the lunar regolith.

dr.bill
May 17, 2010 7:22 pm

Bob_FJ: May 17, 2010 at 6:59 pm
Pardon me for butting in, but I find the argument for an effective radiation T for planets/ moons to be a bit of a stretch.

Hi Bob (FJ),
I completely agree, and even on the Earth, it’s a bit of a stretch, but can perhaps be ‘tolerated’ for some purposes. If I understand Bob (A’s) calculation method, however, it seemed to me that he would get pretty much the same result for the Moon as for the Earth. If that were the case, then I would say that’s there’s a flaw in his methods. If it were not the case, then I would be inclined to take a deeper look at what he’s doing.
/dr.bill

wayne
May 17, 2010 7:41 pm

dr.bill says:
May 17, 2010 at 6:07 pm
Hi Bill, it should not matter, and if it does, I am just wonder exactly why. Let’s make the white albedo 0.8 and the black 0.2, you chose the emissivities, remember, no atmosphere, but spinning, constant input (or not spinning and stationary more like Venus if that is a simpler case.) Don’t really want to rule emissivity totally out of the picture if possible, it seems that should not matter but would also only affect the rate of energy gain when not at equilibrium (the number of years to reach equilibrium). To me, emissivity (contra absorbability) acts similar as albedo in this type of case but for the fourth power rate.
In some respects it’s a harder question than it first seems. Try that one on your students, maybe you have a Nobel capable student right there in your class and he/she can straighten us out! 😉

evilincandescentbulb
May 17, 2010 8:25 pm

Members of the church of global warming alarmist ‘science’ which primarily has been given wings by pseudo-intellectuals in the Western, secular, socialist, government-funded education complex, have gone full circle from the sort of people that generally have been hostile to Judeo/Christian ethics, principles, morals and traditions — and religion in general — but who now have become proselytizers of a new age doomsday religion that is particularly attractive to Leftist-libs and enviro-whackpots with teachings like, human CO2 is destroying the Earth, species face extinction and seas will run red and flood lands killing millions. These are the same people that kill babies in their wombs, say putting a crucifix in a glass of pee is art, burning the American flag is speech, granting Al Gore the Nobel Prize is recogintion of greatness, and look to leaders like Castro, Chavez and Mao and anyone else who is anti-America for their spritual guidence, i.e., they’re America’s flower children gone to seed and as worthless to society as a three dollar bill.

dr.bill
May 17, 2010 11:41 pm

wayne: May 17, 2010 at 7:41 pm
OK Wayne, so what’s the deal here? You just gonna keep saying ‘No’ till I say ‘Yes’? Maybe I should put you in touch with my ex-wives, and they can fill you in on that! ☺ Just kidding (about you, that is). All right then, here we go:
Definitions and Symbols (and no more changing ponies in mid-game):
Two planets, no atmosphere, balls of rock, uniformly irradiated.
α and e – (albedo and emissivity, pure numbers, values 0 to 1)
σ – (Stefan-Boltzmann Constant, 5.670E-08 W/m²K^4)
I – (incoming radiation intensity, W/m²)
T – (ultimate equilibrium temperature of the planet, K)
At equilibrium (which might take some time): Power In = Power Out
This translates to: (1 – α)I = eσT^4
Example of usage:
Dark Planet: α = 0.20, e = 0.60, I = 345 W/m², T = 300K
Light Planet: α = 0.80, e = 0.90, I = 345 W/m², T = 192K
Now, I’m sure that you can plug numbers just as handily as I can, and if you play with the values a bit, you can make either planet the ‘hot one’, depending on what you choose for α and e. The only way to give them the same temperatures – under these stated constraints – is if the ratio (1 – α)/e is the same for both of them (and they get the same incoming power, of course).
One last thing: Let me assure you that I didn’t just invent this stuff. Look up any decent textbook on the Physics of radiating bodies, and you will find the equation that I started with. It is much older than I am, and has been vetted by generations of scientists with no ‘climate agenda’.
You happy now? ☺
/dr.bill

wayne
May 18, 2010 10:47 am

dr.bill says:
May 17, 2010 at 11:41 pm
You happy now? ☺

Yes dr.bill, that schoolbook example says it all. 🙂

dr.bill
May 18, 2010 2:29 pm

wayne: May 18, 2010 at 10:47 am
Maybe in the future I should skip the chatter and just slap down the equations! In addition to my ‘people languages’, I’m pretty well-versed in ‘equation-speak’, and in the end, it IS the language of Physics. Anyway, it’s been fun, Wayne. Whenever I’m not getting what someone else is getting at, and then we figure it out, there’s a benefit all round. ☺
/dr.bill

evilincandescentbulb
May 18, 2010 5:46 pm

In one of the great examples of cerebral contortionism I’ve ever seen, the authors of the study below conclude, saying:
“Lastly, one can invert the title of this paper and ask `Does the occurrence of lower/higher solar activity make a cold/warm winter in Europe more likely (than the climatological mean)?’ Our results strongly suggest that it does, which has implications for seasonal predictions.” (Ibid.)
We must do our best to cut through this obfuscation and filet the mumbo-jumbo if we want to get to the meat of the matter, Ok?
What we see yet another example of erstwhile authors being forced to dance and essentially proclaim without the slightest scientific backing that the role of the sun on the weather in the UK may have a bearing even on the rest of Europe but certainly not on the rest of the Northern Hemisphere and certainly not the rest of the globe. Accordingly, these dancing authors have tapped out a fiat license allowing global warming alarmist to endlessly continue to propagate groundless fears of runaway global warming due to human causes-even though the UK and possibly the rest of Europe may freeze over.
Nevertheless, even this pitiful example of fiat science does not allow the authors to dismiss or explain away the essential and incontrovertible fact that all climate realists know:
“Studies of isotopes generated in the atmosphere by galactic cosmic rays show that the Sun has been exceptionally active during recent decades. This grand solar maximum has persisted for longer than most previous examples in the cosmogenic isotope record and is expected to end soon.” (Ibid.)
As a result, the authors essentially are begging you use your own common sense. They’re actually telling you to turn the title of their own study on its head if you want to discover the real truth. In other words, you must provide your own answer to the real question, that fascist academia will not allow the authors to print in big block letters, e.g.,
THE BIG QUESTION: Were the relatively warmer European winters over the last several decades associated with global warming the result of relatively higher — and indeed `exceptionally’ higher — solar activity?
THE ANSWER: If you are a climate realist, your answer will of course be: `Yes, it’s the sun, stupid.’
Abstract. Solar activity during the current sunspot minimum has fallen to levels unknown since the start of the 20th century. The Maunder minimum (about 1650-1700) was a prolonged episode of low solar activity which coincided with more severe winters in the United Kingdom and continental Europe. Motivated by recent relatively cold winters in the UK, we investigate the possible connection with solar activity. We identify regionally anomalous cold winters by detrending the Central England temperature (CET) record using reconstructions of the northern hemisphere mean temperature. We show that cold winter excursions from the hemispheric trend occur more commonly in the UK during low solar activity, consistent with the solar influence on the occurrence of persistent blocking events in the eastern Atlantic. We stress that this is a regional and seasonal effect relating to European winters and not a global effect. Average solar activity has declined rapidly since 1985 and cosmogenic isotopes suggest an 8% chance of a return to Maunder minimum conditions within the next 50 years (Lockwood 2010 Proc. R. Soc. A 466 303-29): the results presented here indicate that, despite hemispheric warming, the UK and Europe could experience more cold winters than during recent decades.
[Lockwood M, Harrison RG, Woollings T, Solanki SK. Are cold winters in Europe associated with low solar activity? Environ. Res. Lett. 5 (April-June 2010) 024001]
http://iopscience.iop.org/1748-9326/5/2/024001/fulltext

Bob_FJ
May 19, 2010 5:00 pm

For those suggesting that Venus winds redistribute energy uniformly around the planet, here is an interesting report, (Sep 2008, my bold added):
EXTRACT: Firstly, between the equator and the median latitudes of the planet there dominates a superotation with constant winds blowing from East to West, within the clouds decreasing speed with height from 370 km/h to 180 km/h.
At these median latitudes, the winds decrease to a standstill at the pole, where an immense vortex forms. Other aspects of the superrotation that observations with VIRTIS have made possible are that the meridional (North – South) movements are very weak, about 15 km/h, and, secondly, unlike what was previously believed, the superotation appears to be not so constant over time: “We have detected fluctuations in its speed that we do not yet understand”, stated the scientists.
Moreover, for the first time they observed “the solar thermal tide” effect at high latitudes on Venus. “The relative movement of the Sun on the clouds and the intense heat deposited on them makes the superotation more intense at sunset than at sunrise”, they stated.

Very interesting; but if the north-south winds are weak, how can they possibly effectively evenly redistribute energy from the hotspot under the sun to the high latitudes? Up there, surviving sunlight is nominally spread over a much larger unit area, after nominally travelling through a much greater depth of atmosphere and clouds. (per the latitudinal cosecant; e.g. 1.414 at 45 degrees, year average.)
Strange place Venus, what?!

evilincandescentbulb
May 23, 2010 10:14 am

An anti-humanist world government thwarted by George Bush?
In the end, it was all about the money, George Bush saved the world.
We all see now that a non-existing of problem — global warming — was precisely created to be the fear so big that only world government could tackle the problem, right?
There was a grand attempt to quickly stampede the herd by the UN. And, supporting the hoax were the obliging secular socialist organs of Western society, the mainstream media and the governmental-education machine and union.
But reality and reason has in the end seen through the AGW hoax as a scientific fraud and also puts a spotlight on a continuing threat to the priciple that all humanity has a God-given right to liberty: the money and power grab by big government liberal fascism under color of global warming.
“Banks and investors are pulling out of the carbon market after the failure … [at] Copenhagen … Carbon financiers have already begun leaving banks in London because of the lack of activity and the drop-off in investment demand…
“Banks had been scaling back their plans to invest in carbon markets before Copenhagen. Fewer new clean energy projects need to be financed as, because of the recession, there are fewer global emissions to offset. The price of carbon credits has also fallen, while plans to introduce national trading schemes, particularly in the US and Australia, remain uncertain…
“Carbon markets were central to the Kyoto Protocol, which expires in 2012 and obliged developed countries that exceed their targets to purchase credits from clean energy projects in the developing world. Policymakers will meet again in Mexico in November in an attempt to revive the climate change talks.”
http://www.guardian.co.uk/environment/2010/jan/24/carbon-emissions-green-copenhagen-banks

Bob_FJ
May 23, 2010 3:48 pm

It seems that Steve’s hypothesis is dependent on convective circulation such that the descending gases are in a constant cycle of recompression. It has also been suggested that convection is likely to be higher on Venus than Earth. Here are some data which supports that speculation. They show that thermal conductivity of pure CO2 under Venus conditions is considerably higher than the air on Earth, which means that heat transfer into and through the gases is more rapid, and hence convection should be faster and have greater impetus. (although it is not completely true to apply; AOTBE)
(I hope the formatting of the table works out OK)
Height (km) Ref: [a]
——–Temp. K Ref: [a]
————– Atmospheric pressure (bar) Ref: [a]
——————————–Thermal conductivity (mW/MK) Ref: [b]
0—- 735— 92.10— ~56
5—- 697— 66.65— ~52
10— 658— 47.39— ~47
15— 621— 33.04— ~43
20— 579— 22.52— ~42
25— 537— 14.93— ~36 Sulphuric haze & water vapour above Ref: [d]
30— 495— 9.851— ~33
35— 453— 5.917— ~29
40— 416— 3.501— ~25
45— 383— 1.979— ~23
……………………………… Somewhat Earth-like above 50 Km
50— 348— 1.066— ~21 Sulphuric clouds & water vapour above. T= 75C
55— 300— 0.531— ~17 Temperature, T = 23C
60— 263— 0.236— ~17 Sulphuric clouds & water vapour above. T= -10C
~~~~~~~~~~~~~~~~~~~~~~~~~~~~ Compare with:
0—- 273— 1.032—- 14.7.…….. Earth Air at zero C Ref: [c]
[a] http://en.wikipedia.org/wiki/Atmosphere_of_Venus
[b] http://www.nist.gov/srd/PDFfiles/jpcrd723.pdf
[c] http://encyclopedia.airliquide.com/Encyclopedia.asp?GasID=26
[d] http://en.wikipedia.org/wiki/File:Venusatmosphere.svg

evilincandescentbulb
May 27, 2010 7:38 am

AGW prognosticating was a smithy’s craft. In an age of technology, reason and hope, they traded for sheepskins by pounding out the coffin nails that were used to bury science.

June 12, 2010 1:27 am

Much later than everyone else.. a new article on Science of Doom:
Venusian Mysteries